qcm de mathÉmatiques gÉomÉtrie probabilitÉsgilbert.damin.free.fr/qcm/tome2.pdf · research inc...

144
BACCALAURÉAT CONCOURS ENTRÉE À L’UNIVERSITÉ QCM de MATHÉMATIQUES GÉOMÉTRIE - PROBABILITÉS Gilbert DAMIN Agrégé de Mathématiques Professeur en Classe Préparatoire au Lycée Pierre de Fermat à Toulouse

Upload: trinhphuc

Post on 06-Feb-2018

223 views

Category:

Documents


1 download

TRANSCRIPT

BACCALAURÉAT

CONCOURS

ENTRÉEÀ L’UNIVERSITÉ

QCM de MATHÉMATIQUES

GÉOMÉTRIE-

PROBABILITÉS

Gilbert DAMINAgrégé de Mathématiques

Professeur en Classe Préparatoire

au Lycée Pierre de Fermat à Toulouse

Chez le même éditeur

• Questionnements Automatisables de Mathématiques aux Concours ENAC, 90 / 92,Gilbert DAMIN / Serge DAUDE 240 pages (1992).

• QCM de Mathématiques, ANALYSE, Baccalauréat et Entrée à l’Université,Gilbert DAMIN 128 pages (1996).

INTRODUCTION

L’utilisation des QCM (Questionnaires à Choix Multiples) est de plus en plus fréquentedans les examens et concours, ceci à tous niveaux et pour un nombre croissant de matières.

Pour plus d’information, voir le tome d’analyse chez le même éditeur ou consulter surInternet le site de l’auteur http://gilbert.damin.free.fr

Actuellement, venu d’outre atlantique des batteries de tests QCM se développentrapidement. Ils permettent de déterminer le niveau de connaissance et de compréhensiondans un domaine technique particulier. Les formations du prochain millénaire serontcertainement de ce type.

L’Europe aussi, met en place sur Internet un système d’évaluation et d’accréditation descompétences sous la forme de QCM, la mobilité en dépend.

Tout l’enjeu et le bénéfice de la Validation des Acquis Professionnels seront liés à cesdifférents modes d’évaluation.

Nous rappelons que nous avons classés pour notre commodité les QCM selon les troistypes en usage dans la plupart des ouvrages, des examens, ou des concours et que nousdéveloppons dans cet ouvrage le type III (Analyse de l’ensemble des assertions d’unemême question et plusieurs questions liées).

Il faut noter que pour tout type de QCM la non réponse est préférable à une réponse auhasard, en raison des pénalisations des réponses fausses. Ces pénalités sont, en principe,calculées de telle manière qu’un candidat répondant au hasard obtiendra toujours unenote plus ou moins négative.

Nous espérons que ce livre vous aidera à découvrir les Questionnements Automatis-ables et vous montrera que la rigueur et la méthode sont les seuls moyens pour faire desprogrès en mathématiques.

Ce livre a été écrit par l’auteur avec le logiciel c°Scientific Work Place [1] de TCI SoftwareResearch Inc qui permet d’écrire en c°LaTEX [2] sans en connaître toute la syntaxe. Toutesles figures ont été réalisées avec c°PICTEX [3].

[1] Voir le site http://mackichan.com[2] Langage très utilisé qui permet d’écrire des mathématiques et bien d’autres choses. Voir par exemplele site de l’association ASTEX http://www.univ-orleans.fr/EXT/ASTEX[3] Voir le site http://www.ucc.ie/info/Tex/pictex.html

QUELQUES CONSEILS

De manière pratique, il est conseillé de lire pour chaque exercice proposé dans ce livre latotalité des questions, puis de traduire en énoncé classique ce qui est demandé, et aprèsavoir traité l’exercice complètement, de répondre aux questionnaires QCM.

Cela sera comparable à vos habitudes et fera un excellent entraînement pour aussi bienles sujets classiques que les sujets QCM, et ceci pour tout type.

Ces exercices s’adressent aux élèves de Terminale Scientifique qui désirent préparer dansde bonnes conditions leur accès Post-Baccalauréat ou des concours.

Un dernier conseil :

Lire très attentivement chaque assertion, car elle peut renfermer un piège. Tous les motsont leur importance et ont une signification précise.

Il est ainsi possible de juger votre niveau de compréhension littéraire d’un texte scientifique.

Nous rappelons que dans les Questionnements Automatisables qui suivent, seul au plusdeux réponses sont exactes et que certaines questions sont liées. Ainsi il est fréquent qu’unemême question soit posée plusieurs fois sous des formes différentes, ceci afin d’éliminer lesréponses au hasard.

Cela veut dire que chaque question comporte de zéro à deux assertions exactes.

Dans le cas où aucune des quatre assertions proposées n’est jugée exacte par le candidat,celui ci doit cocher la case e de la question. Ce cas est le plus difficile car il est déroutant,prudence.

Un exemple :

Question 1 :La valeur du produit 2 * 3 esta) −1. b) 6. c) 4. d) 0.

Question 2 :Les racines de l’équation x2 − 1 = 0 sonta) −1. b) i. c) 1. d) −2.

Question 3 :La somme 2− 5 vauta) −2. b) 3. c) 5. d) 1.

a b c d eQ1 :

Q2 :

Q3 :

............................................................................................ ............................................

................................................ ............................................................................................ ............................................

................................................

............................................................................................ ............................................

................................................ ............................................................................................

............................................................................................ ............................................

................................................ ............................................................................................ ............................................

................................................

..........................................................................................................

..................

.

................................

.

................................

.

................................

.

................................

.

................................

.

................................

.

................................

.

................................

.

................................

.

................................

.

................................

.

................................

.

................................

.

................................

.

.........................................................................................................................................................................................................................................................................................................................................................................................................................................................................................................................................................................................................................................................................................................................................................................................................................................................................................................................................................................................................................................................................................................................................................................................................................................................................................................................................................................................................................................

..........................................................................................................

..................

.

................................

.

................................

.

................................

.

................................

.

................................

.

................................

.

................................

.

................................

.

................................

.

................................

.

................................

.

................................

.

................................

.

................................

.

......................................................................................................................................................................................................................................................................................................................................................................................................................................................................................................................................................................................................................................................................................................................................................................................................................................................................................................................................................................................................................................................................................................................................................................................................................................................................................................................................................................................................................................... ............................................

........................................................................................................................................................................................................................................................................................................................................................................................................................................................................................................................................................................................................................................................................................................................................................................................................................................................................................................................................................................................................................................................................................................................................................................................................................................................................................................................................................................................................................................................................................................................................................................................................................................................................................................................................................................................................................................................................................................................................................................................................

..........................................................................................................

.................

.................................

.................................

.................................

.................................

.................................

.................................

.................................

.................................

.................................

.................................

.................................

.................................

.................................

.................................

...........................................................................................................................................................................................................................................................................................................................................................................................................................................................................................................................................................................................................................................................................................................................................................................................................................................................................................................................................................................................................................................................................................................................................................................................................................................................................................................................................................................................................................................

UN PEU DE MÉTHODE

Le problème [1] qui suit a été posé au baccalauréat dans les termes suivants :

Nous considérons l’application f de C dans lui même définie par

f : z → Z =z + i λ z

1 + i λ

où z est le conjugué de z et λ un paramètre réel strictement positif.

Pour quelles valeurs de λ l’application f est-elle bijective ? Exprimer alors f−1 (z) enfonction de z et de z.

1. Nous appelons m, m0 et M les images respectives de z, z et Z dans le plan complexeet φ la transformation affine ponctuelle qui au point m fait correspondre le point M,ie : φ (m) =M.

a) Déterminer l’ensemble E des points invariants par φ ie : N du plan tels queφ (N) = N.

b) Calculer pour z /∈ E le module et l’argument du nombre complexeζ =

Z − z

Z − z.

En déduire pour z fixé non réel, l’ensemble des points M lorsque le paramètre λvarie.

2. Dans cette question λ = 2. Calculer les coordonnées (X, Y ) de M en fonction descoordonnées (x, y) de m, préciser l’image par φ de la droite d’équation x = 1.

Nous proposons de résoudre ce problème en répondant auxQuestionnements Automa-tisables suivants.

Nous considérons l’application f de C dans lui même définie par

f : z → Z = f(z) =z + i λ z

1 + i λ

[1] Note du l’auteur : Ce problème a été traité simultanément de manière classique et par QCM par 157élèves de 6 terminales.

Les résultats ont établi une corrélation parfaite entre les deux séries de notes.

C’est à la suite de plusieurs expériences concordantes de ce type que les Questionnements Automatisablesfurent validés pour certains concours.Pour plus d’information, voir du même auteur “ Un grand pas vers le FUTUR, Docimologie en QCM ”publié au CNDP de Toulouse 1986.

6 Questionnements Automatisables

où z est le conjugué de z et λ un paramètre réel strictement positif.

Question n 01 :

L’application f est bijective si λ est

a) nul b) égal à 1. c) différent de 1. d) différent de −1.

Nous écrivons f−1 (z) sous la forme f−1 (z) = α z + β z.

Question n 02 :

La valeur de α est

a)1

1 + i λ. b)

1

1− λ2. c)

i λ

1− λ2. d)

1 + i λ

1− λ2.

Question n 03 :

La valeur de β est

a)−i λ (1− i λ)

1− λ2. b)

i λ

1− i λ. c) −λ (λ+ i)

1− λ2. d)

λ

1 + i λ.

Nous désignons par φ la transformation affine ponctuelle qui au point m d’affixe z faitcorrespondre le point M d’affixe Z = f(z) ie : φ (m) =M.

L’ensemble des points invariants par φ est l’ensemble des points N du plan tels queφ (N) = N.

Question n 04 :

L’ensemble des points E invariants par φ esta) l’axe des imaginaires. b) l’axe des réels.

c) l’ensemble vide. d) une droite.

Nous définissons pour z /∈ E le nombre complexe

ζ =Z − z

Z − z.

Question n 05 :

Si ζ est défini alors son module |ζ| a pour expression

a)√λ. b) −λ. c)

1

λ. d) λ.

Question n 06 :

L’argument arg ζ a pour valeur (k désigne un entier relatif)

a) −π2à 2 k π près. b)

π

2à 2 k π près.

c) −π2à k π près. d)

π

2à k π près.

Question n 07 :

Si z est fixé, tel que =m z > 0, alors l’ensemble des points M est

Introduction 7

a) un demi-cercle situé à droite d’undiamètre vertical.

b) un demi-cercle situé à gauche d’undiamètre vertical.

c) situé sur un cercle. d) un segment de droite

Dans toutes les questions suivantes λ = 2. Soient (X, Y ) les coordonnées de M et (x, y)celles de m.

Question n 08 :

L’expression de X est

a)4x+ 5 y

5. b)

5x+ 4 y

5. c)

4x− 5 y5

. d)5x− 4 y

5.

Question n 09 :

L’expression de Y est

a)3 y

5. b) −3 y

5. c)

5x− 4 y5

. d) −5x+ 4 y5

.

Question n 10 :

L’image par φ de la droite d’équation x = 1 est

a) un cercle de centre O. b) une droite passant par O.

c) un cercle non centré en O. d) une droite ne passant pas par O.

Corrigé

Nous allons résoudre ce problème tel qu’il était posé de manière classique, puis nousrépondrons aux assertions proposées. Ceci nous permettra de mieux comprendre laconstruction des Questionnements Automatisables et nous prouvera que cettenouvelle approche est bien identique à l’approche classique d’un problème. Nous voyonsainsi la différence entre les QCM et les Questionnements Automatisables.

Vous comprenez maintenant le pourquoi d’un nouveau nom.

1. Pour déterminer si f est une application bijective de C dans lui même, le plus simpleest d’exprimer z en fonction de Z.

Z =z + i λ z

1 + i λ⇔ (1 + i λ) Z = z + i λ z

soit en prenant le conjugué de la deuxième expression

(1− i λ) Z = z − i λ z

et en éliminant z entre les deux égalités (il suffit de multiplier la seconde par i λ etd’ajouter ensuite) ¡

1− λ2¢z = (1 + i λ) Z − i λ (1− i λ) Z. (1)

Il est clair que z ne peut être calculé que si et seulement si λ2 6= 1, ce qui équivaut àλ 6= 1 car par hypothèse λ est un paramètre réel strictement positif.Nous en déduisons que f−1 existe pour λ 6= 1 et que son expression est

f−1 (z) =1 + i λ

1− λ2z − i λ (1− i λ)

1− λ2z. (2)

8 Questionnements Automatisables

........

........

........

........

........

........

........

........

........

........

........

........

........

........

........

........

........

........

........

........

........

........

........

........

........

........

........

........

........

........

........

........

........

........

........

........

........

........

........

........

........

........

........

........

........

........

........

........

........

........

........

........

........

........

........

........

........

........

........

........

........

........

........

........

........

................................

..........................i y

.................................................................................................................................................................................................................................................................................................................................................................................... ..........................x

O

−π/2

m

m0

M

...........................................................................................................................................................................................................................................................................................................................................................................................................................................................................

........................................

........................................

........................................

........................................

.........

......................

.................

................................................

.....

....

.....

....

.....

....

.....

....

.....

....

.....

....

.....

....

.....

....

.....

....

.....

....

.....

....

.....

....

.....

....

.....

....

.....

.....

.....

....

.....

....

.....

....

.....

....

.....

....

.....

....

.....

....

.....

....

.....

....

.....

....

.....

....

.....

....

.....

.....

..

****************************************************************************************************************************

************************************************************************************************************************************************************************************************************************************************************************************************************************************************************************************************************************************************************************************************************************************************************************************************************************************************************************************************************************************************************************************************************************************************************************************************************************************************************************************************************************************************************************************************************************************************

*****************************

**********************************

****************************************

***************************************************

******************************************************************************************************

************************

Cas où =m z > 0

Figure 1

........

........

........

........

........

........

........

........

........

........

........

........

........

........

........

........

........

........

........

........

........

........

........

........

........

........

........

........

........

........

........

........

........

........

........

........

........

........

........

........

........

........

........

........

........

........

........

........

........

........

........

........

........

........

........

........

........

........

........

........

........

........

........

........

........

................................

..........................i y

.................................................................................................................................................................................................................................................................................................................................................................................... ..........................x

O

−π/2

m

m0

M

.........................................................................................................................................................................................................................................................................................................................................................................................................................................................................................................................................................................................................................................................................................

..............

.........................................................................

.....

....

.....

....

.....

....

.....

....

.....

....

.....

....

.....

....

.....

....

.....

....

.....

....

.....

....

.....

....

.....

....

.....

....

.....

.....

.....

....

.....

....

.....

....

.....

....

.....

....

.....

....

.....

....

.....

....

.....

....

.....

....

.....

....

.....

....

.....

.....

..********************************************************************************************************************************************************************************************************************************************************************************************************************************************************************************************************************************************************************************************************************************************************************************************************************************************************************************************************************************************************************************************************************************************************************************************************************************************************************************************************************************************************************************************************************************************************************************************************************************************************************************************************************************************************************************************************************************************************************************************************************************************************************

Cas où =m z < 0

Figure 2

2. Nous pouvons aussi exprimer la relation (1) d’une autre manière en développant lenumérateur du deuxième terme

f−1 (z) =1

1− i λz − λ (λ+ i)

1 + i λz. (3)

a) Le point N d’affixe z est invariant par φ si et seulement si f (z) = z. Ce qui donnela relation

z =z + i λ z

1 + i λ⇔ i λ (z − z) = 0 (4)

soit z − z = 0 car λ > 0.

L’ensemble des points invariants par φ est donc l’axe des réels (Remarquer que cerésultat est vrai pour tout λ).

b) Nous avons

ζ =Z − z

Z − z

=

z + i λ z

1 + i λ− z

z + i λ z

1 + i λ− z

= −i λ z − z

z − z= −i λ car z est non réel ie : z /∈ E .

Nous en déduisons que

|ζ| = λ et arg ζ = −π2à 2 k π près [1] avec k ∈ Z. (5)

Si z est non réel fixé alors les points m, m0 et M d’affixes respectives z, z et Zvérifient d’après (5)

mM

m0M= λ et

³−−−→m0M,

−−→mM

´= −π

2,

[1] Nous écrivons aussiarg ζ = −π

2(mod 2π) .

Introduction 9

exprimant lorsque λ varie que le point M décrit un demi cercle de diamètre mm0.La position de ce demi cercle dépend de =m z, voir figure 1 et figure 2.

3. Pour λ = 2, nous isolons la partie réelle et la partie imaginaire de Z = f (z)

X + i Y =x+ i y + 2 i (x− i y)

1 + 2 isoit en multipliant et en divisant par la quantité conjuguée du dénominateur 1− 2 i

X + i Y =[x+ i y + 2 i (x− i y)] (1− 2 i)

5

=5x+ 4 y

5− i

3 y

5.

D’où la partie réelle X et la partie imaginaire Y sont

X =5x+ 4 y

5et Y = −3 y

5. (6)

f étant bijective car λ = 2 6= 1, nous pouvons exprimer x et y en fonction de X et deY, à partir de (6).

x =1

3(3X + 4Y ) et y = −5

3Y.

Nous avons ainsi pour x = 1 la relation 3X + 4Y = 3 vérifiée par les coordonnées dupoint M.

L’ensemble de ces points est la droite dont une équation est

3x+ 4 y − 3 = 0. (7)

Nous pouvons maintenant répondre à toutes les assertions des 10 questionsposées.

D’après la relation (1) , nous pouvons calculer z si et seulement si λ2 6= 1, donc l’assertion1 c) est vraie et les trois autres fausses.Nous avons les réponses :

Question 01 : a) Fausse b) Fausse c) Vraie d) Fausse e) Fausse

D’après la relation (2) , la valeur de α est1 + i λ

1− λ2, donc l’assertion 2 d) est vraie et les

trois autres fausses.

Nous avons les réponses :

Question 02 : a) Fausse b) Fausse c) Fausse d) Vraie e) Fausse

Nous devons être prudent dans les assertions de la question 3. En effet le dénominateur

n’est pas toujours réel. D’après les relations (2) et (3) , la valeur de β est−i λ (1− i λ)

1− λ2

ou −λ (λ+ i)

1− λ2donc les assertions 2 a) et 2 c) sont vraies et les deux autres fausses.

La relation (3) permet de gagner des points. Il faut être rigoureux dans les Question-nements Automatisables.

Nous avons les réponses :

Question 03 : a) Vraie b) Fausse c) Vraie d) Fausse e) Fausse

10 Questionnements Automatisables

D’après la relation (4) , l’ensemble des points invariants par φ est l’axe réel, donc lesassertions 4 b) et 4 d) sont vraies et les deux autres fausses. Encore un piège comme à laquestion précédente, c’est comme un fusil à répétition.

Nous avons les réponses :

Question 04 : a) Fausse b) Vraie c) Fausse d) Vraie e) Fausse

Vous avez remarqué que le calcul de ζ exige que z − z 6= 0 donc la réserve du préambulede la question 5 est parfaitement justifié et donc dans la suite il faut prendre z non réel(voir ce qui se passe dans le cas contraire pour les deux figures).

D’après la relation (5) , les assertions 5 d) et 6 a) sont vraies et les six autres fausses.

Nous avons les réponses :

Question 05 : a) Fausse b) Fausse c) Fausse d) Vraie e) Fausse

Question 06 : a) Vraie b) Fausse c) Fausse d) Fausse e) Fausse

D’après la relation (5) pour z fixé, tel que =m z > 0, ce qui est le cas de la figure 1,l’ensemble des points M est le demi-cercle de diamètre mm0 situé dans la partie droite.Les assertions 7 a) et 7 c) sont donc vraies et les deux autres fausses. Remarquer le piègede l’assertion 7 c).

Nous avons les réponses :

Question 07 : a) Vraie b) Fausse c) Vraie d) Fausse e) Fausse

D’après la relation (6) , les assertions 8 b) et 9 b) sont vraies et les six autres fausses.

Nous avons les réponses :

Question 08 : a) Fausse b) Vraie c) Fausse d) Fausse e) Fausse

Question 09 : a) Fausse b) Vraie c) Fausse d) Fausse e) Fausse

Enfin, la relation (7) , montre que l’image par φ de la droite x = 1 est une droite nepassant pas par O, donc l’assertion 10 d) est vraie et les trois autres fausses.

Nous avons les réponses :

Question 10 : a) Fausse b) Fausse c) Fausse d) Vraie e) Fausse

Vous connaissez dorénavant quelques règles de construction des QuestionnementsAutomatisables, vous pouvez en faire vous même.

Bon travail.

ÉNONCÉS

Enoncés 13

01

Un peu de logique :

Sophie, Anne-Marie et Christelle dans une même classe de terminale, ont chacune desamis dans une autre classe de terminale. Sachant que :

• Aucun ami de Sophie n’ignore le théorème de Pythagore.• Aucun ami de Anne-Marie ne connaît le théorème de Pythagore.• Tout ami de Christelle connaît le théorème de Pythagore.

Question n 01 :

Nous pouvons en déduire que

a) Aucun ami de Christelle n’est ami d’Anne-Marie.b) Pour être à la fois ami de Sophie et de Christelle, il ne faut pas connaître le théorèmede Pythagore.

c) Pour être à la fois ami de Sophie et de Christelle, il ne faut pas connaître le théorèmede Pythagore.

d) Tout ami de Anne-Marie ne connaît pas le théorème de Pythagore.

Encore un peu de logique :

Dans un QCM de contrôle continu donné dans une classe de terminale, il est précisépour une question ayant quatre affirmations notées (a0), (b0), (c0) et (d0) , qu’une seulede ces quatre affirmations est exacte.

Un bon élève à remarqué (avec raison) que pour cette question :

• Si (b0) est vraie alors (c0) l’est aussi.• Si (a0) est vraie alors au moins l’une des affirmations (b0) ou (d0) est vraie (et peut-être les deux).

• Enfin (d0) est fausse si et seulement si (c0) est vraie.

Question n 02 :

Nous pouvons en déduire que

a) Il est possible que (d0) soit vraie. b) (d0) est fausse.c) (c0) est nécessairement faux. d) (a0) est vraie.

14 Questionnements Automatisables

02

Dans le plan affine euclidien P rapporté au repère orthonormé Oxy, on donne les pointsA(2, 0) et B(0,m), où m est un paramètre réel non nul.

La droite perpendiculaire en A à la droite AB coupe l’axe Oy en C et la parallèle àl’axe Ox passant par B en D.

Question n 01 :

Si nous associons au plan P le plan complexe alors l’affixe du point A est zA = 2 et celle dupoint B est zB = mi.

a) <e zD = <e zB car la droite BD est parallèle à l’axe Ox.

b) zB − zA = i (zC − zA) car l’angle³−→AC,

−→AB

´est droit.

c) zD =m2 − 42

+mi. d) <e zC = 0 et =mzD = −m.

Question n 02 :

Nous avons les relations

a) AC ×AD = AB2. b) AC2 +AB2 = BD2.

c) AB × CD = BD ×BC. d) CA× CD = AB2.

Dans les deux questions suivantes, on suppose m = 4.

Question n 03 :

Une équation de la droite CD est

a) y =x

2− 2. b) y =

x

2− 1.

Les coordonnées de C et D sont

c) C(0,−1) et D(4,−10). d) C(0, 1) et D(10, 4).

Question n 04 :

D peut être considéré comme le barycentre des points A, B et C affectés respectivementdes coefficients α, β et γ.

a) Les points A, C et D étant alignés, alors α, β et γ n’existent pas.

Dans le cas contraire, nous pouvons choisir

b) α = 5, β = 1, γ = −4. c) α = 5, β = 0, γ = −4. d) α = 2, β = 0, γ = 1.

Dans la question suivante, on suppose m quelconque, non nul.

Question n 05 :

Soit I le point de concours des médianes du triangle ABC, ses coordonnées sont

a)µ2

3,m2 − 43m

¶. b)

µ1

3,m2 − 43m

¶.

Enoncés 15

L’ensemble des points I lorsque m varie dans R∗ est surc) une droite. d) un cercle.

Question 06 :

La translation t de vecteur−→CA transforme le triangle CAB en un triangle AFE et le point

I en un point J.

a) AFE est rectangle en A.b) J est le point de concours (orthocentre) des hauteurs du triangle AFE.c) La longueur du segment [IJ ] est constante.d) Le point J est situé sur un droite lorsque m décrit R∗+.

03

Dans le plan affine euclidien P, on considère le triangle ABC rectangle en A, tel queAC = 2AB = 2m. (m réel strictement positif).

Soit I (respectivement J), le barycentre des points A, B et C, affectés des coefficients2,−2 et 1 (respectivement −2, 3 et 3).

On note D le point défini par−−→AD =

−→AB +

−→AC.

Question n 01 :

a) Le quadrilatère ABDC est un carré car les diagonales AD et BC sont perpendicu-laires.

b) Le points de concours des droites AD et BC est l’isobarycentre des points A, B, Cet D.

c) Les quatre points A, B, C et D sont cocycliques.d) Le triangle BCD est rectangle en B.

Question n 02

a)−→CI = 2

−−→DC. b)

−→AI = 2

−−→BC.

c)−→DI = 3

−→AB. d)

−→AJ =

3

4

−−→AD.

Question n 03

a) La droite IB coupe le segment [AC] enson milieu.

b) Les droites IA et CJ sont parallèles.

c) Les droites IB et CJ sont parallèles. d) Les droites AD et CJ sont perpendic-ulaires.

Question n 04 :

Nous associons au plan affine P le plan complexe. Soit s la similitude définie pars : z −→ Z = a z + b ,

telle que s(B) = A et s(A) = I.

a) Les conditions données ne permettent pas de déterminer la similitude s car il manqueune donnée.

16 Questionnements Automatisables

Si l’assertion a) est rejetée alors

b) a = 2− 2 i. c) b = ma. d) s(D) = J.

Question n 05 :

Soit Ω le point invariant de la similitude s.

a) Ω est l’intersection des médiatrices dessegments [AB] et [AI].

b) Les angles³−→ΩB,

−→ΩA´et³−−→ΩA,

−→ΩI´

sont égaux.c) La droite ΩA est une bissectrice del’angle

³−−→ΩB,

−→ΩI´.

d) Si K est l’intersection des droites ΩBet AI alors le triangle AΩK est isocèle.

Question n 06

L’ensemble E1 des points M tels que

4°°°2−−→AM − 2−−→BM +

−−→CM

°°° = °°°−2−−→AM + 3−−→BM + 3

−−→CM

°°°est

a) la médiatrice du segment [AJ ]. b) la médiatrice du segment [JI].

c) le cercle de diamètre AJ. d) le cercle de diamètre JI.

Question n 07

L’ensemble E2 des points M tels que

2AM2 − 2BM2 + CM2 = −5m2

est

a) un cercle passant par C. b) inclus (strictement) dans E1.

c) un cercle de centre I. d) une droite passant par J.

04

Le plan est rapporté au repère orthonor-mal Oxy (Unité = 1 cm). C1 ∪ C3 est lacourbe d’équation y =

1

x(C1 correspond

à x > 0 et C3 à x < 0) et C2 ∪ C4 est lacourbe d’équation y = −1

x(C2 correspond

à x > 0 et C4 à x < 0)

On note C = C1 ∪ C2 ∪ C3 ∪ C4. .........................................................................................................................................................................................................................................................................................................................................................

..........................

y

............................................................................................................................................................................................................................................................................................................................................................................................................................................................................................................... ..........................xo

−2 2

−2

2

A

B C

D

.............................................................................................................................................................................................................................................

.............................................................................................................................................................................................................................................

..................................................................................

...........................................................................................................................................................

............................................................................................................................................................................................

................................................................................................................

........

........

........

........

........

........

........

.......

...........................................................................................

...........................................................................................

..

..

..

..

.

..

..

..

..

..

..

..

..

..

.

..

..

..

..

..

..

..

..

..

..

..

..

..

..

..

.

..

..

..

..

..

..

..

..

..

..

.

..

..

..

..

..

..

..

..

..

..

..

.

..

..

..

..

..

..

..

..

..

..

..

.

..

..

..

..

..

..

..

..

..

..

..

..

.

..

..

..

..

..

..

..

..

..

..

..

..

..

.

..

..

..

..

..

..

..

..

..

..

..

..

..

..

.

..

..

..

..

..

..

..

..

..

..

..

..

..

..

..

..

.

..

..

..

..

..

..

..

..

..

..

..

..

..

..

..

..

.

..

..

..

..

..

..

..

..

..

..

..

..

..

..

..

..

..

..

.

..

..

..

..

..

..

..

..

..

..

..

..

..

..

..

..

..

..

..

..

..

..

..

..

..

..

..

..

..

..

..

..

.

..

..

..

..

..

..

..

..

..

..

..

..

..

..

..

..

..

..

..

..

..

..

..

..

.

..

..

..

..

..

..

..

..

..

..

..

..

..

..

..

..

..

..

..

..

..

..

..

..

..

..

..

..

.

..

..

..

..

..

..

..

..

..

..

..

..

..

..

..

..

..

..

..

..

..

..

..

..

..

..

..

..

..

..

..

.

..

..

..

..

..

..

..

..

..

..

..

..

..

..

..

..

..

.

..

..

..

..

..

..

..

..

..

..

..

..

..

..

..

..

..

..

..

..

..

..

..

..

..

..

..

..

..

..

..

..

..

..

..

.

..

..

..

..

..

..

..

..

..

..

..

..

..

..

..

..

..

..

..

..

..

..

..

..

..

..

..

..

..

..

..

..

..

..

..

.

..

..

..

..

..

..

..

..

..

..

..

..

..

..

..

..

..

..

..

..

..

..

..

..

..

..

..

..

..

..

..

..

..

..

..

.

..

..

..

..

..

..

..

..

..

..

..

..

..

..

..

..

..

..

..

..

..

..

..

..

..

..

..

..

..

..

..

..

..

..

..

.

..

..

..

..

..

..

..

..

..

..

..

..

..

..

..

..

..

..

..

..

..

..

..

..

..

..

..

..

..

..

..

..

..

..

..

.

..

..

..

..

..

..

..

..

..

..

..

..

..

..

..

..

..

..

..

..

..

..

..

..

..

..

..

..

..

..

..

..

..

..

..

..

..

..

..

..

..

..

..

..

..

..

..

..

..

..

..

..

..

.

..

..

..

..

..

..

..

..

..

..

..

..

..

..

..

..

..

..

..

..

..

..

..

..

..

..

..

..

..

..

..

..

..

..

..

.

..

..

..

..

..

..

..

..

..

..

..

..

..

..

..

..

..

..

..

..

..

..

..

..

..

..

..

..

..

..

..

..

..

..

..

.

..

..

..

..

..

..

..

..

..

..

..

..

..

..

..

..

..

..

..

..

..

..

..

..

..

..

..

..

..

..

..

..

..

..

..

.

..

..

..

..

..

..

..

..

..

..

..

..

..

..

..

..

..

..

..

..

..

..

..

..

..

..

..

..

..

..

..

..

..

..

..

.

..

..

..

..

..

..

..

..

..

..

..

..

..

..

..

..

..

..

..

..

..

..

..

..

..

..

..

..

..

..

..

..

..

..

..

.

..

..

..

..

..

..

..

..

..

..

..

..

..

..

..

..

.

..

..

..

..

..

..

..

..

..

..

..

..

..

..

..

..

..

..

..

..

..

..

..

..

..

..

..

..

..

.

..

..

..

..

..

..

..

..

..

..

..

..

..

..

..

..

..

..

..

..

..

..

..

..

..

.

..

..

..

..

..

..

..

..

..

..

..

.

..

..

..

..

..

..

..

..

..

..

..

..

..

..

..

..

..

..

..

..

..

.

..

..

..

..

..

..

..

..

..

..

..

..

..

..

..

..

..

..

..

.

..

..

..

..

..

..

..

..

..

..

..

..

..

..

..

..

..

..

..

..

..

..

..

..

..

.

..

..

..

..

..

..

..

..

..

..

..

..

..

..

..

.

..

..

..

..

..

..

..

..

..

..

..

..

..

.

..

..

..

..

..

..

..

..

..

..

..

..

..

.

..

..

..

..

..

..

..

..

..

..

..

.

..

..

..

..

..

..

..

..

..

..

..

.

..

..

..

..

..

..

..

..

..

..

..

.

..

..

..

..

..

.

..

..

..

..

..

..

..

..

..

.

..

..

..

..

..

..

..

..

..

.

..

..

..

..

..

..

..

..

..

.

Figure 4.1

Enoncés 17

Question n 01 :

La courbe C1 ∪ C2 admeta) O pour centre de symétrie. b) Oy pour axe de symétrie.

La courbe C1 ∪ C3 admetc) O pour centre de symétrie. d) Ox pour axe de symétrie.

Question n 02 :

Un point M du plan de coordonnées x et y appartient à C si et seulement sia) (xy − 1) (xy + 1) = 0. b) xy = |1|.c) |xy| = 1. d) x2y2 + 1 = 0.

A et B désignent les points de C4 d’abscisses respectives −2 et −12.

C et D désignent les points de C1 d’abscisses respectives 12et 2.

∆1 et ∆2 désignent les droites d’équations respectives y = x et y = −x.Question n 03 :

a) Les points A, B, C et D sont alignés. b) ABCD est un rectangle.

c) A et B sont symétriques par rapport àla droite ∆2.

d) B et D sont symétriques par rapport àla droite ∆1.

Question n 04 :

L’aire de la portion grisée A (en unités d’aire) est donnée par

a) 2Z 2

−2(|xy|− 1) dx. b) 4 ln 2.

c) 4 (1 + 2 ln 2). d) 4

ÃZ 1

0

dx+

Z 2

12

1

xdx

!.

On appelle g la fonction définie surl’intervalle [−2, 2] par sa courbe représen-tative G, voir ci-contre la figure 4.2.L’arc AB est inclus dans C4, l’arc CD estinclus dans C1 et BC est un segment.

Pour −2 6 x 6 2 nous notons

G(x) =

Z x

−2g(t) dt ,

(exprimé en unité d’aire).................................................................................................................................................................................................................................................................................................................................................................

..........................

y

......................................................................................................................................................................................................................................................................................................................................................................................................................................................................................................................................................................... ..........................xO

−2 2−12

12

A

B C

D

............................................................................................................................................................................................................................................................................................................................................................................................................................................................................................................................................................................................................................................................

.....

.....

.....

....

.....

....

.....

..

.....

.....

.....

....

.....

....

.....

....

.....

....

.....

....

.....

....

.....

....

.....

....

.....

....

.....

....

.....

....

.....

....

.....

.....

.....

..

.....

.....

.....

....

.....

....

.....

..

.....

.....

.....

....

.....

....

.....

....

.....

....

.....

....

.....

....

.....

....

.....

....

.....

....

.....

....

.....

....

.....

....

.....

.....

.....

..

Figure 4.2

Question n 05 :

a) G(−2) = −1. b) G

µ−12

¶= 0.

18 Questionnements Automatisables

c) Pour −2 6 x 6 −12

G(x) = ln

µ−2x

¶.

d) Pour −126 x 6 1

2

G(x) est constante.

Question n 06 :

a) Pour1

26 x 6 2G(x) = 2 + ln 23 − ln x.

b) G(2) est égale à l’aire de la portiongrisée A de la figure 4.1.

On considère les points de coordonnées I (−2, 0), J (0, 2) et K (2, 0).c) La droite IJ est la tangente à G aupoint d’abscisse +1.

d) Le triangle IJK est un trianglerectangle et isocèle.

Question n 07 :

On considère l’aire A0 comprise entre le segment [IJ ] et la courbe G pour x compris entre−2 et 0.

En unités d’aire

a) A0 =Z 0

−2[g(x)− x+ 1] dx. b) A0 = 2

Z 1

0

[g(x) + x− 1] dx.

c) A0 = A4. d) A0 = 2 ln 2 + 1.

05

Nous considérons dans le repère orthonorméOxy le quadrilatrère OABC représenté ci-contre.

Nous notons D2 la droite passant par lespoints A et B, et par D1 la droite passantpar les points B et C.

........

........

........

........

........

........

........

........

........

........

........

........

........

........

........

........

........

........

........

........

........

........

........

........

........

........

........

........

........

........

........

........

.................................

..........................

y

................................................................................................................................................................................................................................................................................................................................................................................................................................................................................... ..........................xO

3 4

1

2

A

B

C

................................................................................................................................................................................................................................................................................................................................................................................................

........................................................................................................................................................................................................................................................................................

.

.

.

.

.

.

.

.

.

.

.

.

.

.

.

.

.

.

.

.

.

.

.

.

.

.

.

.

.

.

.

.

.

.

.

.

.

.

.

.

.

.

.

.

.

.

.

.

.

.

.

.

.

.

.

.

.

.

.

.

.

.

.

.

.

.

.

.

.

.

.

.

.

.

.

.

.

.

.

.

.

.

.

.

.

.

.

.

.

.

.

.

.

.

.

.

.

.

.

.

.

.

.

.

.

.

.

.

.

.

.

.

.

.

.

.

.

.

.

.

.

.

.

.

.

.

.

.

.

.

.

.

.

.

.

.

.

.

.

.

.

.

.

.

.

.

.

.

.

.

.

.

.

.

.

.

.

.

.

.

.

.

.

.

.

.

.

.

.

.

.

.

.

.

.

.

.

.

.

.

.

.

.

.

.

.

.

.

.

.

.

.

.

.

.

.

.

.

.

.

.

.

.

.

.

.

.

.

.

.

.

.

.

.

.

.

.

.

.

.

.

.

.

.

.

.

.

.

.

.

.

.

.

.

.

.

.

.

.

.

.

.

.

.

.

.

.

.

.

.

.

.

.

.

.

.

.

.

.

.

.

.

.

.

.

.

.

.

.

.

.

.

.

.

.

.

.

.

.

.

.

.

.

.

.

.

.

.

.

.

.

.

.

.

.

.

.

.

.

.

.

.

.

.

.

.

.

.

.

.

.

.

.

.

.

.

.

.

.

.

.

.

.

.

.

.

.

.

.

.

.

.

.

.

.

.

.

.

.

.

.

.

.

.

.

.

.

.

.

.

.

.

.

.

.

.

.

.

.

.

.

.

.

.

.

.

.

.

.

.

.

.

.

.

.

.

.

.

.

.

.

.

.

.

.

.

.

.

.

.

.

.

.

.

.

.

.

.

.

.

.

.

.

.

.

.

.

.

.

.

.

.

.

.

.

.

.

.

.

.

.

.

.

.

.

.

.

.

.

.

.

.

.

.

.

.

.

.

.

.

.

.

.

.

.

.

.

.

.

.

.

.

.

.

.

.

.

.

.

.

.

.

.

.

.

.

.

.

.

.

.

.

.

.

.

.

.

.

.

.

.

.

.

.

.

.

.

.

.

.

.

.

.

.

.

.

.

.

.

.

.

.

.

.

.

.

.

.

.

.

.

.

.

.

.

.

.

.

.

.

.

.

.

.

.

.

.

.

.

.

.

.

.

.

.

.

.

.

.

.

.

.

.

.

.

.

.

.

.

.

.

.

.

.

.

.

.

.

.

.

.

.

.

.

.

.

.

.

.

.

.

.

.

.

.

.

.

.

.

.

.

.

.

.

.

.

.

.

.

.

.

.

.

.

.

.

.

.

.

.

.

.

.

.

.

.

.

.

.

.

.

.

.

.

.

.

.

.

.

.

.

.

.

.

.

.

.

.

.

.

.

.

.

.

.

.

.

.

.

.

.

.

.

.

.

.

.

.

.

.

.

.

.

.

.

.

.

.

.

.

.

.

.

.

.

.

.

.

.

.

.

.

.

.

.

.

.

.

.

.

.

.

.

.

.

.

.

.

.

.

.

.

.

.

.

.

.

.

.

.

.

.

.

.

.

.

.

.

.

.

.

.

.

.

.

.

.

.

.

.

.

.

.

.

.

.

.

.

.

.

.

.

.

.

.

.

.

.

.

.

.

.

.

.

.

.

.

.

.

.

.

.

.

.

.

.

.

.

.

.

.

.

.

.

.

.

.

.

.

.

.

.

.

.

.

.

.

.

.

.

.

.

.

.

.

.

.

.

.

.

.

.

.

.

.

.

.

.

.

.

.

.

.

.

.

.

.

.

.

.

.

.

.

.

.

.

.

.

.

.

.

.

.

.

.

.

.

.

.

.

.

.

.

.

.

.

.

.

.

.

.

.

.

.

.

.

.

.

.

.

.

.

.

.

.

.

.

.

.

.

.

.

.

.

.

.

.

.

.

.

.

.

.

.

.

.

.

.

.

.

.

.

.

.

.

.

.

.

.

.

.

.

.

.

.

.

.

.

.

.

................................................................................................................................................................................................................................

Question n 01 :

a)−→AB a pour coordonnées (3,−1). b)

−−→BC a pour coordonnées (1,−2).

Les droites D1 et D2 sontc) parallèles car les vecteurs

−→AB et

−−→BC

sont différents.d) perpendiculaires car le produit scalaire−→

AB · −−→BC n’est pas nul.

Question n 02 :

Le coefficient directeur de la droite D1 esta) positif car les points B et C ont desabscisses positives.

b) négatif car il existe des points de cettedroite ayant une ordonnée négative.

Enoncés 19

De plus

c) 3 (x−1)+y = 0 est une équation de ladroite D2 car cette équation est vérifiéepour x = 1 et y = 0.

d) 3 (y−1)−x = 0 est une équation de ladroite D2 car cette équation est vérifiépour x = 0 et y = 1, et pour x = 3 ety = 2.

La surface de la portion de plan hachurée (en unités d’aire) peut être calculée par laformule

Question n 03 :

a)Z 3

0

µ1

3x+ 1

¶dx−

Z 4

3

(2x− 8) dx. b)1

3

Z 3

0

(x+ 3) dx−Z 4

3

(−2x+ 8) dx.

c)Z 3

0

(−2x+ 8) dx+Z 3

0

(2x− 8) dx. d)1

3

Z 3

0

(x+ 3) dx+ 1.

06

Dans le plan affine euclidien, nous considérons les points distincts A et B. Nous posonsAB = l.

Soit r la rotation de centre A et d’angle θ de mesureπ

3et r0 la rotation de centre B et

de même angle θ.

Nous posons C = r (B) , D = r0 (A) , I l’orthocentre du triangleABC, J l’orthocentredu triangle ADB et K le symétrique de I par rapport à la droite (BC) .

Question 01 :

a) AI =l

2. b) AIBJ est un losange.

c) AJBI est un carré. d) IJ = l.

Question 02 :

a) Les points A, D, B et C sont cocy-cliques.

b) Les points A, J, B et I sont cocy-cliques.

c) AJKC est un carré. d) AJKC est un rectangle.

Question 03 :

a) AJ =l

2. b) BK = IJ.

c) IJB n’est pas un triangle équilatéral. d) Les droites (IB) et (JK) ne sont pasperpendiculaires.

Nous notons f = r r0 et g = r0 r.Question 04 :

a) f est l’identité. b) A est l’unique point invariant par f.

c) f(A) = B et f(B) = A. d) f(A) = B et f(B) = C.

20 Questionnements Automatisables

Question 05 :

a) g est l’identité. b) B est l’unique point invariant par g.

c) g(B) = C et g(C) = A. d) g(A) = B et g(B) = C.

Question 06 :

a) f est une rotation de centre I etd’angle

π

3.

b) f est une rotation de centre J et

d’angle −2π3.

c) f est une translation car la composéede deux rotations d’angles égaux estune translation.

d) f ne peut-être une translation car lasomme des angles des rotations r et r0

n’est pas un multiple de 2π.

Nous notons g−1 l’application réciproque de g.

Question 07 :

a) g−1 n’existe pas car g n’est pas bijec-tive.

b) g−1 = r−1 ( r0)−1 .

c) g−1 = g car le produit r r0 estcommutatif

d) g−1 = ( r0)−1 r−1.

Question 08 :

a) f g−1 est une rotation de centre O,où O est le milieu du segment [AB] .

b) f g−1 est une rotation de centre C

et d’angle −2π3.

f g−1 est une translation de vecteur

c)−−→BF =

−→JK. d)

−→AC.

Question 09 :

a) g f est l’identité. b) g f est une symétrie axiale.

c) g−1 f est l’identité. d) g−1 f est une symétrie axiale.

07

On donne un triangle quelconque ABC, trois points P, Q et R pris respectivement surBC, CA et AB, tels que les droites AP, BQ et CR soient concourantes en un point Odistinct de A, B et C.

Question n 01 :

On considère l’homothétie de centre P qui transforme A en O, C en C 0 et B en B0.

a) OB0 // AB b) RB0 // AC

c) QC 0 // AB d) RQ // BC

Enoncés 21

Question n 02 :

On posePO

PA=

X

BCoù le numérateur X est

a) BB0. b) B0C 0. c) BP. d) CC 0.

Question n 03 :

On poseRO

RC=

Y

BCoù le numérateur Y est

a) B0B. b) PC 0. c) C 0C. d) RC 0.

Question n 04 :

On poseQO

QB=

Z

BCoù le numérateur Z est

a) BC. b) B0C 0. c) C 0C. d) OB.

Question n 05 :

La valeur dePO

PA+

RO

RC+

QO

QBest

a)1

2. b) 0. c) 1. d) −1.

Question n 06 :

Si les trois rapports de la question précédente 5 sont égaux alors le point O est pour letriangle ABC

a) le centre de gravité. b) l’orthocentre.c) le centre du cercle circonscrit. d) le centre du cercle inscrit.

Nous savons qu’il existe trois réels α, β et γ tels que α + β + γ = 1 et tels que O soit lebarycentre des points A,B et C affectés respectivement des coefficients α, β et γ.

Question n 07 :

La valeur dePO

PAest

a) α. b) α− β. c) γ. d) α+ β.

Question n 08 :

La valeur deRO

RCest

a) 1− α. b) 1− α− β. c) β. d) α.

Question n 09 :

La valeur deQO

QBest

a) γ. b) α+ β. c) α. d) 1− α.

Question n 10 :

Cet exercice est basée sur une propriété connue sous le nom du théorème

a) Thalès. b) Euclide.c) Céva. d) Ménélaüs.

22 Questionnements Automatisables

08

Soit un quadrilatère convexe ABCD.

Sur chaque côté, vers l’extérieur voirla figure ci-contre, nous construisonsquatre triangles rectangles isocèlesAMB (rectangle en M, et donc telque AM = BM), BNC (rectangle en N etdonc tel que BN = CN), CPD (rectangleen P et donc tel que CP = DP ) et DQA(rectangle en Q et donc tel que DQ = AQ).

Nous notons O le point de concours desdiagonales MP et NQ du quadrilatèreMNPQ.

AB

C

D

M

N

P

QO

.............................................................................

......................................................................................................................................................................................................................................................................................................

........................................................

........................................................

........................................................

................................................................................................................................................................................................................................................................................................................................................................................................................................................................................................................................................................................................................................................................................................................................................................................................................................................................................................................................................................................................................................................................................

........

........

........

........

........

........

........

........

........

........

........

........

........

........

........

........

........

........

........

........

........

.

........ ........ ........ ........ ........ ........ ........ ........ ........ ........ ........ ........ ........ ........ ........ ........ ........ ........ ........ ........ ........ .

.........................................................................................................................................

................................

.........................................................................................................................................................................

.........................................................................................................................................................................

..................................................................................................................................................

.. ..

..... ....... .......

Nous supposons dans cette question que le quadrilatère ABCD est un carré.

Question 01 :

a) Le quadrilatère MNPQ est un carré. b) Les huit points A,B,C,D,M,N, P etQ sont cocycliques.[1]

c) Les droites AC, BD, MP et NQ sontconcourantes.

d) Les deux quadrilatères ABCD etMNPQ ont la même aire.

Nous supposons dans les deux questions suivantes que le quadrilatère ABCDest un losange tel que le triangle ABD soit équilatéral.

Question 02 :

a) Les quatre points A, B, C et D sontcocycliques.

b) Les quatre points A, M, B et O sontcocycliques.

c) L’angle \AMO a pour valeur 60. d) Le triangle AMO est équilatéral.

Question 03 :

a) Le quadrilatère MNPQ est un carré. b) Les droites AB et NQ sont parallèles.

c) Les trois pointsQ, A etM sont alignés. d) Le triangle OMQ est équilatéral.

Nous supposons dans les questions suivantes que le quadrilatère ABCD estdéfini dans un plan affine euclidien rapporté à un repère orthonormal Axy,par A = (0, 0), B = (5, 1) , C = (7, −5) et D = (0, −3) .Nous proposons une étude analytique du quadrilatèreMNPQ. Soit Ω un point déterminédu plan affine euclidien et ϕ la similitude produit de l’homothétie de centre Ω et de rapport

k =1√2et de la rotation de centre Ω et d’angle θ =

π

4.

[1] Cocycliques veut dire situés sur un même cercle.

Enoncés 23

Nous considérons le plan complexe usuel associé au plan affine euclidien.

Question 04 :

Au point m d’affixe z la transformation ϕ associe le point M d’affixe Z tel queZ = αz + β.

a) Si l’affixe de Ω est fixée alors α et βsont uniques.

b) L’image de M par ϕ est B et l’imagepar ϕ de C est M.

ω est l’affixe du point Ω.

c) Z =1√2e−i π/4z − ω. d) Z =

1 + i

2(z − ω) + ω.

Question 05 :

En donnant à ω différentes valeurs nous obtenons les coordonnées suivantes pour les pointsM, N, P et Q :

a) M = (2, −3) . b) N = (−9, 1) .

c) P =

µ5

2,15

2

¶. d) Q =

µ3

2, −32

¶.

Question 06 :

Une équation de la droite (MP ) est

a) y = −21x+ 45. b) x = 2.

c) L’assertion a) ne peut être vraie car ladroite (MP ) est parallèle à l’axe Ay.

d) L’assertion a) ne peut être vraie car ladroite (MP ) est identique à la droite(NQ) .

Question 07 :

Une équation de la droite (NQ) est

a) y =1

21x+

10

7. b) y = −1.

c) L’assertion a) peut être vraie car ladroite (NQ) a obligatoirement sur lafigure dessinée un coefficient directeurpositif.

d) L’assertion c) peut être vraie car ladroite (NQ) est parallèle à la droite(AB).

Question 08 :

a) Une équation de la droite (AC) est

y = −107x.

b) La droite (AC) passe par le point O.

c) Les droites (MP ) et (NQ) sont per-pendiculaires.

d) Les longueurs des segments [MP ] et[NQ] sont égales.

24 Questionnements Automatisables

09

Nous considérons un triangle quelconqueABC. Soit M le milieu du segment [BC] .

Les triangles BAB0 et C 0AC sont rec-tangles isocèles de sommet A, cf figure ci-contre.

Nous considérons l’homothétie h de centreB et de rapport 2 (Nous notons A0 = h (A)

et r la rotation de centre A et d’angle +π

2.

Nous avons C 0 = r(C) et B = r(B0).

A

B0

B

C

M

C 0

................................................................................................................................................................................................................................................................................................................................................................................................................................................

........

........

........

........

........

........

........

........

........

........

........

........

........

........

........

........

........

........

........

........

.........

........................................................................................................................................................................................................................................................................................................................................................................................................................................................................................................................................................................................................................................................................................................................................................................................................................................................

......................................................................................................................................................................................................................................................................

......................................................................................

.. ..

...... ........ ........ ....

........ ........ ................ ........ ................ ........ ................ ........ ................ ..

Question n 01 :

L’image du segment [AM ] par h est

a) le segment [A0C] tel que A soit le milieudu segment [A0B].

b) le segment [A0C] tel queA soit le milieudu segment [A0M ].

Nous avons aussic) Les droites (AM) et (A0C 0) sontparallèles.

d) Les droites (AM) et (A0C 0) sontperpendiculaires.

Question n 02 :

Pour la transformation s = r h, nous avonsa) C 0 = s(M) et A0 = s(B). b) C 0 = s(M) et B0 = s(A).

Nous en déduisons que les deux droites (AM) et (B0C 0) sont

c) parallèles et°°°−−→AM°°° = 2 °°°−−→B0C 0

°°°. d) perpendiculaires et°°°−−→AM°°°= 1

2

°°°−−→B0C 0°°°.

On considère le plan complexe tel que les affixes des pointsA, B etC soient respectivement0, 1 et r eiϕ(r > 0 et 0 < ϕ <

π

2).

Question n 03 :

Nous avons

a) B0 a pour affixe −2 i. b) C 0 a pour affixe r e−i ϕ.

c) M a pour affixe r ei ϕ / 2. d) A0 a pour affixe −2.On rappelle que la mesure complexe d’un vecteur

−−→XY , que l’on note

h−−→XY

i, est la

différence zY −zX entre les affixes des points Y et X. On note α =h−−→AM

iet β =

h−−→B0C 0

i.

Enoncés 25

Question n 04 :

Nous avons

a) α = ir ei ϕ + 1

2. b) α =

r ei ϕ − 12

.

c) β = r e−i ϕ + i. d) β = i (r ei ϕ + 1).

Question n 05 :

Nous avons

a) β =i α

2, ce qui prouve que les droites

(AM) et (B0C 0) sont parallèles.

b) β = −2 i α, ce qui prouve que lesdroites (AM) et (B0C 0) sont perpen-diculaires.

c) β = −i α2, ce qui prouve que les droites

(AM) et (B0C 0) sont parallèles.

d) β = 2 i α, ce qui prouve que les droites(AM) et (B0C 0) sont perpendiculaires.

10

On considère la fonction de C dans lui même définie par

f(z) = z4 −√2 z3 − 4

√2 z − 16.

Le problème propose d’étudier les racines de l’équation f(z) = 0.

Question n 01 :

Nous pouvons dire, sans calculer les racines de l’équation f(z) = 0, que toutes les racines

a) sont réelles car tous les coefficients decette équation sont réels.

b) ont une partie imaginaire non nullecar f(z) est un polynôme de degré 4,Il en est ainsi pour toute équation duquatrième degré.

Il existe deux constantes α et β telles que, ∀ z ∈ C, nous avons

f(z) = (z2 + 4) (z2 + α z + β).

alors

c) α = −√2 et β = 4. d) α =√2 et β = −4.

Question n 02 :

Nous déduisons de ce qui précède que les nombres complexes suivants sont des racinesde l’équation f(z) = 0.

a) −1 et 2√2. b) −√2 et −2 i. c) −i et 2√2. d) 2 i et 2√2.

26 Questionnements Automatisables

Question n 03 :L’équation f(z) = 0 admet quatre racines distinctes dans C, soient A, B, C et D lesimages dans le plan complexe de ces racines.

a) Les racines précédentes n’étant pastoutes différentes alors les pointsA, B, C et D ne sont pas tous dis-tincts.

b) Les points A, B, C et D sont cocy-cliques (ie: situés sur un même cercle).

c) Dans le cas où l’assertion b) est jugéevraie alors le rayon du cercle est

R =3√2.

d) Les points A, B, C et D sont alignéstrois à trois.

Question n 04 :

Nous associons à f la fonction g de la variable rélle x définie par

g(x) = x4 −√2x3 − 4

√2x− 16

de courbe représentative C dans un repère orthogonal Oxy.Nous avons pour la fonction numérique g

a) g est continue et au moins dérivabledeux fois sur R.

b) C admet trois points d’inflexions (dis-tincts).

c) L’équation g0(x) = 0 admet une et uneseule racine.

d) ∀ x ∈ R, g(x) 6 0.

11

Nous considérons le nombre complexe Z = 1 + sin α − i cos α, où α ∈]0, π[ et on sepropose de déterminer le module ρ et l’argument θ du complexe Z = ρ ei θ.

Question n 01 :

Nous avons les relations trigonométriques, pour tout x réel

a) 1− sin x = 2 sin2x

2. b) 1 + cos x = 2 cos2

x

2.

c) 1 + sin x = 2 cos2³x2− π

4

´. d) 1− cos x = 2 sin2

³π4− x

2

´.

Question n 02 :

Il est possible d’exprimer (1 + sin x)2 + cos2 x sous la forme

a) 2 (1 + cos x). b) 2 (1 + sin x).

c) 4 cos2³x2− π

4

´. d) 4 sin2

³π4− x

2

´.

Question n 03 :

Pour tout α ∈]0, π[, nous avonsa) −π

46 α

2− π

46 π

4. b)

π

46 π

4− α

26 −π

4.

Enoncés 27

Nous pouvons exprimer ρ sous la forme

c) ρ = 2 cos³α2− π

4

´. d) ρ = 2 sin

³π4− α

2

´.

Question n 04 :

Nous pouvons déduire de ce qui précède que

a) θ =α

2− π

4. b) θ =

π

4− α

2.

En conclusion, nous pouvons écrire Z sous la forme

c) Z = 2 cos³α2− π

4

´eiα

2− π

4 . d) Z = 2 sin³π4− α

2

´eiπ

4− α

2 .

12

Nous considérons, dans l’ensemble des nombres complexes C, l’équation

2 z3 − (1 + 2 i) z2 + (25 i− 1) z + 13 i = 0. (E)

Question n 01 :

a) Toutes les racines de (E) ont une partieimaginaire non nulle.

b) Une racine de (E) est α = −12.

(E) admet

c) au moins deux racines réelles car c’estune équation de degré 3.

d) une seule racine réelle α = −12.

L’équation (E) peut être écrite sous la forme

2 (z − α)¡z2 + a z + b

¢= 0.

Question n 02 :

La valeur de a est

a) 1. b)1

2. c) −1

2. d) 0.

Question n 03 :

La valeur de b est

a) 10 i. b) −4 i. c) 13. d) 13 i.

Question n 04 :

Le discriminant ∆ = a2 − 4 b de l’équation du second degréz2 + a z + b = 0 (E’)

est le carré du nombre complexe

a) 5 + 5 i. b) −5 + 5 i. c) 5− 5 i. d) 5 i.

28 Questionnements Automatisables

Nous notons β et γ les racines de (E0) (avec <e β > 0).

Question n 05 :

Nous avonsa) β = 3− 2 i et γ = 3α. b) αβ = 13 i et γ = −2 + 3 i.c) 3β + 2 γ est réel. d) <e γ < 0 et =mβ > 0.

Question n 06 :

a) L’équation (E) n’admet pas trois racines distinctes.

Dans le cas où les trois racines sont distinctes alors A, B, C étant les points d’affixesrespectives α, β, γ.

b) A, B, C sont alignés. c) A est le milieudu segment [BC].

d) Le triangle ABC estisocèle en A.

Si l’on accepte l’assertion 6 a) alors on choisira la case e) pour les trois questions suivantes.

Nous désignons par O le barycentre des points A, B et C affectés respectivement descoefficients 10, λ et µ.

Question n 07 :

La valeur de λ est

a) 10. b) 3. c) 2. d) −1.Question n 08 :

La valeur de µ est

a) 10. b) 3. c) 2. d) 1.Question n 09 :

Nous avonsa) O est centre de gravité du triangle

ABC.b) O est l’orthocentre du triangle ABC.

c) O est le centre du cercle circonscrit autriangle ABC.

d) O est le centre du cercle inscrit autriangle ABC.

13

Nous considérons les nombres complexes z et Z tels que

z = 2 cos2 θ + i sin 2θ, Z =1

z2, avec − π

2< θ <

π

2.

Question n 01 :

Nous avonsa) |z| = 2 sin θ. b) |z Z| = 1.

c) Arg z = −θ. d) Arg Z = −2 θ .

Enoncés 29

Question n 02 :

Nous avons

a) |Z| = 1

4 cos2 θ. b) |Z| = |z| pour |θ| = π

3.

c) |z|+ |Z| = 1 pour une valeur de θ. d) Z est nul pour θ = 0.

Soit l’équation en ζζ2 = z − 1 (E)

où z est défini ci-dessus.

Question n 03 :

a) (E) admet quatre solutions distinctes.

b) Il existe une et une seule valeur de θ telle que les solutions de (E) soient réelles.

c) Toutes les solutions de (E) ont le mêmemodule.

d) Toutes les solutions de (E) ont le mêmeargument.

Question n 04 :

Les nombres complexes z, Z et ζ vérifient pour tout θ

a) ζ2 − 1 = 1

Z. b) |ζ − 1|2 = |z|.

c) Z (z − 1) = |Z|. d)1

ζ2= z − Z − 1.

14

Dans le plan complexe, on donne les points O, A et B d’affixes respectives 0, 3 et 4 i.

Soit la fonction ϕ de la variable complexe z définie pour z 6= 4 iϕ(z) =

z − 3z − 4 i .

On note C l’ensemble des points d’affixe z tels que ϕ(z) soit un imaginaire pur non nul.

Soit M un point quelconque d’affixe z de l’ensemble C.Question n 01 :

Les droites (AM) et (BM) sont

a) parallèles. b) perpendiculaires.

Il existe un point M de C distinct de O tel qu’en ce pointc) les droites (OM) et (AB) soientperpendiculaires.

d) OAMB soit un carré.

Question n 02 :

C esta) un demi-cercle contenant O. b) le cercle de diamètre [AB].

c) contenu dans le cercle de diamètre[AB].

d) le segment [AB] privé du point B.

30 Questionnements Automatisables

Question n 03 :

L’équation ϕ(z) = i admet une et une seule solution notée α.

a) α a pour argument −π4. b) α a pour module

7√2.

Si β est une solution de l’équation ϕ(z) = −i alorsc) 4β4 + 1 = 0. d) β = α.

15

Nous considérons dans le repère orthonormé Oxy, les points Iµ−12, 0

¶, J (0, 1) et le

cercle C de diamètre [IJ ] .

Question n 01 :a) C est tangent à l’axe Oy et coupe l’axe

Ox en deux points distincts.b) C et l’axe Ox ont au moins un pointcommun.

c) La projectionΩ du centre de C sur l’axeOx admet pour abscisse

1

4.

d) Une équation de C estx2 − 1

2x+ y2 − y = 0 . (1)

Les tangentes à C parallèles à l’axe Oy coupent l’axe Ox en deux points, P d’abscissepositive et Q d’abscisse négative. Nous posons α = OP et β = OQ.

Question n 02 :

a) α et β sont les solutions de (1) pour y =1

2.

b) α > 0, α+ β =1

4et αβ = −1

4.

c) α =1

2IJ =

−1 +√54

. d) α = −β.

Question n 03 :

Pour une équation dans C, à coefficients réels, du quatrième degré

a) les racines sont toutes distinctes ousont toutes réelles.

b) les racines sont obligatoirementdistinctes.

c) les quatre racines sont conjuguées deuxà deux.

d) une racine au moins est réelle si leterme constant de l’équation est nul.

Question n 04 :

Dans C l’équationz4 + z3 + z2 + z + 1 = 0 (2)

admet quatre racines (admis).

Enoncés 31

a) Les quatre racines sont distinctes. b) Certaines racines sont réelles.

c) Le produit des racines est 1. d) La somme des racines est 1.

Nous notons zk = e2ikπ/5 avec k = 0..4 [1].

Question n 05 :a) zk vérifie l’équation z5 + 1 = 0 . b) z2 = α, z4 = β et z2 + z4 est réel.

Nous posons Z = z +1

zdans (2), et nous obtenons l’équation

c) Z2 + Z − 2 = 0. d) (Z − 1) (Z + 2) = 0.Question n 06 :

a) α = cos2π

5et β = cos

5. b) α = cos

5et β = cos

5.

Les points Ak d’affixe zk avec k = 0..4 sont les sommets d’un pentagone régulier.

c) A1 et A3 sont situés sur une tangenteà C parallèle à l’axe Oy.

d) Les points A2 et A4 sont symétriquespar rapport au point I.

Vous pouvez dorénavant construire un pentagone régulier avec une règle et un compas.

16

Nous considérons le polynôme dans C

f(z) = 4 z3 − 6 i√3 z2 − 3

³3 + i

√3´z − 4.

Question 01 :

f admet un zéro réel α0 qui vaut

a) −1. b)1

2. c) −1

2. d)

3

2.

f est divisible par le polynôme g(z) = z2 +Az +B tel que

Question 02 :

A vaut

a)1 + i

√3

2. b)

3√3− i

2 i. c) −1 + 3 i

√3

2. d)

3√3 + i

2 i.

Question 03 :

B vaut

a) 1− i. b) −2. c)1

2. d) 2.

[1] Nous notons, ce qui est courant en informatique, 0..4 l’ensemble des entiers compris entre 0 et 4, soit0, 1, 2, 3 et 4.

32 Questionnements Automatisables

Question 04 :

Une racine carrée du discriminant du trinôme du second degré g(z) est

a)−3 + i

√3

2. b)

√3− 3 i2

. c)3− i

√3

2. d)

√3 + 3 i

2.

g admet deux zéros complexes α1 et α2, avec <e α1 < 0, tels queQuestion 05 :

α1 vaut

a) −1− i√3. b) −1

2+ i√3. c)

−1 + i√3

2. d)

1− i√3

2.

Question 06 :

α2 vaut

a) −1− i√3. b)

1− i√3

2. c) 1− i

√3. d) 1 + i

√3.

Question 07

α0, α1 et α2 sont des éléments d’une suite géométrique dont la raison q vaut

a) −1 + i√3. b)

−1 + i√3

2. c) −1 + 3 i√3. d) 1− i

√3.

Question 08 :

Pour la suite géométrique αn = α0 qn, où n ∈ N, nous avons

a) =mα3 > 0. b) α3 = 4. c) <e α4 < 0. d) =mα4 < 0.

17

Nous désignons par j la racine cubique de l’unité dont la partie imaginaire eststrictement positive.

En d’autre termes, si nous notons j = ρ ei θ, où ρ est le module de j et θ son argument,alors ρ et θ vérifient les relations

ρ3 = 1 et θ =2 k π

3où k ∈ Z.

Question n 01 :

La valeur exacte du nombre complexe j est

a)−1 + i

√3

2. b)

1 + i√3

2. c)

−1− i√3

2. d)

1− i√3

2.

Dans le plan complexe, nous considérons les points A, B, C et D d’affixes respectives

−j2, 2j2, − 2

j2et4

j4.

Une équation cartésienne du cercle circonscrit (Γ) au triangle ABC est, dans le repèreorthonormal Oxy, de la forme

x2 + y2 + a x+ b y + c = 0.

Enoncés 33

Question n 02 :

La valeur de a est

a) −√3. b) −3. c)√3. d) 3.

Question n 03 :

La valeur de b est

a) −1. b)√3. c) 1. d) −3

2.

Question n 04 :

La valeur de c est

a) 0. b) −4. c)√3. d) 2

√3.

Question n 05 :a) Le point D est sur le cercle (Γ) . b) Le cercle (Γ) passe par l’origine O.

c) Le centre du cercle (Γ) est sur lesegment [BD] .

d) Le centre du cercle (Γ) est sur lesegment [AD] .

Question n 06 :

La valeur du nombre complexeh−−→CB

iest

a) −2− 2 i√3. b) 2− 2 i√3. c) 3− 2 i√3. d) −32+ i

√3

2.

Question n 07 :

La valeur du nombre complexeh−−→CD

iest

a) −2 i− 3√3. b) −3− i√3. c) −3 + i

√3. d) −2 + 2 i√3.

Question n 08 :

La valeur du nombre complexeh−−→AB

iest

a) −32+ i√3. b) −5

2− 5 i

√3

2. c) −

√3

2+ i

3

2. d) −3

2+ i

√3

2.

Question n 09 :

La valeur du nombre complexeh−−→AD

iest

a) −2 + 2 i√3. b) −52− i

5√3

2c) −

√3

1+ i

3

2. d) −3− i

√3.

Nous posons z0 =

h−−→CB

ih−−→CD

i et z00 =h−→AB

ih−−→AD

iQuestion n 10 :

Les quatre points A, B, C et D sont cocycliques car

a) |z0| = |z00|. b) |z0 z00| = 4. c) arg z0 = arg z00. d) arg z0=− arg z00.

34 Questionnements Automatisables

18

Le problème proposé ici est du même style que l’exercice proposé page 5 dans l’exposédes méthodes des Questionnements Automatisables.

Le texte original est le suivant :

1. Dans le plan affine P rapporté au repère orthonormal Axy, nous considérons les pointsB (a, a) et I (0, λ) , où a est un paramètre réel non nul et λ un paramètre réel telsque a 6= λ.

a) Calculer en fonction de λ les coordonnées du point d’intersection M de la droite(BI) et de la médiatrice ∆ du segment [AI] .

b) Montrer que ∆ est une bissectrice de l’angle (MA, MB) .

c) Déterminer et construire l’ensemble H des points M lorsque λ décrit R.

2. Dans le plan complexe d’origine O, associé à P, nous considérons la transformation fqui au point M d’affixe z associe le point M 0 d’affixe z0 tel que

z0 = f(z) = z2 − 2 (4 + i) z + 15.

a) Calculer les affixes α et β (avec =mα > 0) des points U et V dont l’image par fest le point O d’affixe nulle.

b) Exprimer en fonction du module |z0| et de l’argument arg z0 le produit des longueursdes segments UM et VM et l’angle polaire [1] d’une des bissectrices D de l’angle(MU, MB) .

c) Montrer qu’une condition nécessaire et suffisante pour que z0 soit réel est que D aitune direction fixe.

d) Déduire du 1, l’ensemble E des pointsM dont l’image est un réel strictement négatif.

3. Calculer pour z = x+ i y, les parties réelle x0 et imaginaire y0 de z0 = f(z) en fonctionde x et y.

En déduire l’ensemble E du 2.d).

Un approche en Questionnements Automatisables de ce problème peut-être lasuivante, à vous d’en imaginer d’autres :

Dans le plan P rapporté au repère orthonormal Axy,nous considérons les points B (a, a) et I (0, λ) où a est un paramètre réel non nul et λun paramètre réel tels que a 6= λ.

Soit le point d’intersectionM de la droite (BI) et de la médiatrice ∆ du segment [AI] .

[1] L’angle polaire d’une droite D est l’angle de cette droite avec l’axe de référence horizontal−→Ax, soit³−→

Ax, D´. Nous rappelons que les mesures de ces angles sont à k π prés, avec k entier relatif.

Enoncés 35

Question n 01 :

L’abscisse du point M est

a) − aλ

a− λ. b) − aλ

2 (a− λ). c)

aλ/2

λ− a. d)

aλ/2

a+ λ.

Question n 02 :

L’ordonnée du point M est

a)λ

2. b) −2

a. c) aλ. d) −λ

2.

Question n 03 :

∆ est une bissectrice de l’angle (MA, MB) car pour tout I

a) MA =MI =MB. b) MA =MB

2.

c) MA =MB 6=MI. d) MA =MI 6=MB.

Question n 04 :

L’ensemble H des points M lorsque λ décrit R est

a) une hyperbole dont une équation esty =

ax

2x− a.

b) seulement une branche d’une hyper-bole.

c) une hyperbole passant par les points Aet B.

d) une hyperbole admettant le point Bpour centre de symétrie.

Dans le plan complexe associé à P, nous considérons la transformation f qui au pointM d’affixe z associe le point M 0 d’affixe z0 tel que

z0 = f(z) = z2 − 2 (4 + i) z + 15.

Soient α et β (avec =mα > 0) les affixes des points U et V dont l’image par f est lepoint A d’affixe nulle.

Question n 05 :

L’affixe α (avec =mα > 0) du point U est

a) 6 + 3 i. b) −2 + 2 i. c) 2− i. d) −6 + 3 i.Question n 06 :

L’affixe β du point V est

a) α. b) −α. c) α2. d) −α2.Question n 07 :

La valeur du produit des longueurs de segments UM × VM est

a) |z0| . b)1

|z0| . c) |z0|2 . d) |z| .Question n 08 :

Si θ est l’angle polaire d’une des bissectrices D de l’angle (MU, MB) alors 2 θ vaut

a) arg z0 à un multiple de π près. b) arg z0 à un multiple de 2π près.

c) − arg z0 à un multiple de π près. d) − arg z0 à un multiple de 2π près.

36 Questionnements Automatisables

Question n 09 :

Nous avons

a) l’angle polaire de−−→UV est −π

4. b) le segment [UV ] coupe l’axe des

imaginaires.La droite D a une direction fixe si

c) z0 est réel. d) z0 est imaginaire pur.

Question n 10 :

L’ensemble E esta) est une hyperbole complète. b) contenu (strict) dans une hyperbole.

c) admet un centre de symétrie. d) contient les points U et V.

Nous posons z = x+ i y et z0 = x0 + i y0.

Question n 11 :

Nous avons

a) x0 = x2 + y2 − 8x+ 2 y + 15. b) x0 = y02.

c) x0 = (x− y − 3) (x+ y − 5). d) y0 = x y − x− 4 y.

Question n 12 :

Une équation de E est

a) y =x

x− 4 . b) x =4 y

y − 1 .c) E est dans la partie contenant O del’angle formé par les deux droites dontles équations sont x − y − 3 = 0 etx+ y − 5 = 0.

d) Une équation de E peut s’écrire sous laforme du système½

x y − x− 4 y = 0(y + 1) (y − 3) > 0 .

19

Nous considérons la fonction f de la variable complexe z définie par

f : z → Z = f(z) =i z − 1z − 2 i .

Nous notons D son ensemble de définition [1] et D0 l’ensemble[2] des images.

Question n 01 :

L’application f est

[1] D est l’ensemble des complexes z tels que le nombre complexe f(z) existe.[2] D0 est l’ensemble de valeurs de f(z) lorsque z décrit D.

Enoncés 37

a) involutive, ie : f2 = Id et D = D0 (iddésigne l’identité).

b) telle que sa restriction à R soit unebijection de R sur lui même.

c) telle que f(z) n’est réel que si z estimaginaire, ie : z ∈ D ∩ iR.

d) telle que f(z) est un imaginaire pur siz est un réel, ie : z ∈ D ∩ R.

Nous posons Z = X + i Y et z = x + i y, où X, Y, x et y sont réels et nous pouvonsécrire que

X + i Y =αx+ β y + γ + i (x2 + y2 + δ x+ ε y + ϕ)

x2 + (y − 2)2 .

Question n 02 :

Nous avons

a) α = −3 et ε = −1. b) α = 0 et β = 0.

c) δ = 0 et β = 1. d) α = −3 et β = 1.

Question n 03 :

Nous avons

a) α = −1 et β = 0. b) δ = 0 et γ = 1.

c) ε = −1 et ϕ = −2. d) ε = 0 et ϕ = −2.

Question n 04 :

Nous avons

a) ε = −1 et δ = 1. b) δ = 1.

c) ϕ = −2. d) β = 0 et γ = −1.

Nous appelons U (respectivement V ) l’ensemble des points m d’affixe z tels que Z soitréel (respectivement imaginaire pur).

Nous notons A et B les points d’affixes respectives −i et 2 i.

Question n 05 :

U est sura) un cercle passant par A et par B. b) une droite passant par A.

c) un cercle passant par B mais nepassant pas par A.

d) une droite passant par B.

Question n 06 :

V est sura) une droite passant par A. b) le cercle de diamètre [AB] .

c) confondu avec U. d) un cercle passant par B mais nepassant pas par A.

Soit r la rotation d’angle −π2définie par r (z) = z e−i π / 2.

38 Questionnements Automatisables

Question n 07 :

r (U) est

a) le cercle de diamètre [AB] . b) confondu avec U.

c) l’axe réel. d) l’axe imaginaire.

Question n 08 :

r (V ) est

a) le cercle de diamètre [AB] . b) confondu avec V.

c) sur l’axe réel. d) sur l’axe imaginaire.

Question n 09 :

L’équation cartésienne de la médiatrice ∆ du segment [AB] est

a) y = −12. b) y =

1

2. c) x =

3

2. d) x =

1

2.

Question n 10 :

Le point de ∆ dont l’image par f est le point A est

a) le point A lui même. b) le point B.

c) l’origine du repère. d) le milieu du segment [AB] .

20

La répartition des 260 candidats au baccalauréat de l’année 2041 [1] est en pourcentageselon le baccalauréat obtenu la même année :

Bac C : 25% Bac D : 35% Bac G : 25% Autres : 15%

L’année et les données sont évidemment fictives.

Question n 01 :

a) 156 candidats ont obtenu en 2041 lebaccalauréat C ou D.

b) 56 candidats ont obtenu en 2041 lebaccalauréat G.

Les candidats sont répartis en deux groupes égaux, celui formé de ceux ayant le baccalauréat

c) C ou D et celui formé de ceux ayantle baccalauréat G ou un baccalauréatautre que C, D ou G.

d) C ou G et celui formé de ceux ayantle baccalauréat D ou un baccalauréatautre que C, D ou G.

[1] L’auteur suppose que le baccalauréat sera ré-inventé, sous cette forme en 2041.

Enoncés 39

Une étude plus fine a montré que les certains candidats possèdent au plus deuxbaccalauréats différents

* 13 candidats ont eu le baccalauréat C en 2041 et le baccalauréat D avant 2041.

* 26 candidats ont eu le baccalauréat G en 2041 et le baccalauréat D avant 2041.

* les 221 autres candidats n’ont qu’un seul baccalauréat obtenu en 2041.

Question n 02 :

Il est possible de représenter l’ensemble des candidats, en fonction des baccalauréats obtenus,à l’aide du diagramme suivant

a)

. . . . . . . . . . . . . . . . .. . . . . . . . . . . . . . . . .. . . . . . . . . . . . . . . . .. . . . . . . . . . . . . . . . .. . . . . . . . . . . . . . . . .. . . . . . . . . . . . . . . . .. . . . . . . . . . . . . . . . .. . . . . . . . . . . . . . . . .. . . . . . . . . . . . . . . . .. . . . . . . . . . . . . . . . .. . . . . . . . . . . . . . . . .. . . . . . . . . . . . . . . . .. . . . . . . . . . . . . . . . . . . . . . . . . . . . . .. . . . . . . . . . . . . . . . . . . . . . . . . . . . .. . . . . . . . . . . . . . . . . . . . . . . . . . . . . .. . . . . . . . . . . . . . . . . . . . . . . . . . . . .. . . . . . . . . . . . . . . . . . . . . . . . . . . . . .. . . . . . . . . . . . . . . . . . . . . . . . . . . . .. . . . . . . . . . . . . . . . . . . . . . . . . . . . . .. . . . . . . . . . . . . . . . . . . . . . . . . . . . .. . . . . . . . . . . . . . . . . . . . . . . . . . . . . .. . . . . . . . . . . . . . . . . . . . . . . . . . . . .. . . . . . . . . . . . . . . . . . . . . . . . . . . . . .. . . . . . . . . . . . . . . . . . . . . . . . . . . . .. . . . . . . . . . . . . . . . . . . . . . . . . . . . . .. . . . . . . . . . . . . . . . . . . . . . . . . . . . .. . . . . . . . . . . . . . . . . . . . . . . . . . . . . .. . . . . . . . . . . . . . . . . . . . . . . . . . . . .. . . . . . . . . . . . . . . . . . . . . . . . . . . . . .. . . . . . . . . . . . . . . . . . . . . . . . . . . . .. . . . . . . . . . . . . . . . . . . . . . . . . . . . . .. . . . . . . . . . . . . . . . . . . . . . . . . . . . .. . . . . . . . . . . . . . . . . . . . . . . . . . . . . .. . . . . . . . . . . . . . . . . . . . . . . . . . . . .. . . . . . . . . . . . . . . . . . . . . . . . . . . . . .

. . . . . . . . . . . . . .. . . . . . . . . . . . .. . . . . . . . . . . . . .. . . . . . . . . . . . .. . . . . . . . . . . . . .. . . . . . . . . . . . .. . . . . . . . . . . . . .. . . . . . . . . . . . .. . . . . . . . . . . . . .

BacCBacD

BacG

Autresb)

. . . . . . . . . . . . . . .. . . . . . . . . . . . . .. . . . . . . . . . . . . . .. . . . . . . . . . . . . .. . . . . . . . . . . . . . .. . . . . . . . . . . . . .. . . . . . . . . . . . . . .. . . . . . . . . . . . . .. . . . . . . . . . . . . . .. . . . . . . . . . . . . .. . . . . . . . . . . . . . .. . . . . . . . . . . . . .. . . . . . . . . . . . . . .. . . . . . . . . . . . . .. . . . . . . . . . . . . . .. . . . . . . . . . . . . .. . . . . . . . . . . . . . .. . . . . . . . . . . . . .. . . . . . . . . . . . . . .. . . . . . . . . . . . . .

. . . . . . . . . . . . . . . . . . . . . . . . . . . . . .. . . . . . . . . . . . . . . . . . . . . . . . . . . . .. . . . . . . . . . . . . . . . . . . . . . . . . . . . . .. . . . . . . . . . . . . . . . . . . . . . . . . . . . .. . . . . . . . . . . . . . . . . . . . . . . . . . . . . .. . . . . . . . . . . . . . . . . . . . . . . . . . . . .. . . . . . . . . . . . . . . . . . . . . . . . . . . . . .. . . . . . . . . . . . . . . . . . . . . . . . . . . . .. . . . . . . . . . . . . . . . . . . . . . . . . . . . . .. . . . . . . . . . . . . . . . . . . . . . . . . . . . .. . . . . . . . . . . . . . . . . . . . . . . . . . . . . .. . . . . . . . . . . . . . . . . . . . . . . . . . . . .. . . . . . . . . . . . . . . . . . . . . . . . . . . . . .. . . . . . . . . . . . . . . . . . . . . . . . . . . . .. . . . . . . . . . . . . . . . . . . . . . . . . . . . . .. . . . . . . . . . . . . . . . . . . . . . . . . . . . .. . . . . . . . . . . . . . . . . . . . . . . . . . . . . .. . . . . . . . . . . . . . . . . . . . . . . . . . . . .. . . . . . . . . . . . . . . . . . . . . . . . . . . . . .. . . . . . . . . . . . . . . . . . . . . . . . . . . . .. . . . . . . . . . . . . . . . . . . . . . . . . . . . . .. . . . . . . . . . . . . . . . . . . . . . . . . . . . .. . . . . . . . . . . . . . . . . . . . . . . . . . . . . .

. . . . . . . . . . . .. . . . . . . . . . .. . . . . . . . . . . .. . . . . . . . . . .. . . . . . . . . . . .. . . . . . . . . . .. . . . . . . . . . . .. . . . . . . . . . .. . . . . . . . . . . .. . . . . . . . . . .. . . . . . . . . . . .

BacCBacD

BacG

Autres

c) Le nombre des candidats ayant seule-ment le baccalauréat C est égal aunombre des candidats reçus au bac-calauréat C en 2041.

d) Le nombre des candidats ayant seule-ment le baccalauréat D est égal aunombre des candidats reçus au bac-calauréat D en 2041.

Le jury a admis 100 candidats à la session 2041.

Question n 03 :

Si l’on suppose que tous les candidats ont la même probabilité d’être admis alors

a) 25 candidats admis ont seulement lebaccalauréat C.

b) 25 candidats admis ont au plus lebaccalauréat G.

c) 50 candidats admis ont au plus lebaccalauréat D.

d) 5 candidats admis ont les baccalauréatsD et G.

Question n 04 :

On suppose que certains candidats admis démissionnent :

* tous ceux qui ont les baccalauréats C et D.

* la moitié de ceux qui ont les baccalauréat G et D.

* les deux tiers de ceux qui ont un baccalauréat autre que C, D ou G.

Finalement la promotion de 2041 est

a) égale à 85 personnes. b) inférieur à 85 personnes.

Dans cette promotion

c) il y a au plus 5 personnes ayant deuxbaccalauréats.

d) il y a au plus 70 personnes ayant auplus un baccalauréat.

40 Questionnements Automatisables

21

Nous considérons une classe de 40 élèves qui préparent un concours. Dans cette classeil y a 8 filles et 32 garçons. Les résultats (des années antérieures) montrent que 75%des filles sont reçues et 50% des garçons.

Nous notons F l’événement “ être une fille ”, F est l’événement contraire, c’est-à-dire“ être un garçon ”, R l’événement “ être reçu ou reçue ” et R l’événement contraire“ être collé ou collée ”.

Nous supposons l’équiprobabilité pour tout élève, fille ou garçon, d’être reçu ou collé àce concours.

Nous définissons la probabilité de l’événement A |B par la relation

Prob (A|B) = Prob (A ∩B)Prob (B)

où A et B sont deux événements tels que Prob (B) 6= 0. (Nous dirons que Prob (A |B) estla probabilité de l’événement A sachant que l’événement B est réalisé).

Ainsi l’énoncé ci-dessus donne les valeurs

Prob (R |F ) = 0, 75 et Prob (R | F ) = 0, 50oùR |F est l’événement “ être reçue sachant que l’on est une fille ” etR | F est l’événement“ être reçu sachant que l’on est un garçon ”.

Nous rappelons que le cardinal d’un ensemble E, noté Card (E), est le nombre d’élémentsde cet ensemble. Ainsi l’énoncé ci-dessus donne les valeurs

Card (F ) = 8 et Card¡F¢= 32.

Question n 01 :

La valeur de Prob (F ) est

a) 0,2. b) 0,75.

La valeur de Prob¡F¢est

c) telle que Prob (F )+Prob¡F¢= 1. d) nulle.

Question n 02 :

La valeur de Prob (R ∩ F ) est

a) 0,15. b)0, 75

0, 2= 0, 375.

c) Les événements R ∩ F et R ∩ F sontincompatibles.

d) L’événement R ∩ F est l’événementcontraire de R ∩ F .

Question n 03 :a) Card (R ∩ F ) = 8et Card (R ∩ F ) = 16.

b) Card (R ∩ F ) = 6et Card (R ∩ F ) = 32.

Enoncés 41

c) Card (R ∩ F ) = 2et Card (R ∩ F ) = 16.

d) Card (R ∩ F ) = 32et Card (R ∩ F ) = 8.

Question n 04 :

La valeur de Card (R) est

a) 8× 0, 75 + 32× 0, 5 = 22. b) 40× 0, 75× 0, 5 = 16.

De plus

c) Card (R)+Card¡R¢= 1. d) Prob (R) =

22

40.

Question n 05 :

a) Prob (F |R) = Prob (F ∩R)Prob (R)

. b) Prob (F |R) = 8

22.

c) Prob (F |R) = Prob (F ∩ R)Prob

¡R¢ . d) Prob (F |R) = 16

22.

Question n 06 :

a) Prob (F |R)+Prob (F |R) = 1. b) Prob (F |R)+Prob (F |R) = 3

4+1

2.

c) Prob (F |R)+Prob (F | R) = 8

40. d) Prob (F |R)+Prob (F | R) = 83

99.

22

Au cours d’une journée une PME fabrique 10 tables et 40 chaises parmi lesquelles ondénombre 2 tables et 10 chaises défectueuses.

Un contrôleur vérifie la fabrication journalière en prenant, au hasard une sélection dedeux meubles, formée d’une table parmi les 10 et d’une chaise parmi les 40.

Nous notons C l’événement : la chaise choisie est non défectueuse et C l’événement :la chaise choisie est défectueuse.

Nous notons T l’événement : la table choisie est non défectueuse et T l’événement : latable choisie est défectueuse.

Pour une sélection donnée, nous notons

p1 la probabilité pour le contrôleur d’avoir un et un seul meuble défectueux.

p2 la probabilité pour le contrôleur d’avoir une table défectueuse et une chaise nondéfectueuse.

p3 la probabilité pour le contrôleur d’avoir une table non défectueuse et une chaisedéfectueuse.

Question n 01 :

a) Prob (T ) =2

40. b) Prob

¡T¢=8

10.

42 Questionnements Automatisables

c) Prob¡C¢=10 + 2

40. d) Prob (C) =

10

40.

Question n 02 :

a) Prob (C)+Prob¡C¢= 1. b) Prob (C) = 1−Prob ¡C¢ .

c) Prob (T )+Prob¡T¢= 0. d) Prob (T ) = 1+Prob

¡T¢.

Question n 03 :

Nous avons Prob (C et T )+Prob¡C et T

¢égal à

a) Prob¡C¢=3

4. b) Prob (C) =

1

4.

Nous avons Prob (C et T )+Prob¡C et T

¢égal à

c) Prob (T ) =1

4. d) Prob

¡T¢=2

10.

Question n 04 :

a) p1 =Prob¡C et T

¢+Prob

¡C et T

¢. b) p1 =Prob (C et T )+Prob

¡C et T

¢.

c) p2 =Prob¡C et T

¢. d) p3 =Prob

¡C et T

¢.

Question n 05 :a) p1 + p2 + p3 = 1. b) p1 = p2 = p3.

c) p1 = p2 × p3. d) p1 = p2 + p3.

Question n 06 :

a) p2 =3

20. b) p1 =

7

20. c) p1 =

4

20. d) p3 =

3

20.

23

Une boîte contient 3 boules blanches (B) et k (on suppose k > 2) boules rouges (R).Nous effectuons deux tirages au hasard, ce qui veut dire que tous les tirages sontéquiprobables.

* Dans les questions 1 et 2 les tirages sont effectués avec remise, c’est-à-dire que noustirons une première boule qui est ensuite remise dans la boîte et ensuite nous tirons ànouveau une boule.

* Dans les questions 3 et 4 les tirages sont effectués sans remise, c’est-à-dire que noustirons les deux boules simultanément.

Nous notons pk la probabilité d’obtenir un tirage bicolore.

Question n 01 :a) Il y a 2 (k + 3) tirages car à chaquetirage on a k + 3 possibilités.

b) Il y a (k+3)! tirages car chaque tirageest un arrangement.

Enoncés 43

c) Il y a seulement 4 tirages car les seulespossibilités sont BB, BR, RB et RR.

d) Il y a (k+3)2 tirages car chaque tirageest une 2-liste de k + 3 éléments.

Question n 02 :

a) limk→+∞

pk =1

2. b) pk >

1

2pour tout k > 2.

Il existe une seule valeur de k pour laquelle

c) pk =1

2. d) pk =

3

4.

Question n 03 :

a) Le nombre de tirages est(k + 3) (k + 2)

2.

b) L’assertion a) est obligatoirement fausse car(k + 3) (k + 2)

2ne peut être un entier,

sauf pour certaines valeurs particulières de k, en nombre fini.

c) Il y a 6 tirages formés de deux boulesblanches car il y a 3 choix possiblespour la première et 2 pour la seconde.

d) Il y a 3k tirages formés d’une bouleblanche et d’une boule rouge.

Dans cette question k = 5. On note p la probabilité d’un tirage unicolore.

Question n 04 :

a) p = 0 car le nombre de boules blanches est inférieur au nombre de boules rouges.

b) p =2

5, car nous avons 2 boules blanches et 5 boules rouges.

c) p =10

25=2

5, car il y a 2 ∗ 5 = 10 tirages d’une boule blanche et d’une boule rouge

et 52 = 25 tirages de deux boules dans cette boîte.

d) p =15

28, car il y 3 ∗ 5 = 15 possibilités de tirer une blanche et une rouge et

8 ∗ 7 / 2 = 28 tirages de deux boules.

24

Nous désignons par Prob (X) la probabilité d’un événement X, par X l’événementcontraire de l’événement X.

Les étudiants peuvent s’il le souhaitent pratiquer un ou deux sports.

40% des étudiants pratiquent le sport A, 25% le sport B et 10% à la fois A et B.

Nous notons A l’événement : “ L’étudiant lambda pratique le sport A ” et on note Bl’événement : “ L’étudiant lambda pratique le sport B ”.

44 Questionnements Automatisables

Question n 01 :

La probabilité p1 pour que lambda pratique au moins un des deux sports est

a) Prob (A ∩B). b) Prob¡A ∪ B¢ .

c) 1−Prob (A ∩B)· d) Prob (A ∪B).

Question n 02 :

La valeur exacte de p1 est

a) 0,65. b) 0,55. c)5

9. d)

5

11.

Question n 03 :

La probabilité p2 pour que lambda pratique un et un seul des deux sports est

a) Prob¡A ∪B¢ . b) Prob

¡A ∩B¢+Prob ¡A ∩ B¢ .

c) Prob (A ∪B)+Prob (A ∩B)· d) Prob (A)+Prob (B)−Prob (A ∩B).Question n 04 :

La valeur exacte de p2 est

a) 0,35. b) 0,55. c)5

9. d)

5

11.

Question n 05 :

La probabilité p3 pour que lambda pratique uniquement le sport A est

a) Prob (A). b) Prob (A)−Prob (A ∩B).

La valeur exacte de p3 est

c)3

10. d)

3

7.

Question n 06 :

La probabilité p4 pour que lambda pratique uniquement le sport B est

a) Prob (B). b) Prob (B)+Prob (A ∩B).

La valeur exacte de p4 est

c)3

10. d)

3

20.

Question n 07 :

Nous supposons que le nombre des étudiants est de 360.

a) 90 étudiants pratiquent uniquement lesport A.

b) 162 étudiants ne pratiquent aucunsport.

c) 36 étudiants pratiquent les deux sports. d) 234 étudiants pratiquent au moins unsport.

CORRIGÉS

Corrigés 47

01

Les problèmes de logique sont souvent des problèmes ensemblistes comme ceux que nousrencontrons dans les exercices de probabilités.

Nous proposons un exemple illustrant ce concept.

Toutefois, il est nécessaire d’être très prudent dans l’interprétation des hypothèses enétant précis et rigoureux. Notamment chaque mot doit être pris dans son sens le plusstrict.

La nature de ce type de QCM exige beaucoup d’attention et surtout une lecture critiquedes hypothèses et des assertions.

Une première question illustrant ce point de vue.Nous représentons graphiquement les hypothèses, à partir de l’équivalence ensembliste

(a) =⇒ (b) ⇐⇒ A ⊂ B ,

qui exprime que si l’assertion (a) implique l’assertion (b) alors l’ensemble A des élémentsvérifiant (a) est inclus dans l’ensemble B des éléments vérifiant (b).

Il est clair que nous pouvons faire une partition des lycéens amis des trois jeunes filles enPythagorofiles Π (ie : ceux connaissant le théorème de Pythagore) et en PythagorophobesΠ (ie : ceux ignorant le théorème de Pythagore, les malheureux...).

Si nous notons respectivement, S les lycéens amis de Sophie, A les lycéens amis d’Anne-Marie et C les lycéens amis de Christelle alors les hypothèses de cette question setraduisent en langage ensembliste de la manière suivante :

Phrase Logique Langage ensemblisteAucun ami de Sophie n’ignore PythagoreAucun ami d’Anne-Marie ne connaît PythagoreTout ami de Christelle connaît Pythagore

S ⊂ ΠA ⊂ ΠC ⊂ Π

Les Questionnements Automatisables analysent en particulier la compréhension dutexte, ainsi il faut bien traduire que “Aucun n’ignore” n’est autre que “Tout ami connaît”.

S et C étant inclus dans Π peuvent avoir une intersection vide ou non vide, ce qui nousconduit à deux diagrammes de Venn (En fait, il faudrait aussi distinguer les cas S ⊂ Cet C ⊂ S, mais cela n’est pas nécessaire en raison des assertions proposées).

. . . . . . . . . . . . . . . . . . . . . . . .. . . . . . . . . . . . . . . . . . . . . . . .. . . . . . . . . . . . . . . . . . . . . . . .. . . . . . . . . . . . . . . . . . . . . . . .. . . . . . . . . . . . . . . . . . . . . . . .. . . . . . . . . . . . . . . . . . . . . . . .. . . . . . . . . . . . . . . . . . . . . . . .. . . . . . . . . . . . . . . . . . . . . . . .. . . . . . . . . . . . . . . . . . . . . . . .. . . . . . . . . . . . . . . . . . . . . . . .. . . . . . . . . . . . . . . . . . . . . . . .. . . . . . . . . . . . . . . . . . . . . . .. . . . . . . . . . . . . . . . . . . . . . .. . . . . . . . . . . . . . . . . . . . . . .. . . . . . . . . . . . . . . . . . . . . . .. . . . . . . . . . . . . . . . . . . . . . .. . . . . . . . . . . . . . . . . . . . . . .. . . . . . . . . . . . . . . . . . . . . . .. . . . . . . . . . . . . . . . . . . . . . .. . . . . . . . . . . . . . . . . . . . . . .. . . . . . . . . . . . . . . . . . . . . . .. . . . . . . . . . . . . . . . . . . . . . .. . . . . . . . . . . . . . . . . . . . . . .. . . . . . . . . . . . . . . . . . . . . . .

. . . . . . . . . . . . . . . . . . . . . . . . . . . .. . . . . . . . . . . . . . . . . . . . . . . . . . . .. . . . . . . . . . . . . . . . . . . . . . . . . . . .. . . . . . . . . . . . . . . . . . . . . . . . . . . .. . . . . . . . . . . . . . . . . . . . . . . . . . . .. . . . . . . . . . . . . . . . . . . . . . . . . . . .. . . . . . . . . . . . . . . . . . . . . . . . . . . .. . . . . . . . . . . . . . . . . . . . . . . . . . . .

S

A

Π

. . . . . . . . . . . . . . . .. . . . . . . . . . . . . . .. . . . . . . . . . . . . . . .. . . . . . . . . . . . . . .. . . . . . . . . . . . . . . .. . . . . . . . . . . . . . .. . . . . . . . . . . . . . . .. . . . . . . . . . . . . . .. . . . . . . . . . . . . . . .. . . . . . . . . . . . . . .. . . . . . . . . . . . . . . .. . . . . . . . . . . . . . .. . . . . . . . . . . . . . . .

. . . . . . . . . . . . . . . . . . . . .. . . . . . . . . . . . . . . . . . . .. . . . . . . . . . . . . . . . . . . . .. . . . . . . . . . . . . . . . . . . .. . . . . . . . . . . . . . . . . . . . .. . . . . . . . . . . . . . . . . . . .. . . . . . . . . . . . . . . . . . . . .. . . . . . . . . . . . . . . . . . . .. . . . . . . . . . . . . . . . . . . . .. . . . . . . . . . . . . . . . . . . .. . . . . . . . . . . . . . . . . . . . .. . . . . . . . . . . . . . . . . . . .. . . . . . . . . . . . . . . . . . . . .. . . . . . . . . . . . . . . . . . . .. . . . . . . . . . . . . . . . . . . . .. . . . . . . . . . . . . . . . . . . .. . . . . . . . . . . . . . . . . . . . .

. . . . . . . . . . . . . . . . . . . . . . . . . . . .. . . . . . . . . . . . . . . . . . . . . . . . . . . .. . . . . . . . . . . . . . . . . . . . . . . . . . . .. . . . . . . . . . . . . . . . . . . . . . . . . . . .. . . . . . . . . . . . . . . . . . . . . . . . . . . .. . . . . . . . . . . . . . . . . . . . . . . . . . . .. . . . . . . . . . . . . . . . . . . . . . . . . . . .. . . . . . . . . . . . . . . . . . . . . . . . . . . .

S

A

C Π

Π

Diagramme 1 Diagramme 2

L’assertion 1 a) exprime que C ∩A est non vide, ce qui d’après les deux diagrammes estvrai, donc cette assertion est vraie.

48 Questionnements Automatisables

L’assertion 1 b) exprime que (S ∩C) ⊂ Π, ce qui est faux d’après le premier diagramme,donc cette assertion est fausse (Ne pas oublier que les hypothèses exigent vrai pour lesdeux diagrammes possibles).

L’assertion 1 c) exprime que (S ∩ C) ⊂ ¡A ∩ Π¢ = A, ce qui est faux d’après le premierdiagramme, donc cette assertion est encore fausse.

Enfin l’assertion 1 d) exprime A ⊂ Π, ce qui n’est autre que la deuxième hypothèseexprimée différemment, donc cette assertion est vraie.

Nous avons les réponses :

Question 01 : a) Vraie b) Fausse c) Fausse d) Vraie e) Fausse

Un autre type de problème de logique consiste en l’écriture d’une table de vérité.Malheureusement les tables de vérités sont peu utilisées alors qu’elles sont le fondement del’informatique et permettent de mieux comprendre les algorithmes ainsi que les relationsensemblistes en probabilité.

L’élément de base d’une table de vérité est qu’une assertion est vraie ou fausse, ce quin’est qu’une évidence.

Une deuxième question illustrant ce point de vue.

L’énoncé précise qu’une et une seule des quatre tables de vérités (α), (β), (γ) ou (δ),suivantes est vraie.

(a0) (b0) (c0) (d0)Vraie Fausse Fausse Fausse

(α)

(a0) (b0) (c0) (d0)Fausse Vraie Fausse Fausse

(β)

(a0) (b0) (c0) (d0)Fausse Fausse Vraie Fausse

(γ)

(a0) (b0) (c0) (d0)Fausse Fausse Fausse Vraie

(δ)

Nous examinons chaque tableau avec les trois hypothèses qui nous sont proposées.

L’hypothèse 1(b0) Vraie =⇒ (c0) Vraie

élimine le tableau (β) mais ATTENTION pas le tableau (γ) . En effet en notant B et Cles ensembles correspondants, cette hypothèse traduit B ⊂ C , ce qui permet d’envisagerdes éléments de C qui ne sont pas dans B .

Ainsi la proposition(b0) Fausse =⇒ (c0) Vraie

n’est pas à priori incorrecte. Nous n’avons aucune indication ...

L’hypothèse 2(a0) Vraie =⇒ (b0) Vraie ou (d0) Vraie

Corrigés 49

élimine le tableau (α) .

Enfin l’hypothèse 3(d0) Fausse ⇐⇒ (c0) Vraie

est équivalente à(d0) Vraie ⇐⇒ (c0) Fausse

et permet l’existence des tableaux (γ) et (δ) et de conclure pour les quatre assertionsproposées à la question 2.

D’après (δ) l’affirmation (d0) peut être vraie, donc l’assertion 1 a) est Vraie.

D’après (γ) et (δ) les assertions 1 b), 1 c) et 1 d) sont fausses.

ATTENTION le mot nécessairement n’est pas obligatoire à l’assertion 1 b), en effet direqu’une affirmation est vraie, c’est sous entendre nécessairement ou obligatoirement.

Certains mots invitent à la prudence ...

Nous avons les réponses :

Question 02 : a) Vraie b) Fausse c) Fausse d) Fausse e) Fausse

NB : Avez vous remarqué que dans ces deux questions de logique, nous n’avons examinéles assertions proposées qu’après avoir traduit toutes les hypothèses. Une erreur fréquentedans ce type de QCM est de lire en diagonale l’énoncé et de passer trop rapidement auxassertions proposées. Bonjour les dégâts ...

C’est cela les Questionnements Automatisables. C’est très simple pour ceux quisavent lire et qui réfléchissent et ... très néfaste pour les autres.

02

Pour toute valeur du paramètre réel strictement positif m, le triangle BCD est rectangleen B et la droite BA est la hauteur de ce triangle relative à l’hypoténuse CD.

La deuxième question, nous propose des relations métriques relatives aux trianglesrectangles.

Il existe deux méthodes d’exploration pour les exercices de ce type :

* Utilisation de la géométrie analytique ou des nombres complexes.

* Transformations affines dans C.Nous développerons dans ce qui suit simultanément ces deux méthodes. En laissant lapossibilité au lecteur de choisir la méthode de sa convenance.

Nous déterminons en premier les équations cartésiennes des droites AB et ∆ (Droitepassant par A et perpendiculaire à AB).

La forme générale de l’équation de AB est : y = a x+ b, où a et b sont déterminés par lesystème ½

m = a ∗ 0 + b0 = a ∗ 2 + b .

qui traduit le fait que les points B(0, m) et A(2, 0) sont situés sur cette droite.

50 Questionnements Automatisables

D’où l’équation de la droite AB

y = −m2x+m. (1)

Nous avons la représentation graphique suivante faite pour m > 0 [1]

........

........

........

........

........

........

........

........

........

........

........

........

........

........

........

........

........

........

........

........

........

........

........

........

........

........

........

........

........

........

........

........

........

........

........

........

........

........

........

........

........

........

........

........

........

........

........

........

........

........

...........................

..........................

y

................................................................................................................................................................................................................................................................................................................................................................................................................................................................................................................................................................................................................................................... ..........................xO A

B

C

D

m

2

.........................................................................................................................................................................................................................................................................................................................................................................................................................................................................................................................................................................................................................

...............................................................................................................................................................................................................................................................................................................................................................................................................................................................................................................................................................................................................................................................................................

...................................................................

.......................................

...............

..........

............................................................................................................

................................................................................................................................................................................................................................................

Figure 2.1

La forme générale de l’équation de ∆ est : y = a0x+ b0, où a0 est tel que

a a0 = −1qui caractérise le fait que les droites AB et ∆ sont perpendiculaires et b0 tel que le pointA(2, 0) soit situé sur ∆.

Ainsi a0 =2

met 0 =

2

m2 + b0, d’où l’équation de ∆ est

y =2

mx− 4

m. (2)

En particulier, nous pouvons dans le cas m = 4 dire que l’équation de la droite CD quin’est autre que la droite ∆ est d’après (2)

y =x

2− 1 .

Ce qui permet de dire que l’assertion 3 a) est fausse et que l’assertion 3 b) est vraie.

Il est fréquent dans les Questionnements Automatisables que l’étude classique duproblème posé permette de répondre à certaines questions. Ceci est normal et logique carce type de QCM est construit de cette façon. Les réponses au hasard sont écartées auprofit des réponses raisonnées.

En conclusion, ayez de la mémoire et raisonnez de manière très classique, sinon vousn’obtiendrez aucun résultat.

Le point C étant le point d’intersection de ∆ et de l’axe Oy, admet donc pour ordonnée

− 4met ainsi a pour affixe

zC = − 4m

i.

En particulier pour m = 4, le point C admet pour coordonnées (0, −1) ce qui prouve quel’assertion 3 d) est fausse et que la première partie de l’assertion 3 c) est vraie, il nousfaut contrôler les coordonnées de D.

[1] Il est conseillé de faire une figure pour m < 0 afin de contrôler les résultats dans tous les cas.

Corrigés 51

Les coordonnées de D vérifient le système(y = m

y =2

mx − 4

m

qui établit que l’affixe de D est

zD =m2 + 4

2+mi ,

donc que les assertions 1 c) et 1 d) sont fausses. En particulier pour m = 4, le point Dadmet pour coordonnées (10, 4) ce qui prouve que l’assertion 3 c) est fausse.

NB : Vous avez remarqué que les assertions 3 c) et 3 d) sont à moitié vraies et à moitiéfausses, donc ... fausses toutes les deux. En mathématiques une bouteille à moitié videou à moitié pleine est non entièrement remplie, donc vide.

Nous avons les réponses :

Question 03 : a) Fausse b) Vraie c) Fausse d) Fausse e) Fausse

NB : Nous avons répondu à la question 3 et à certaines assertions de la question 1. C’estencore une différence entre QCM et texte classique, cela peut s’appeler la souplesse et lafaculté d’adaptation.

Examinons maintenant les assertions a) et b) de la question 1.

zB et zD ont même partie imaginaire mais des parties réelles distinctes, l’assertion 1 a)est donc fausse.

L’angle³−→AC,

−→AB

´est bien droit, mais sa mesure est −π

2si m > 0 et

π

2si m < 0, ce qui

implique que l’assertion 1 b) est fausse (elle est vraie pour m < 0).

ATTENTION : Ne pas lire trop vite les assertions car entre i et -i ... c’est pile ou face ...Un petit coup d’oeil sur la figure c’est mieux.

Dur de commencer un exercice ou toutes les assertions proposées sont fausses.

Il faut donc répondre Vraie à l’assertion 1 e).

Nous avons les réponses :

Question 01 : a) Fausse b) Fausse c) Fausse d) Fausse e) Vraie

Nous rappelons que les affixes des points A, B, C et D sont respectivement

zA = 2 , zB = mi , zC = − 4m

et zD =m2 + 4

2+mi .

La connaissance des affixes des points A, B, C et D donne aussi la possibilité de répondreaux assertions de la question 2 mais aussi de la question 4, car nous pouvons calculer leslongueurs (ou plus simplement les carrés des longueurs) des segments proposés.

Nous avons successivement pour AB2

AB2 = |zB − zA|2= (0− 2)2 + (m− 0)2= 4 +m2 . (3)

52 Questionnements Automatisables

Pour AC2

AC2 = |zC − zA|2

= (0− 2)2 +µ− 4m− 0¶2

= 44 +m2

m2. (4)

Pour AD2

AD2 = |zD − zA|2

=

µm2 + 4

2− 2¶2+ (m− 0)2

= m2 4 +m2

4. (5)

Pour BC2

BC2 = |zB − zC |2

= (0− 0)2 +µ− 4m−m

¶2=

(4 +m2)2

m2. (6)

Pour BD2

BD2 = |zB − zD|2µ0− m2 + 4

2

¶2+ (m−m)2

=(4 +m2)

2

4. (7)

Enfin pour CD2

CD2 = |zC − zD|2µ0− m2 + 4

2

¶2+

µ− 4m−m

¶2=

(4 +m2)3

4m2. (8)

D’après (3), (4) et (5) nous avons la relation

AC2 ×AD2 = 44 +m2

m2m2 4 +m2

4=¡4 +m2

¢2= AB2

ce qui établit que l’assertion 2 a) est vraie.

Cette assertion est une relation classique du triangle rectangle qui exprime que la hauteurrelative à l’hypoténuse est la moyenne proportionnelle des segments qu’elle détermine surcelle ci.

D’après (3), (4) et (7) nous avons la relation

AC2 +AB2 = 44 +m2

m2+ 4 +m2 =

(4 +m2)2

m26= BD2 =

(4 +m2)2

4

ce qui établit que l’assertion 2 b) est fausse. Nous aurions pu aussi remarquer que lepremier membre n’est autre qu’une partie du théorème de Pythagore et en conclure qu’ilest égal à BC2 ce qui est en général différent de BD2 sauf pour m = 2.

Corrigés 53

D’après (3), (6), (7) et (8) nous avons la relation

AB2 × CD2 =¡4 +m2

¢ (4 +m2)3

4m2=(4 +m2)

4

4m2=(4 +m2)

2

4

(4 +m2)2

m2= BD2 ×BC2

ce qui établit que l’assertion 2 c) est vraie.

Cette assertion est une autre relation classique du triangle rectangle qui exprime que leproduit des deux côtés de l’angle droit est égal à celui de l’hypoténuse et de sa hauteurrelative.

Il est en principe inutile de vérifier que l’assertion 2 d) est fausse car ce type de QCM nepeut comporter qu’au plus deux assertions exactes par question.

Toutefois pour le plaisir, nous contrôlons.

D’après (3), (4) et (8) nous avons la relation

CA2 × CD2 = 44 +m2

m2

(4 +m2)3

4m2=(4 +m2)

4

m46= ¡4 +m2

¢4= AB4

ce qui établit que l’assertion 2 d) est fausse.

Cette assertion est une autre relation classique d’un triangle rectangle mais mal exprimé.

En effet un côté de l’angle de droit est la moyenne proportionnelle de l’hypoténuse et desa projection sur celle ci.

Dans notre cas nous aurionsCA× CD = CB2.

Nous avons les réponses :

Question 02 : a) Vraie b) Fausse c) Vraie d) Fausse e) Fausse

Nous supposons dans ce qui suit que m > 0 et nous utilisons les similitudes pour établirles relations des assertions 2 a) et 2 c).

Les angles orientés (sens trigonométrique)³−→CA,

−−→CB

´et³−→BA,

−−→BD

´sont égaux (cf figure

2.1), donc les deux triangles rectangles CAB et BAD sont semblables (ie : il existe unesimilitude s telle que l’image du triangle CAB soit le triangle BAD).

Or une similitude fait intervenir une homothétie de rapport k > 0.

Ce rapport peut être obtenu à partir des longueurs des côtés des triangles CAB et BADBA

CA=

BD

CB=

AD

AB= k .

Nous en déterminons la relation de l’assertion 2 a) en prenant le premier et le troisièmerapport.

De manière analogue, les angles orientés (sens trigonométrique)³−→CA,

−−→CB

´et³−−→CD,

−−→CB

´sont égaux (il y a parfois des évidences), donc les deux triangles rectangles CAB et CBDsont semblables (ie : il existe une similitude s0 telle que l’image du triangle CAB soit letriangle CBD).

Or comme précédemment une similitude fait intervenir une homothétie de rapport k0 > 0.

Ce rapport peut être obtenu à partir des longueurs des côtés des triangles CAB et CBD

CB

CA=

BD

AB=

CD

CB= k0 .

Nous en déterminons la relation de l’assertion 2 c) en prenant le premier et le troisièmerapport.

54 Questionnements Automatisables

Nous pouvons (cela n’est pas demandé) calculer k et k0 en déterminant l’expressioncomplexe des similitudes s et s0.

Nous notons s la similitude telle que

s : z −→ Z = a z + b ,

où a et b vérifient le système (cf figure 2.2)

s (C) = B ⇔ mi = −4 im

a + b

s(A) = A ⇔ 2 = 2 a + b

s (B) = D ⇔ m2 + 4

2+mi = mia + b

.

A l’aide des deux premières équations nous obtenons a = −mi

2et b = 2+mi , la troisième

équation nous permettant de vérifier

s : z −→ Z = −mi

2z + 2 +mi .

........

........

........

........

........

........

........

........

........

........

........

........

........

........

........

........

........

........

........

........

........

........

........

........

........

........

........

........

........

........

........

........

........

........

........

........

........

........

........

........

........

........

........

........

........

........

........

........

........

........

...........................

..........................

y

................................................................................................................................................................................................................................................................................................................................................................................................................................................................................................................................................................................................................................................... ..........................xO A

B

C

D

m

2

.........................................................................................................................................................................................................................................................................................................................................................................................................................................................................................................................................................................................................................

...............................................................................................................................................................................................................................................................................................................................................................................................................................................................................................................................................................................................................................................................................................

............................................................................................................................................................................................................................................................................ ....................................C 0

B0

................................................................................................................................................................................................................................................

.............................................................................................................................................

..................................

........................................................................................................................................................................................................................................................

Figure 2.2

Ainsi le rapport k de l’homothétie et l’angle θ de la rotation sont

k = |a| = m

2et θ = Arg a = −π

2.

En conclusion s est la similitude de centre A d’angle θ = −π2et de rapport k =

m

2.

Nous décomposons la similitude s en deux transformations

CABRotation (A,−π / 2)−−−−−−−−−−−→ C 0AB0 Homothétie (A, k)−−−−−−−−−−→ BAD

La similitude s0 n’est pas du même type que la similitude s . C’est une similitude diteindirecte alors que s est une similitude directe.

Il faut faire en premier, une symétrie par rapport à l’axe des imaginaires car le pointinvariant C est situé sur cet axe.Ce qui revient à remplacer l’affixe z d’un point par l’opposé de son conjugué −z .Nous notons s0 la similitude telle que

s0 : z −→ Z = −a z + b ,

Corrigés 55

où a et b vérifient le système (cf figure 2.3)

s0 (C) = B ⇔ −4 im

= −4 im

a + b

s0(A) = A ⇔ mi = −2 a + b

s0 (B) = D ⇔ m2 + 4

2+mi = mia + b .

A l’aide des deux premières équations nous obtenons a =2−mi

2et b = 2 , la troisième

équation nous permettant de vérifier

s0 : z −→ Z =2−mi

2z + 2 .

........

........

........

........

........

........

........

........

........

........

........

........

........

........

........

........

........

........

........

........

........

........

........

........

........

........

........

........

........

........

........

........

........

........

........

........

........

........

........

........

........

........

........

........

........

........

........

........

........

........

...........................

..........................

y

.............................................................................................................................................................................................................................................................................................................................................................................................................................................................................................................................................................................................................................................................................................................................. ..........................x

O

A

B

C

Dm

2

.........................................................................................................................................................................................................................................................................................................................................................................................................................................................................................................................................................................................................................

.......................................................................................................................................................................................................................................................................................................................................................................................................................................................................................................................................................................................................................................................................................................................................................

........................................................................................................................................................................................................................................................................................................................................................................................................

............................................................................................................................................................................................................................................................

.................................................................................................................................................................................................................................................. ...........

.............. ...........

A0

A00

B00

................................................................................................................................................................................................................................................

................................................................................................................................................................................................................

...................................................................................................

............................................................................................................................................................................................................................

Figure 2.3

Ainsi le rapport k0 de l’homothétie et l’angle θ0 de la rotation sont

k0 = |a| =√m2 + 4

2et θ0 = Arga = −Arctanm

2,

où θ0 = −Arctanm2désigne l’angle aigu négatif tel que tan θ0 = −m

2.

En conclusion s0 est la similitude indirecte obtenue par la symétrie d’axe Oy et de lasimilitude directe de centre C d’angle θ0 et de rapport k0 .

Nous décomposons la similitude indirecte s0 en trois transformations

CABSymétrieOy−−−−−−−→ CA0B

Rotation (C, θ0)−−−−−−−−→ CAB00 Homothétie (C, k0)−−−−−−−−−−→ CBD

Reprenons maintenant l’ordre des questions.

Ω étant un point quelconque du plan affine.

Par définition : Si et seulement si α+ β + γ 6= 0 alors il existe un unique point D définipar l’égalité vectorielle

−→ΩD =

α−→ΩA+ β

−→ΩB + γ

−→ΩC

α+ β + γ. (9)

D est le barycentre des points A, B et C affectés des coefficients respectifs α, β et γ.

Il est fréquent de remplacer Ω par D dans la relation (9) mais cela fait disparaître ledénominateur, ce qui est fort regrettable pour ceux qui n’ont pas une excellente mémoire.

D’autre part si nous choisissons Ω en O la relation (9) donne les coordonnées dubarycentre.

56 Questionnements Automatisables

Les trois points A, B et C n’étant pas alignés, nous savons que tout pointM du plan peutêtre considéré comme le barycentre de ces trois points affectés de coefficients convenables.

Il est possible de choisir les coefficients de telle manière que leur somme soit égale à 1.Dans ces conditions ces coefficients s’appellent coordonnées barycentriques du point M.

Ainsi l’assertion 4 a) est fausse. ATTENTION : Encore une assertion à moitié vraie et àmoitié fausse. C’est un piège pour ceux qui lisent trop vite ...

En prenant Ω en D la relation (9) devient

α−−→DA+ β

−−→DB + γ

−−→DC = 0 ,

ce qui permet d’affirmer que β = 0 car les points C, A et D sont alignés.

Ainsi l’assertion 4 b) est fausse.

Le point D est, pour tout m, extérieur au segment [CD] donc les vecteurs−−→DA et

−−→DC

sont de même sens.Il en résulte que α et γ sont de signes opposés et en valeur absolue tels que

DC

DA= −α

γ.

Or pour m = 4 les coordonnées des points A, C et D sont

A(2, 0), C(0, −1) et D(10, 4) ,

soitα

γ= −

q(0− 10)2 + (−1− 4)2q(2− 10)2 + (0− 4)2

= −54.

........

........

........

........

........

........

........

........

........

........

........

........

........

........

........

........

........

........

........

........

........

........

........

........

........

........

........

........

........

........

........

........

........

........

........

........

........

........

........

........

........

........

........

........

........

........

........

........

........

........

........

........

........

........

........

........

........

........

........

........

........

..........................

..........................

y

............................................................................................................................................................................................................................................................................................................................................................................................................................................................................................. ..........................xO

Ensemble des

points I

A

B

C

I

........................................................................................................................................................................................................................................................................................................................................................................................................................................................................................................................

..................................................................................................................................................................................................................................................................................................................................................................................................................................................................................................................................................................

.......................................................................................................................................

................

................

................

................

................

............

.........................................................................................................................................................................................

................................................................................................................................

........................................................................................................

Figure 2.4

Nous pouvons accepter l’assertion 4 c) et refuser l’assertion 4 d).

Nous avons les réponses :

Question 04 : a) Fausse b) Fausse c) Vraie d) Fausse e) Fausse

Encore un peu d’analytique dans le cas où m est quelconque.

Nous savons que le point de concours I des médianes du triangle ABC est l’isobarycentredes sommets. Si nous choisissons α = β = γ = 1 alors la relation (9) devient en prenantΩ en O

−→OI =

−→OA+

−−→OB +

−→OC

3.

Corrigés 57

D’où les coordonnées de l’isobarycentre sont

I

µ1

3(2 + 0 + 0) =

2

3,1

3

µ2 +m− 4

m

¶=

m2 − 43m

¶.

Ainsi l’assertion 5 a) est vraie et l’assertion 5 b) est fausse.

L’abscisse de I étant constante, il est évident que l’ensemble des points I lorsque m

décrit R∗ est situé sur la droite dont une équation est x =2

3. L’étude de la fonction

φ (m) =m2 − 43m

prouve que toute cette droite est décrite par I.

Nous avons les réponses :

Question 05 : a) Vraie b) Fausse c) Vraie d) Fausse e) Fausse

Toute translation conserve les propriétés des figures, donc en particulier J est le point deconcours des médianes du triangle AEF qui est rectangle en F.

Ainsi les deux assertions 6 a) et 6 b) sont fausses.

De même−→CA =

−→IJ donc ne peut être constant. En effet ses composantes sont

µ2,4

m

¶.

Donc l’assertion 6 c) est fausse.

Enfin l’assertion 6 d) est vraie d’après 5 c) car l’ensemble des points J est déduit del’ensemble des points I par la translation t.

ATTENTION : Il est fréquent que les questions soient liées, deux réponses contradictoiresentraînent obligatoirement une pénalité qui est souvent modulée en fonction de la gravitéde la faute commise.

Nous avons les réponses :

Question 06 : a) Fausse b) Fausse c) Fausse d) Vraie e) Fausse

03

Une figure ne prouve rien mais permet souvent de formuler des conjectures.

Dans la première question (cf figure 3.1) une figure bien faite, à la règle et au compas,donne toutes les réponses.

Le quadrilatère ABDC est un rectangle mais non un carré car AC = 2AB donc l’assertion1 a) est fausse. Remarquer que si ce quadrilatère était un carré alors les diagonalesseraient perpendiculaires, nous rencontrerons souvent des assertions à moitié vraie et àmoitié fausse.

Le point de concours des droites AD et BC est le centre ω du rectangle ABDC doncc’est bien l’isobarycentre des sommets.

Donc l’assertion 1 b) est vraie.

Les sommets de ce rectangle sont sur un cercle de centre ω et de rayon m

√5

2.

Donc l’assertion 1 c) est vraie.

58 Questionnements Automatisables

Cette assertion permet de vérifier votre vocabulaire mathématique.

En principe pour ce type de QCM, la quatrième assertion est obligatoirement fausse.

C’est une évidence car le triangle BCD est rectangle en D et non en B.

........

........

........

........

........

........

........

........

........

........

........

........

........

........

........

........

........

........

........

........

........

........

........

........

........

........

........

........

........

........

........

........

........

........

........

........

........

........

........

........

........

........

........

........

........

........

........

........

........

........

........

........

........

........

........

........

........

........

........

........

........

........

...............................

..........................

y

........................................................................................................................................................................................................................................................................................................................................................................................................................................................................................................................................................................................................................................................................................................................................................................... ..........................x

AB

CD

I

J

ω

m−2m

2m

3m4

3m2

.........................................................................................................................................................................................................................................................................................................................................................................................................................................................................................................................................................................................................................................................................................................................................................................................................................................................................................................................................................................................................................................................................................................................................................................................................................................................................................................................................................................................................................................................................................................................................................................................................................................

.......................................................................................................................................................................................................................................................................................................................................................................................................................................................

................................................................................................................................................................................................................................................................................................................................................................................................................................................................

....

.....

....

.....

....

.....

....

.....

....

.....

....

.....

....

.....

....

.....

....

.....

....

.....

.....

.....

.....

.....

.....

....

.....

....

.....

....

.....

....

.....

....

.....

....

.....

....

.....

....

.....

....

.....

....

.....

....

.....

.

................................................................................................................................................................................................................................................

Figure 3.1

Nous avons les réponses :

Question 01 : a) Fausse b) Vraie c) Vraie d) Fausse e) Fausse

O étant un point quelconque du plan affine euclidien P, pour α+ β + γ 6= 0, il existe ununique point M tel que

−−→OM =

α−→OA+ β

−−→OB + γ

−→OC

α+ β + γ, (1)

appelé barycentre des points A, B et C affectés des coefficients respectifs α, β et γ.

Pour I barycentre des points A, B et C affectés des coefficients respectifs 2, −2 et 1,l’équation (1) s’écrit −→

OI = 2−→OA− 2−−→OB +−→OC . (2)

Si nous prenons O en C alors (2) devient−→CI = 2

−→CA− 2−−→CB

= 2−→CA+ 2

−−→BC

= 2−→BA

= 2−−→DC ,

ce qui n’est autre que l’assertion 2 a), qui est donc vraie.

La figure nous permet de conjecturer que les droites BC et AI ne sont pas parallèles,donc que l’assertion 2 b) est fausse.

Toutefois comme une figure ne prouve rien ... nous pouvons remplacer dans (2) le pointO par A, ce qui donne

−→AI = −2−→AB + −→AC (2’)

= −−→AB +−→BA+ −→AC= −−→AB +−−→BC,

Corrigés 59

et nous permet de re-conclure.

Si dans l’équation (2) nous remplaçons le point O par D alors

−→DI = 2

−−→DA− 2−−→DB +

−−→DC

= 2−−→DA+ 2

−−→BD +

−−→DC

= 2−→BA+

−−→DC

= 3−→BA ,

ce qui est l’assertion 2 c) à un signe près ...et est donc fausse.

Le point J est défini de manière analogue au point I en utilisant l’équation (1) avecα = −2, β = 3 et γ = 3

−→OJ =

−2−→OA+ 3−−→OB + 3−→OC4

. (3)

Si nous prenons O en A, l’équation (3) devient−→AJ =

3

4

³−→AB +

−→AC´

(3’)

=3

4

−−→AD

ce qui prouve que l’assertion 2 d) est vraie.

Nous avons les réponses :

Question 02 : a) Vraie b) Fausse c) Fausse d) Vraie e) Fausse

Pour la question suivante, nous pouvons utiliser la figure 3.1, dans l’hypothèse où celle ciest très bien faite, ou associer un repère Axy au plan P et contrôler analytiquement lesassertions proposées.

La figure a l’avantage de la rapidité, mais dans certains cas il peut y avoir doute ... nousle verrons plus loin ...

Dans le repère Axy les coordonnées de B et C sont

B (m, 0) et C (0, 2m) .

D’après (20) les coordonnées de I sont

I (−2m, 2m)

et d’après (30) ceux de J sont

J

µ3m

4,3m

2

¶.

La figure 3.1 permet d’envisager que l’assertion 3 c) est vraie et que les autres assertionsde la question 03 sont fausses. Nous contrôlons ceci analytiquement.Une équation de la droite IB est de la forme y = a x+ b, avec a et b vérifiant le système½

2m = −2ma + b0 = ma + b

exprimant que les points I et B sont situés sur cette droite.

Après calcul, nous obtenons a = −23et b =

2m

3, d’où l’équation de IB est

y = −23x+

2m

3.

60 Questionnements Automatisables

b est l’ordonnée du point d’intersection des droites IB et AC, ce qui prouve que l’assertion3 a) est fausse.

De manière analogue une équation de la droite CJ est de la forme y = a0 x + b0, avec a0

et b0 vérifiant le système (2m = 0 a0 + b03m

2=

3m

4a0 + b0

exprimant que les points C et J sont situés sur cette droite.

Après calcul, nous obtenons b0 = 2m et a0 = −23, d’où l’équation de CJ est

y = −23x+ 2m.

Les coefficients directeurs a et a0 de ces deux droites sont égaux donc les deux droites IBet CJ sont parallèles ce qui prouve que l’assertion 3 c) est vraie.

Nous développons une autre technique pour les assertions 3 b) et 3 d).

Les composantes des vecteur−→IA,−→CJ et

−−→AD sont

−→IA

¯0− (−2m) = 2m0− 2m = −2m,

−→CJ

¯¯¯3m

4− 0 =

3m

4

3m

2− 2m = −m

2

et −−→AD

¯m− 0 = m2m− 0 = 2m.

Ce qui permet de re-vérifier, car 2 c) est vraie, que−→IA n’est pas colinéaire à

−→CJ, donc

l’assertion 3 b) est fausse et que−−→AD .

−→CJ = m ∗ 3m

4− 2m ∗ m

2= −m

2

46= 0 ,

donc les deux droites AD et CJ ne sont pas perpendiculaires et ainsi l’assertion 3 d) estfausse.

Nous répétons encore une dernière fois ... au cas où .. qu’une figure très bien faite permetde répondre à toutes les assertions de la question 3.

Nous avons les réponses :

Question 03 : a) Fausse b) Fausse c) Vraie d) Fausse e) Fausse

Une similitude dans C est parfaitement déterminée par les deux points B et A et leursimages respectives A et I , ceci sans condition particulière.

Donc l’assertion 4 a) est fausse, ce qui sera vérifié dans la suite.

La relation Z = a z + b est déterminée par le système½0 = ma + b

−2m+ 2mi = 0 + b ,

défini par les affixes des points B et A et de leurs images respectives A et I.

Ce qui donne b = 2m (−1 + i) et a = 2 (1− i), c’est un contrôle pour l’assertion 4 a).

Corrigés 61

La similitude s est donc définie par

s : z −→ Z = 2 (1− i) z + 2m (−1 + i) . (4)

Nous en déduisons que l’assertion 4 b) est vraie et que l’assertion 4 c) est fausse.

Pour l’assertion 4 d), nous devons faire le calcul car le triangle IAJ pourrait être l’imagepar la similitude s du triangle rectangle ABD.

Il est nécessaire pour cela que l’angle³−→AI,−→AJ´soit droit, ce qui n’est pas évident sur la

figure ... une figure peu tromper énormément ...sauf si elle est parfaite.

D’après (20) et (30) le produit scalaire

−→AI .−→AJ =

3

4

³−2−→AB +−→AC

´·³−→AB +

−→AC´

= −32

−→AB2 +

3

4

−→AC2

=3m2

2

est non nul donc l’assertion 4 d) est fausse.

Une autre manière de contrôler l’assertion 4 d) est d’utiliser l’expression complexe (4) dela similitude s.

Pour cela il faut vérifier l’égalité

zJ?= 2 (1− i) zD + 2m (−1 + i)

c’est-à-dire3m

4(1 + 2 i)

?= 2 (1− i)m (1 + 2 i) + 2m (−1 + i) ,

ce qui après calcul n’est pas vérifié (c’est une évidence à cause du1

4du premier membre).

Nous avons les réponses :

Question 04 : a) Fausse b) Vraie c) Fausse d) Fausse e) Fausse

Le point invariant Ω de la similitude est la solution dans C de l’équation

zΩ = 2 (1− i) zΩ + 2m (−1 + i) ,

soit

zΩ =2m

5(3 + i) .

La figure 3.2 permet de constater que Ω ne peut être sur la médiatrice de [AB].

Donc l’assertion 5 a) est fausse.

Nous savons que la similitude [1] s est le produit commutatif d’une rotation r (Ω, θ) decentre Ω et d’angle θ et d’une homothétie h (Ω, k) de centre Ω et de rapport strictementpositif k.

Par hypothèse sur s nous avons les relations

ΩA

ΩB=

ΩI

ΩA= k

[1]Nous rappellons que le programme de terminale n’envisage que les similitudes directes, donc pournous toute similitude est directe, sauf spécification contraire ... cf exemple d’une similitude indirectedans l’exercice 02.

62 Questionnements Automatisables

et ³−→ΩB,

−→ΩA´=³−→ΩA,

−→ΩI´= θ . (5)

Ce qui prouve que les assertions 5 b) et 5 c) sont vraies, cf figure 3.2.

Il est inutile, en principe, de vérifier que l’assertion 5 d) est fausse.

A titre de complément, montrons que le triangle KΩI est un triangle rectangle en Ω.

En effet les coordonnées des vecteurs−→ΩB et

−→ΩI sont

−→ΩB

¯¯¯m− 6m

5= −m

5

0− 2m5

= −2m5

,

−→ΩI

¯¯¯−2 m− 6m

5= −16m

5

2m− 2m5

=8m

5.

Nous en déduisons le produit scalaire

−→ΩB .

−→ΩI =

³−m5

´×µ−16m

5

¶+

µ−2m5

¶×µ8m

5

¶= 0 , (6)

et le résultat.

Pour établir que le triangle AΩK n’est pas isocèle, nous avons deux possibilités :

•Montrer que les longueurs des côtés AΩ, AK et ΩK sont différentes.

• Evaluer les angles (AK, AΩ) et (KA, KΩ) car (ΩK, ΩA)=π

4d’après les relations (5)

et (6).

La première est plus courte car il suffit de déterminer les composantes des vecteurs−→AΩ,

−−→AK et

−−→ΩK.

Une équation de la droite AK, ou ce qui est identique AI, est de manière évidente

y = −x .Une équation de la droite ΩK, ou ce qui est identique ΩB, est de la forme y = a x + b,où a et b vérifient le système ( 2m

5=

6m

5a + b

0 = ma + b ,

qui exprime que les points Ω et B sont situés sur cette droite.

Après calcul, nous obtenons a = 2 et b = −2m, d’où une équation de ΩK est

y = 2x− 2m.

Corrigés 63

........

........

........

........

........

........

........

........

........

........

........

........

........

........

........

........

........

........

........

........

........

........

........

........

........

........

........

........

........

........

........

........

........

........

........

........

........

........

........

........

........

........

........

........

........

........

........

........

........

........

........

........

........

........

........

........

........

........

........

........

........

........

........

........

........

........

........

........

........

........

........

........

........

........

........

........

........

...............................

..........................

y

............................................................................................................................................................................................................................................................................................................................................................................................................................................................................................................................................................................................................................................................................................................................................................................................... ..........................x

A B

I

J

D

K

Ω

6m5

2m5

2m3

−2m3

Bissectrice de l’angle (−→ΩB,

−→ΩI)

................................................................................................................................................................................................................................................................................................................................................................................................................................................................

...............................................................................................................................................................................................................................................................................................................................................................................................................................................................................................................................................................................................................................................................................................................................................................................................................................................................................................................................................................................................................................................................................................................................................................................................................................................................................................................................................................................................................................................................................................................................................................................................

..........................................................................................

........

........

........

........

........

........

........

........

........

........

........

........

........

........

........

........

........

........

........

........

........

........

........

........

........

........

........

........

........

........

........

........

........

........

........

........

........

........

........

........

........

........

........

........

........

........

........

........

........

...

...........................................

.........................................................................................................................................................

.......................................................................................................................................

......

......

......

......

......

......

......

......

......

......

......

....

.....

....

.....

....

.....

....

.....

....

....................................................................................................................................

................................................................................................................................................ ....

.....

....

.....

....

.....

....

.....

....

.....

....

.....

....

.....

....

.....

....

.....

....

.....

....

.....

.....

.....

....

.....

....

.....

....

.....

....

.....

....

.....

....

.....

....

.....

....

.....

....

.....

....

.....

....

.....

....

........................................................................................................................................................................................................................................................................................................................................................................................................................................................................................

....

.....

....

.....

....

.....

....

.....

....

.... ........................................................................................................................................................................................................................................................................................................ .........

..............................................................................................................................................................................................

Figure 3.2

Les coordonnées de K sontµ2m

3, −2m

3

¶.

Les composantes des vecteurs−−→ΩK,

−−→AK et

−→AΩ sont

−−→ΩK

¯¯¯2m

3− 6m

5= −8m

15

−2m3− 2m

5= −16m

15,

−−→AK

¯¯¯2m

3

−2m3

,

−→AΩ

¯¯¯6m

5

2m

5.

Ce qui permet de calculer les normes de ces vecteurs

−→AΩ2 =

µ6m

5

¶2+

µ2m

5

¶2=8m2

5,

−−→AK2 =

µ2m

3

¶2+

µ−2m3

¶2=8m2

9,

−−→ΩK2 =

µ−8m15

¶2+

µ−16m15

¶2=64m2

45.

Ce qui prouve que le triangle AΩK ne peut être isocèle.

Remarque : Le rapportAΩ

ΩKest de 1.06, ce qui est proche de 1. Donc ATTENTION a

une lecture trop rapide de la figure ...

64 Questionnements Automatisables

Nous avons les réponses :

Question 05 : a) Fausse b) Vraie c) Vraie d) Fausse e) Fausse

De manière classique la fonction→f : M −→

→f (M) = α

−−→AM + β

−−→BM + γ

−−→CM

est dite fonction vectorielle de Leibniz.

Si la somme des coefficients α + β + γ est non nulle, nous introduisons le barycentre Gdes points A, B et C affectés des coefficients respectifs α, β et γ.

Nous obtenons→f (M) = α

³−→AG+

−−→GM

´+ β

³−−→BG+

−−→GM

´+ γ

³−→CG+

−−→GM

´= α

−→AG++β

−−→BG+ γ

−→CG+(α+ β + γ)

−−→GM

= (α+ β + γ)−−→GM

par définition du barycentre G.

........

........

........

........

........

........

........

........

........

........

........

........

........

........

........

........

........

........

........

........

........

........

........

........

........

........

........

........

........

........

........

........

........

........

........

........

........

........

........

........

........

........

........

........

........

........

........

........

........

........

........

........

........

........

........

........

........

........

........

........

........

........

........

........

........

........

........

...............................

..........................

y

....................................................................................................................................................................................................................................................................................................................................................................................................................................................................................................................................................................................... ..........................x

AB

CI

J

............................................................................................................................................................................................................................................................................................................................................................................................................................................................................................................................................................................................................................................................................................................................................................................................................................................................................................................................................................................................................................................................................................................

........

........

........

........

........

........

........

........

........

........

........

........

........

........

........

........

........

........

........

........

........

........

........

........

........

........

........

........

........

........

........

........

........

........

........

........

...........................................................................................................................................................

....................................................................................................................................................................................................................................................................................................................................................................................................................................................................................................................................................................................................................................................................................................................................................................................

.......................................................................................................................................................................................................... ................

E1

............................................................................................... ..

.. .... .... .... .... .... .... ....

**************************************************************************************************************************************************************************************************************************************************************************************************************************************************************************************************************************************************************************

Figure 3.3

En appliquant ce qui précède pour α = 2, β = −2 et γ = 1, puis pour = −2, β = 3 etγ = 3, nous obtenons les équations

2−−→AM − 2−−→BM +

−−→CM =

−−→IM (7)

et−2 −−→AM + 3

−−→BM + 3

−−→CM = 4

−−→JM (8)

D’après (7) et (8) l’ensemble E1 est l’ensemble des points M du plan affine euclidien Ptels que

IM = JM .

Ce qui caractérise la médiatrice du segment [IJ ], cf figure 3.3.

Une équation de E1 est obtenue avec −−→IM2 =

−−→JM2

soit en prenant M (x, y)

(x+ 2m)2 + (y − 2m)2 =µx− 3m

4

¶2+

µy − 3m

2

¶2

Corrigés 65

et après réduction11

2x− y +

83m

16= 0.

Nous en déduisons que seule l’assertion 6 b) est vraie et que les assertions 6 a), 6 c) et 6d) sont fausses.

ATTENTION : Il est clair qu’il ne peut y avoir plus d’une réponse Vraie à la question6. C’est un excellent moyen de contrôler les réponses au hasard ou trop rapide ... Soyezprudent car dans ce cas les pénalités sont plus fortes ...

Nous avons les réponses :

Question 06 : a) Fausse b) Vraie c) Fausse d) Fausse e) Fausse

De manière encore classique la fonction

f : M −→ f(M) = α−−→AM2 + β

−−→BM2 + γ

−−→CM2

est dite fonction scalaire de Leibniz.

Si la somme des coefficients α + β + γ est non nulle, nous introduisons comme pour lafonction vectorielle de Leibniz le barycentreG des pointsA, B et C affectés des coefficientsrespectifs α, β et γ.

........

........

........

........

........

........

........

........

........

........

........

........

........

........

........

........

........

........

........

........

........

........

........

........

........

........

........

........

........

........

........

........

........

........

........

........

........

........

........

........

........

........

........

........

........

........

........

........

........

........

........

........

........

........

................................

..........................

y

................................................................................................................................................................................................................................................................................................................................................................................................................................................................................ ..........................x

AB

CI

............................................................................................................................................................................................................................................................................................................................................................................................................................................................................................................................................................................................................................................

.....................................................................................................

..........................

E2

*********************************

*******

*********************************************************************************************************************************************

***************************************

Figure 3.4

Nous obtenons alors

f(M) = α³−→AG+

−−→GM

´2+β

³−−→BG+

−−→GM

´2+γ

³−→CG+

−−→GM

´2= α

−→AG

2+β−−→BG

2+γ−→CG

2+ 2

³α−→AG++β

−−→BG+ γ

−→CG

´.−−→GM+(α+ β + γ)

−−→GM

2

= α−→AG

2++ β

−−→BG

2+γ−→CG

2+ (α+ β + γ)

−−→GM

2

par définition du barycentre G.

Nous déduisons de ce qui précède, pour α = 2, β = −2 et γ = 1,f (M) = 2

−−→AM

2 − 2−−→BM2+−−→CM

2= 2−→AI

2 − 2−→BI2 + −→CI2 +−−→IM2. (9)

Le calcul du scalaire 2−→AI

2−2−→BI2+−→CI2 est obtenu à l’aide des coordonnées des vecteurs2−→AI

2 − 2−→BI2 + −→CI2 = 2£(−2m)2 + (2m)2¤

−2 £(−2m−m)2 + (2m)2¤− (−2m)2

= −6m2

66 Questionnements Automatisables

Il résulte d’après (9) que l’ensemble E2 est caractérisé par

MI = m.

E2 est donc le cercle de centre I et de rayon m, cf figure 3.4.

Il est évident que ce cercle ne passe pas par le point C.

Nous en déduisons que l’assertion 7 c) est vraie et que les assertions 7 a), 7 b) et 7 d)sont fausses.

Nous avons les réponses :

Question 07 : a) Fausse b) Fausse c) Vraie d) Fausse e) Fausse

04

Les figures 4.1 et 4.2 permettent de répondre à la question 01, à condition toutefois debien identifier les courbes C1 ∪ C2, et C1 ∪ C3. C’est plutôt un exercice de lecture que demathématiques.

........

........

........

........

........

........

........

........

........

........

........

........

........

........

........

........

........

........

........

........

........

........

........

........

........

........

........

........

........

........

........

........

........

........

........

........

........

........

........

.................................

..........................

y

......................................................................................................................................................................................................................................................................................................................................................................................................................... ..........................xO

..................................................................................................................

..........................

C1

............................................................................................................................................

C2

*****************************************************************************************************************************************************************************************************************************************************************************************************************************************************************************************************************************************

***************************************************************************************************************************************************************************************************************************************************************************************************************************************

**************************************************************************

************************

........

........

........

........

........

........

........

........

........

........

........

........

........

........

........

........

........

........

........

........

........

........

........

........

........

........

........

........

........

........

........

........

........

........

........

........

........

........

........

.................................

..........................

y

......................................................................................................................................................................................................................................................................................................................................................................................................................... ..........................xO

..................................................................................................................

..........................

C1

..................................................................................................................

..........................

C3

*****************************************************************************************************************************************************************************************************************************************************************************************************************************************************************************************************************************************

*****************************************************************************************************************************************************************************************************************************************************************************************************************************************************************************************************************************************

Figure 4.1 Figure 4.2

Nous avons les réponses :

Question 01 : a) Fausse b) Fausse c) Vraie d) Fausse e) Fausse

La courbe C est la réunion des deux courbes d’équationy =

1

xet y = −1

x,

que nous pouvons écrire sous la forme

(x y − 1) (x y + 1) = x2y2 − 1 = |x y|2 − 1 = 0. (1)

Ce qui prouve que les assertions 2 a) et 2 c) sont vraies et que les assertions 2 b) et2 d) sont fausses. Ceci peut être établi directement, car |1| = 1 donc l’assertion 2 c) n’estautre que l’équation de C1 ∪ C3, et l’assertion 2 d) est fantaisiste.Nous avons les réponses :

Question 02 : a) Vraie b) Fausse c) Vraie d) Fausse e) Fausse

Corrigés 67

........

........

........

........

........

........

........

........

........

........

........

........

........

........

........

........

........

........

........

........

........

........

........

........

........

........

........

........

........

........

........

........

........

........

........

........

........

........

........

........

........

........

........

........

........

........

........

........

........

........

........

........

........

........

........

........

........

........

..................................

..........................

y

........................................................................................................................................................................................................................................................................................................................................................................................................................................................................................................................................................................................................................................................................................................................................................................................................................................................................................................................................................ ..........................x

O−2 2−1

212

2

12A

B C

D

................................................................................................................................................................................................................................................................................................................................................................................................................................................................................................................................................................................................................................................................................................................................................................................................................................................................................................................................................................................................................................................................................................................................................................................................................................................................................................................................................................................................................................................................................................................................................................

.............................................................................................................................................................................................................................................................................................................................................................................................................................................................................................................................................................................................................................................

.............................................................................................................................................................................................................................................................................................................................................................................................................................................................................................................................................................................................................................................

∆1∆2 C1......................................................................................................................................................

.......................

C4........................................................................................................................................................

.....................

......................................................................

. ... ... ... ... ...

*************************************************************************************************************************************************************************************************************************************************************************************************************************************************************************************************************************************************

***************************

***********************

********************

***********************************************************************************************************************************************************************************************************************************************************************************************************

***********************************

************************************************************************************************************************************

***********************************

************************************************************************************************************************************

Figure 4.3

Il est clair, d’après la figure 4.3, représentant les courbes C1 et C4, que les points A, B, Cet D ne sont pas alignés et que la quadrilatère ABCD ne peut être un rectangle, c’estseulement un trapèze isocèle, donc les assertions 3 a) et 3 b) sont fausses.

L’assertion 3 c) est vraie, car la branche C4 admet bien pour axe de symétrie la deuxièmebissectrice ∆2.

L’assertion 3 d) est un piège, il en existe beaucoup dans les QuestionnementsAutomatisables. Ce type d’assertion permet de vérifier que les candidats savent lire.[1]

Nous avons les réponses :

Question 03 : a) Fausse b) Fausse c) Vraie d) Fausse e) Fausse

Pour les assertions de la question 4, nous avons deux expressions intégrales et deux valeurs,dont une au plus est juste, ce qui est évident.

L’assertion 4 a) est très fantaisiste d’après (1) , donc cette intégrale est nulle.

L’assertion 4 d) propose de décomposer l’aire de la partie du 1er quadrant (x > 0 ety > 0) en deux aires :

L’aire d’un rectangle :

Z 12

0

2 dx = 1

L’aire limitée par C1 et Ox :

Z 2

12

1

xdx = ln2− ln 1

2= 2 ln 2.

Ce qui n’est pas tout à fait l’assertion 4 d), qui est donc fausse.

Nous en déduisons que l’aire hachurée A est (en unités d’aire)A = 2 + 8 ln 2 .

Le résultat précédent permet d’accepter l’assertion 4 c) et de rejeter 4 b).

Nous avons les réponses :

Question 04 : a) Fausse b) Fausse c) Vraie d) Fausse e) Fausse

[1] Savoir lire et compter, premier principe des mathématiques. Il est fréquent qu’une lecture trop rapidedu texte entraîne des erreurs. ATTENTION.

68 Questionnements Automatisables

g est une fonction définie par morceaux de la manière suivante :

g(x) =

−1x

si −2 6 x 6 −12

2 si −126 x 6 1

21

xsi

1

26 x 6 2

.

Par définition G est la primitive de g nulle en x = −2, donc l’assertion 5 a) est fausse.

G

µ−12

¶ne peut être nulle car la fonction g est strictement positive, G est alors

strictement croissante, donc l’assertion 5 b) est fausse.

Déterminons les expressions de G selon les valeurs de x.

* Pour −2 6 x 6 −12, cf figure ci-dessous.

........

........

........

........

........

........

........

........

........

........

........

........

........

........

........

........

........

........

........

........

........

........

........

........

........

........

........

........

........

........

........

........

........

........

........

........

........

........

........

........

.................................

..........................

y

................................................................................................................................................................................................................................................................................................................................................................................................................................................................................................................................................................................................. ..........................x

O−2 2−12

12

A

B C

D

x.........................................................

........

........

........

........

........

........

........

........

........

........

........

........

........

........

.

.

.

.

.

.

.

.

.

.

.

.

.

.

.

.

.

.

.

.

.

.

.

.

.

.

.

.

.

.

.

.

.

.

.

.

.

.

.

.

.

.

.

.

.

.

.

.

.

.

.

.

.

.

.

.

.

.

.

.

.

.

.

.

.

.

.

.

.

.

.

.

.

.

.

.

.

.

.

.

.

.

.

.

.

.

.

.

.

.

.

.

.

.

.

.

.

.

.

.

.

.

.

.

.

.

.

.

.

.

.

.

.

.

.

.

.

.

.

.

.

.

.

.

.

.

.

.

.

........

........

........

........

........

........

........

.

........

........

........

........

........

........

........

........

........

........

........

........

........

........

.

........

........

........

........

........

........

........

........

........

........

........

........

........

........

........

........

........

........

........

........

........

........

........

........

........

........

........

........

........

........

........

........

........

........

........

........

........

........

........

........

........

........

*******************************************************************************************************************************************************************************************************************************************************************************************************************************************************************************************************************************************************************************************************************************************************

*****************************************************************************************************************************************************************************************************************************************************************************************************************************************************************************************************************************************************************************************************************************************************************************************************************************************************

G (x) =

Z x

−2−dt

t

=h− ln |t|

ix−2

= ln2

|x|= ln

µ−2x

¶,

ce qui prouve que l’assertion 5 c) est vraie.

La valeur de Gµ−12

¶est ln 4, ce qui confirme que l’assertion 5 b) est fausse.

* Pour −126 x 6 1

2, cf figure ci-dessous.

G n’étant pas constante, ce que nous savions déjà, donc l’assertion 5 d) est fausse.

Nous avons les réponses :

Question 05 : a) Fausse b) Fausse c) Vraie d) Fausse e) Fausse

REMARQUE : Un candidat peu sûr de lui peut être déstabilisé par le fait que les questions3, 4 et 5 ont la même réponse c). Dur ... pour les indécis.

........

........

........

........

........

........

........

........

........

........

........

........

........

........

........

........

........

........

........

........

........

........

........

........

........

........

........

........

........

........

........

........

........

........

........

........

........

........

........

........

.................................

..........................

y

................................................................................................................................................................................................................................................................................................................................................................................................................................................................................................................................................................................................. ..........................xO

−2 2−12

12

A

B C

D

x.........................................................

........

........

........

........

........

........

........

........

........

........

........

........

........

........

........

........

........

........

........

........

........

........

........

........

........

........

........

........

..

.

.

.

.

.

.

.

.

.

.

.

.

.

.

.

.

.

.

.

.

.

.

.

.

.

.

.

.

.

.

.

.

.

.

.

.

.

.

.

.

.

.

.

.

.

.

.

.

.

.

.

.

.

.

.

.

.

.

.

.

.

.

.

.

.

.

.

.

.

.

.

.

.

.

.

.

.

.

.

.

.

.

.

.

.

.

.

.

.

.

.

.

.

.

.

.

.

.

.

.

.

.

.

.

.

.

.

.

.

.

.

.

.

.

.

.

.

.

.

.

.

.

.

.

.

.

.

.

.

.

.

.

.

.

.

.

.

.

.

.

.

.

.

.

.

.

.

.

.

.

.

.

.

.

.

.

.

.

.

.

.

.

.

.

.

.

.

.

.

.

.

.

.

.

.

.

.

.

.

.

.

.

.

.

.

.

.

.

.

.

.

.

.

.

.

.

.

.

.

.

.

.

.

.

.

.

.

.

.

.

.

.

.

.

.

.

.

.

.

.

.

.

.

.

.

.

.

.

.

.

.

.

.

.

.

.

.

.

.

.

.

.

.

.

.

.

.

.

.

.

.

.

.

.

.

.

.

.

.

.

.

.

.

.

.

.

.

.

.

.

.

.

.

.

.

.

.

.

.

.

.

.

.

.

.

.

.

.

.

.

.

.

.

.

.

.

.

.

.

.

.

.

.

.

.

.

.

.

.

.

.

.

.

.

.

.

.

.

.

.

.

.

.

.

.

.

.

.

.

.

.

.

.

.

.

.

.

.

.

.

.

.

.

.

.

.

.

.

.

.

.

.

.

.

.

.

.

.

.

.

.

.

.

.

.

.

.

.

.

.

.

.

.

.

.

.

.

.

.

.

.

.

.

.

.

.

.

.

.

.

.

.

.

.

.

.

.

.

.

.

.

.

.

.

.

.

.

.

.

.

.

.

.

.

.

.

.

.

.

.

.

.

.

.

.

.

.

.

.

.

.

.

.

.

.

.

.

.

.

.

.

.

.

.

.

.

.

.

.

.

.

.

.

.

.

.

.

.

.

.

.

.

.

.

.

.

.

.

.

.

.

.

.

.

.

.

.

.

.

.

.

.

.

.

.

.

.

.

.

.

.

.

.

.

.

.

.

.

.

.

.

.

.

.

.

.

.

.

.

.

.

.

.

.

.

.

.

.

.

.

.

.

.

.

........

........

........

........

........

........

........

........

........

........

........

........

........

........

........

........

........

........

........

........

........

........

........

........

........

........

........

........

........

........

........

........

........

........*****************************************************************************************************************************************************************************************************************************************************************************************************************************************************************************************************************************************************************************************************************************

******************************************************************************************************************************************************************************************************************************************************************************************************************************************************************************************************************************************************************************************************************************************************************************************************************************************************************************* G (x) = ln 4 +

Z x

−12

2 dt

= ln 4 +h2 tix−12

= 1 + ln 4 + 2x .

La valeur de Gµ1

2

¶est 2 + ln 4.

Corrigés 69

* Enfin pour1

26 x 6 2, cf figure ci-dessous.

........

........

........

........

........

........

........

........

........

........

........

........

........

........

........

........

........

........

........

........

........

........

........

........

........

........

........

........

........

........

........

........

........

........

........

........

........

........

........

........

.................................

..........................

y

................................................................................................................................................................................................................................................................................................................................................................................................................................................................................................................................................................................................. ..........................xO

−2 2−12

12

A

B C

D

x.........................................................

........

........

........

........

........

........

........

........

........

........

........

........

.......

.

.

.

.

.

.

.

.

.

.

.

.

.

.

.

.

.

.

.

.

.

.

.

.

.

.

.

.

.

.

.

.

.

.

.

.

.

.

.

.

.

.

.

.

.

.

.

.

.

.

.

.

.

.

.

.

.

.

.

.

.

.

.

.

.

.

.

.

.

.

.

.

.

.

.

.

.

.

.

.

.

.

.

.

.

.

.

.

.

.

.

.

.

.

.

.

.

.

.

.

.

.

.

.

.

.

.

.

.

.

.

.

.

.

.

.

.

.

.

.

.

.

.

.

.

.

.

.

.

.

.

.

.

.

.

.

.

.

.

.

.

.

.

.

.

.

.

.

.

.

.

.

.

.

.

.

.

.

.

.

.

.

.

.

.

.

.

.

.

.

.

.

.

.

.

.

.

.

.

.

.

.

.

.

.

.

.

.

.

.

.

.

.

.

.

.

.

.

.

.

.

.

.

.

.

.

.

.

.

.

.

.

.

.

.

.

.

.

.

.

.

.

.

.

.

.

.

.

.

.

.

.

.

.

.

.

.

.

.

.

.

.

.

.

.

.

.

.

.

.

.

.

.

.

.

.

.

.

.

.

.

.

.

.

.

.

.

.

.

.

.

.

.

.

.

.

.

.

.

.

.

.

.

.

.

.

.

.

.

.

.

.

.

.

.

.

.

.

.

.

.

.

.

.

.

.

.

.

.

.

.

.

.

.

.

.

.

.

.

.

.

.

.

.

.

.

.

.

.

.

.

.

.

.

.

.

.

.

.

.

.

.

.

.

.

.

.

.

.

.

.

.

.

.

.

.

.

.

.

.

.

.

.

.

.

.

.

.

.

.

.

.

.

.

.

.

.

.

.

.

.

.

.

.

.

.

.

.

.

.

.

.

.

.

.

.

.

.

.

.

.

.

.

.

.

.

.

.

.

.

.

.

.

.

.

.

.

.

.

.

.

.

.

.

.

.

.

.

.

.

.

.

.

.

.

.

.

.

.

.

.

.

.

.

.

.

.

.

.

.

.

.

.

.

.

.

.

.

.

.

.

.

.

.

.

.

.

.

.

.

.

.

.

.

.

.

.

.

.

.

.

.

.

.

.

.

.

.

.

.

.

.

.

.

.

.

.

.

.

.

.

.

.

.

.

.

.

.

.

.

.

.

.

.

.

.

.

.

.

.

.

.

.

.

.

.

.

.

.

.

.

.

.

.

.

.

.

.

.

.

.

.

.

.

.

.

.

.

.

.

.

.

.

.

.

.

.

.

.

.

.

.

.

.

.

.

.

.

.

.

.

.

.

.

.

.

.

.

.

.

.

.

.

.

.

.

.

.

.

.

.

.

.

.

.

.

.

.

.

.

.

.

.

.

.

.

.

.

.

.

.

.

.

.

.

.

.

.

.

.

.

.

.

.

.

.

.

.

.

.

.

.

.

.

.

.

.

.

.

.

.

.

.

.

.

.

.

.

.

.

.

.

.

.

.

.

.

.

.

.

.

.

.

.

.

.

.

.

.

.

.

.

.

.

.

.

.

.

.

.

.

.

.

.

.

.

.

.

.

.

.

.

.

.

.

.

.

.

.

.

.

.

.

.

.

.

.

.

.

.

.

.

.

.

.

.

.

.

.

.

.

.

.

.

.

.

.

.

.

.

.

.

.

.

.

.

.

.

.

.

.

.

.

.

.

.

.

.

.

.

.

.

.

.

.

.

.

.

.

.

.

.

.

.

.

.

.

.

.

........

........

........

........

........

........

........

........

........

........

........

........

........

........

........

........

........

........

........

........

........

........

........

........

........

........

........

........

........

........

........

........

........

........*****************************************************************************************************************************************************************************************************************************************************************************************************************************************************************************************************************************************************************************************************************************

******************************************************************************************************************************************************************************************************************************************************************************************************************************************************************************************************************************************************************************************************************************************************************************************************************************************************************************* G (x) = 2 + ln 4 +

Z x

12

dt

t

= 2 + ln 4 +hln |t|

ix12

= 2 + 3 ln 2 + ln x .

Vous remarquez que l’assertion 6 a) estfausse à un signe près. Cas très délicat, qu’ilfaut bien contrôler.

D’autre part G(2) est la moitié de l’aire grisée A, ce qui est évident, donc l’assertion 6 b)est fausse.

La valeur de G (2) est 2+4 ln 2, ce qui permet de contrôler l’exactitude de l’assertion 4 c).Ce cas est fréquent dans les Questionnements Automatisables, il donne la possibilitéau candidat de vérifier les résultats, et élimine les candidats qui répondent au hasard ...dur ... dur, de tricher ...

Soit un dessin très parfait et nous voyons (cf figure 4.4) que la droite IJ est tangente à lacourbe G au point d’abscisse −1, soit nous remarquons que le point de tangence ne peutavoir lieu en x = 1. C’est encore le cas d’une assertion à moitié vraie et à moitié fausse.Cas très difficile pour ceux qui ne savent pas lire ... jusqu’au bout une phrase.

Ainsi l’assertion 6 c) est fausse.

La figure 4.4 permet de vérifier que l’assertion 6 d) est vraie, car le triangle IJK estisocèle, les segments [IJ ] et [JK] sont bien égaux, et de plus les deux triangles OIJ etOJK sont rectangles isocèles, donc ayant des angles aigus égaux à

π

4.

Ainsi l’assertion 6 d) est vraie.

Nous avons les réponses :

Question 06 : a) Fausse b) Fausse c) Fausse d) Vraie e) Fausse

Nous pouvons exprimer l’aire A0 hachurée sur la figure 4.4 de plusieurs manières, ce quin’écarte à priori aucune des propositions de la question 7. La plus naturelle consiste àconsidérer l’aire comprise entre la courbe G et la droite IJ pour x compris entre −2 et 0,soit

A0 =Z 0

−2[g(x)− x− 2] dx,

ce qui prouve que l’assertion 7 a) est fausse, mais ne permet pas de conclure pourl’assertion 7 b).

70 Questionnements Automatisables

........

........

........

........

........

........

........

........

........

........

........

........

........

........

........

........

........

........

........

........

........

........

........

........

........

........

........

........

........

........

........

........

........

........

........

........

........

........

........

........

........

........

........

........

........

........

........

........

........

........

........

........

...............................

..........................

y

........................................................................................................................................................................................................................................................................................................................................................................................................................................................................................................................................................................................................................................................................................................................................................................................................................ ..........................x

O−2I

J 2

2−1 −12

1

G..............................................................................................................................................

.....................

..

..

.

..

.

.

..

.

..

.

..

.

.

..

.

..

.

.

..

..

.

..

.

..

.

..

.

..

.

..

.

..

.

..

..

.

..

..

.

..

.

.

.

..

.

..

..

..

.

.

..

.

..

.

.

..

.

.

..

..

.

..

.

..

.

..

.........

.

.......................

............... ....

.

..

.

.

..

..

.

.

..

.

..

.

.

.

..

.

..

.

.

..

.

..

.

..

.

.

..

.

..

.

.

..

..

.

..

.

..

.

..

.

.

.

..

.

..

.

..

..

.

.

..

.

..

.

.

..

.

.

..

..

.

.

..

.............

........

........

........

........

........

........

........

........

........

........

........

........

........

........

........

........ ........ ........ ........ ........ ........ ........ ........ ........ ........

........

........

........

........

........

........

........

........

........

........

........

........

........

........

........

........

........

........

........

........

........

........

........

........

........

********************************************************************************************************************************************************************************************************************************************************************************************************************************************************************************************************************************************************************************************************************************************************************************************************************************************************************************************************************************************************************************************************************************************************************************* ***************************************************************************************************************************************************************************************************************************************************************************************************************************************************************************************************************************************************************************************************************************************************************************************************************************************************************************************************************************************

*******************************************************

******************************************************************************************************************************************************************************************************************************************************************************************************************************************************************************************************************************************************************************************************************************************************************************************************************************************************************************************************************************************************************************************************************************************************************************************************************************

Figure 4.4

L’intégrale précédente doit être partagée en deux intégrales car la fonction g est définiepar morceaux sur l’intervalle d’intégration [−2, 0] . Il est préférable, et plus rapide, deremarquer, sur la figure 4.4, la symétrie de l’aire A0 par rapport à la deuxième bissectriced’équation y = −x. Ce qui permet d’exprimer

A0 = 2

Z −1

−2[g(x)− x− 2] dx

= 2

Z −1

−2

·−1x− x− 2

¸dx

= 2

·− ln |x|− x2

2− 2x

¸−1−2

= −1 + 2 ln 2 .Ce qui prouve que les assertions 7 c) et 7 d) sont fausses, pour cette dernière à un signeprès.

Pour répondre à l’assertion 7 b), il est recommander de faire le calcul ...

2

Z 1

0

[g(x) + x− 1] dx = 2

Z 12

0

2 dx+ 2

Z 1

12

·1

x+ x− 1

¸dx

= 4hxi12

0+ 2

·ln x+

x2

2− x

¸112

=7

4+ 2 ln 2 .

Ce qui prouve que l’assertion 7 b) est fausse. ATTENTION : Il serait possible de donnerune valeur particulière à cette intégrale afin que l’assertion soit acceptée, piège délicieux.

Toutes les assertions de la question 7 étant fausses, nous devons accepter la case 7 e).

Nous avons les réponses :

Question 07 : a) Fausse b) Fausse c) Fausse d) Fausse e) Vraie

Corrigés 71

05

Cet exercice est très facile et il faut y prendre garde car dans les QuestionnementsAutomatisables les pénalités sont inversement proportionnelles aux difficultés.Dans cet exercice toute erreur coûte le maximum. Il faut être prudent et sûr de soi ...

La figure 5.1 précise les coordonnées des points A, B et C. Cela nous permet d’obtenirles coordonnées (ou composantes) des vecteurs

−→AB et

−−→BC.

−→AB =

½3− 0 = 32− 1 = 1

et−−→BC =

½4− 3 = 10− 2 = −2 ; (1)

Ce qui prouve que l’assertion 1 a) est fausse et que l’assertion 1 b) est vraie.

L’assertion 1 c) est fantaisiste, car les deux droites ne peuvent être parallèles que si etseulement si les deux vecteurs

−→AB et

−−→BC sont colinéaires, ce qui est faux d’après (1).

Le produit scalaire −→AB−−→BC = 3 ∗ 1 + 1 ∗ (−2) = 1

étant différent de zéro alors les deux droites D1 et D2 ne sont pas perpendiculaires.L’assertion 1 d) est donc fausse. Cette assertion comporte en soi une inexactitude, car lanullité du produit scalaire équivaut à l’orthogonalité, nous pouvions écarter cette assertionpar simple lecture. Les Questionnements Automatisables testent le raisonnementlogique.

........

........

........

........

........

........

........

........

........

........

........

........

........

........

........

........

........

........

........

........

........

........

........

........

........

........

........

........

........

........

........

........

........

........

........

........

........

........

........

........

........

........

........

........

........

........

........

........

........

........

........

........

........

........

........

........

........

........

........

........

..................................

..........................

y

........................................................................................................................................................................................................................................................................................................................................................................................................................................................................................................................................................................................................................................................................................................................................................................................................................................................................................................................................................ ..........................xO

E 3 4

1

2

D2D1

A

B

C

................................................

................................................

................................................

................................................

................................................

.................................................

................................................

................................................

................................................

................................................

................................................

................................................

................................................

.................................................

................................................

................................................

.

...............................................................................................................................................................................................................................................................................................................................................................................................................................................................................................................................................

.

.

.

.

.

.

.

.

.

.

.

.

.

.

.

.

.

.

.

.

.

.

.

.

.

.

.

.

.

.

.

.

.

.

.

.

.

.

.

.

.

.

.

.

.

.

.

.

.

.

.

.

.

.

.

.

.

.

.

.

.

.

.

.

.

.

.

.

.

.

.

.

.

.

.

.

.

.

.

.

.

.

.

.

.

.

.

.

.

.

.

.

.

.

.

.

.

.

.

.

.

.

.

.

.

.

.

.

.

.

.

.

.

.

.

.

.

.

.

.

.

.

.

.

.

.

.

.

.

.

.

.

.

.

.

.

.

.

.

.

.

.

.

.

.

.

.

.

.

.

.

.

.

.

.

.

.

.

.

.

.

.

.

.

.

.

.

.

.

.

.

.

.

.

.

.

.

.

.

.

.

.

.

.

.

.

.

.

.

.

.

.

.

.

.

.

.

.

.

.

.

.

.

.

.

.

.

.

.

.

.

.

.

.

.

.

.

.

.

.

.

.

.

.

.

.

.

.

.

.

.

.

.

.

.

.

.

.

.

.

.

.

.

.

.

.

.

.

.

.

.

.

.

.

.

.

.

.

.

.

.

.

.

.

.

.

.

.

.

.

.

.

.

.

.

.

.

.

.

.

.

.

.

.

.

.

.

.

.

.

.

.

.

.

.

.

.

.

.

.

.

.

.

.

.

.

.

.

.

.

.

.

.

.

.

.

.

.

.

.

.

.

.

.

.

.

.

.

.

.

.

.

.

.

.

.

.

.

.

.

.

.

.

.

.

.

.

.

.

.

.

.

.

.

.

.

.

.

.

.

.

.

.

.

.

.

.

.

.

.

.

.

.

.

.

.

.

.

.

.

.

.

.

.

.

.

.

.

.

.

.

.

.

.

.

.

.

.

.

.

.

.

.

.

.

.

.

.

.

.

.

.

.

.

.

.

.

.

.

.

.

.

.

.

.

.

.

.

.

.

.

.

.

.

.

.

.

.

.

.

.

.

.

.

.

.

.

.

.

.

.

.

.

.

.

.

.

.

.

.

.

.

.

.

.

.

.

.

.

.

.

.

.

.

.

.

.

.

.

.

.

.

.

.

.

.

.

.

.

.

.

.

.

.

.

.

.

.

.

.

.

.

.

.

.

.

.

.

.

.

.

.

.

.

.

.

.

.

.

.

.

.

.

.

.

.

.

.

.

.

.

.

.

.

.

.

.

.

.

.

.

.

.

.

.

.

.

.

.

.

.

.

.

.

.

.

.

.

.

.

.

.

.

.

.

.

.

.

.

.

.

.

.

.

.

.

.

.

.

.

.

.

.

.

.

.

.

.

.

.

.

.

.

.

.

.

.

.

.

.

.

.

.

.

.

.

.

.

.

.

.

.

.

.

.

.

.

.

.

.

.

.

.

.

.

.

.

.

.

.

.

.

.

.

.

.

.

.

.

.

.

.

.

.

.

.

.

.

.

.

.

.

.

.

.

.

.

.

.

.

.

.

.

.

.

.

.

.

.

.

.

.

.

.

.

.

.

.

.

.

.

.

.

.

.

.

.

.

.

.

.

.

.

.

.

.

.

.

.

.

.

.

.

.

.

.

.

.

.

.

.

.

.

.

.

.

.

.

.

.

.

.

.

.

.

.

.

.

.

.

.

.

.

.

.

.

.

.

.

.

.

.

.

.

.

.

.

.

.

.

.

.

.

.

.

.

.

.

.

.

.

.

.

.

.

.

.

.

.

.

.

.

.

.

.

.

.

.

.

.

.

.

.

.

.

.

.

.

.

.

.

.

.

.

.

.

.

.

.

.

.

.

.

.

.

.

.

.

.

.

.

.

.

.

.

.

.

.

.

.

.

.

.

.

.

.

.

.

.

.

.

.

.

.

.

.

.

.

.

.

.

.

.

.

.

.

.

.

.

.

.

.

.

.

.

.

.

.

.

.

.

.

.

.

.

.

.

.

.

.

.

.

.

.

.

.

.

.

.

.

.

.

.

.

.

.

.

.

.

.

.

.

.

.

.

.

.

.

.

.

.

.

.

.

.

.

.

.

.

.

.

.

.

.

.

.

.

.

.

.

.

.

.

.

.

.

.

.

.

.

.

.

.

.

.

.

.

.

.

.

.

.

.

.

.

.

.

.

.

.

.

.

.

.

.

.

.

.

.

.

.

.

.

.

.

.

.

.

.

.

.

.

.

.

.

.

.

.

.

.

.

.

.

.

.

.

.

.

.

.

.

.

.

.

.

.

.

.

.

.

.

.

.

.

.

.

.

.

.

.

.

.

.

.

.

.

.

.

.

.

.

.

.

.

.

.

.

.

.

.

.

.

.

.

.

.

.

.

.

.

.

.

.

.

.

.

.

.

.

.

.

.

.

.

.

.

.

.

.

.

.

.

.

.

.

.

.

.

.

.

.

.

.

.

.

.

.

.

.

.

.

.

.

.

.

.

.

.

.

.

.

.

.

.

.

.

.

.

.

.

.

.

.

.

.

.

.

.

.

.

.

.

.

.

.

.

.

.

.

.

.

.

.

.

.

.

.

.

.

.

.

.

.

.

.

.

.

.

.

.

.

.

.

.

.

.

.

.

.

.

.

.

.

.

.

.

.

.

.

.

.

.

.

.

.

.

.

.

.

.

.

.

.

.

.

.

.

.

.

.

.

.

.

.

.

.

.

.

.

.

.

.

.

.

.

.

.

.

.

.

.

.

.

.

.

.

.

.

.

.

.

.

.

.

.

.

.

.

.

.

.

.

.

.

.

.

.

.

.

.

.

.

.

.

.

.

.

.

.

.

.

.

.

.

.

.

.

.

.

.

.

.

.

.

.

.

.

.

.

.

.

.

.

.

.

.

.

.

.

.

.

.

.

.

.

.

.

.

.

.

.

.

.

.

.

.

.

.

.

.

.

.

.

.

.

.

.

.

.

.

.

.

.

.

.

.

.

.

.

.

.

.

.

.

.

.

.

.

.

.

.

.

.

.

.

.

.

.

.

.

.

.

.

.

.

.

.

.

.

.

.

.

.

.

.

.

.

.

.

.

.

.

.

.

.

.

.

.

.

.

.

.

.

.

.

.

.

.

.

.

.

.

.

.

.

.

.

.

.

.

.

.

.

.

.

.

.

.

.

.

.

.

.

.

.

.

.

.

.

.

.

.

.

.

.

.

.

.

.

.

.

.

.

.

.

.

.

.

.

.

.

.

.

.

.

.

.

.

.

.

.

.

.

.

.

.

.

.

.

.

.

.

.

.

.

.

.

.

.

.

.

.

.

.

.

.

.

.

.

.

.

.

.

.

.

.

.

.

.

.

.

.

.

.

.

.

.

.

.

.

.

.

.

.

.

.

.

.

.

.

.

.

.

.

.

.

.

.

.

.

.

.

.

.

.

.

.

.

.

.

.

.

.

.

.

.

.

.

.

.

.

.

.

.

.

.

.

.

.

.

.

.

.

.

.

.

.

.

.

.

.

.

.

.

.

.

.

.

.

.

.

.

.

.

.

.

.

.

.

.

.

.

.

.

.

.

.

.

.

.

.

.

.

.

.

.

.

.

.

.

.

.

.

.

.

.

.

.

.

.

.

.

.

.

.

.

.

.

.

.

.

.

.

.

.

.

.

.

.

.

.

.

.

.

.

.

.

.

.

.

.

.

.

.

.

.

.

.

.

.

.

.

.

.

.

.

.

.

.

.

.

.

.

.

.

.

.

.

.

.

.

.

.

.

.

.

.

.

.

.

.

.

.

.

.

.

.

.

.

.

.

.

.

.

.

.

.

.

.

.

.

.

.

.

.

.

.

.

.

.

.

.

.

.

.

.

.

.

.

.

.

.

.

.

.

.

.

.

.

.

.

.

.

.

.

.

.

.

.

.

.

.

.

.

.

.

.

.

.

.

.

.

.

.

.

.

.

.

.

.

.

.

.

.

.

.

.

.

.

.

.

.

.

.

.

.

.

.

.

.

.

.

.

.

.

.

.

.

.

.

.

.

.

.

.

.

.

.

.

.

.

.

.

.

.

.

.

.

.

.

.

.

.

.

.

.

.

.

.

.

.

.

.

.

.

.

.

.

.

.

.

.

.

.

.

.

.

.

.

.

.

.

.

.

.

.

.

.

.

.

.

.

.

.

.

.

.

.

.

.

.

.

.

.

.

.

.

.

.

.

.

.

.

.

.

.

.

.

.

.

.

.

.

.

.

.

.

.

.

.

.

.

.

.

.

.

.

.

.

.

.

.

.

.

.

.

.

.

.

.

.

.

.

.

.

.

.

.

.

.

.

.

.

.

.

.

.

.

.

.

.

.

.

.

.

.

.

.

.

.

.

.

.

.

.

.

.

.

.

.

.

.

.

.

.

.

.

.

.

.

.

.

.

.

.

.

.

.

.

.

.

.

.

.

.

.

.

.

.

.

.

.

.

.

.

.

.

.

.

.

.

.

.

.

.

.

.

.

.

.

.

.

.

.

.

.

.

.

.

.

.

.

.

.

.

.

.

.

.

.

.

.

.

.

.

.

.

.

.

.

.

.

.

.

.

.

.

.

.

.

.

.

.

.

.

.

.

.

.

.

.

.

.

.

.

.

.

.

.

.

.

.

.

.

.

.

.

.

.

.

.

.

.

.

.

.

.

.

.

.

.

.

.

.

.

.

.

.

.

.

.

.

.

.

.

.

.

.

.

.

.

.

.

.

.

.

.

.

.

.

.

.

.

.

.

.

.

.

.

.

.

.

.

.

.

.

.

.

.

.

.

.

.

.

.

.

.

.

.

.

.

.

.

.

.

.

.

.

.

.

.

.

.

.

.

.

.

.

.

.

.

.

.

.

.

.

.

.

.

.

.

.

.

.

.

.

.

.

.

.

.

.

.

.

.

.

.

.

.

.

.

.

.

.

.

.

.

.

.

.

.

.

.

.

.

.

.

.

.

.

.

.

.

.

.

.

.

.

.

.

.

.

.

.

.

.

.

.

.

.

.

.

.

.

.

.

.

.

.

.

.

.

.

.

.

.

.

.

.

.

.

.

.

.

.

.

.

.

.

.

.

.

.

.

.

.

.

.

.

.

.

.

.

.

.

.

.

.

.

.

.

.

.

.

.

.

.

.

.

.

.

.

.

.

.

.

.

.

.

.

.

.

.

.

.

.

.

.

.

.

.

.

.

.

.

.

.

.

.

.

.

.

.

.

.

.

.

.

.

.

.

.

.

.

.

.

.

.

.

.

.

.

.

.

.

.

.

.

.

.

.

.

.

.

.

.

.

.

.

.

.

.

.

.

.

.

.

.

.

.

.

.

.

.

.

.

.

.

.

.

.

.

.

.

.

.

.

.

.

.

.

.

.

.

.

.

.

.

.

.

.

.

.

.

.

.

.

.

.

.

.

.

.

.

.

.

.

.

.

.

.

.

.

.

.

.

.

.

.

.

.

.

.

.

.

.

.

.

.

.

.

.

.

.

.

.

.

.

.

.

.

.

.

.

.

.

.

.

.

.

.

.

.

.

.

.

.

.

.

.

.

.

.

.

.

.

.

.

.

.

.

.

.

.

.

.

.

.

.

.

.

.

.

.

.

.

.

.

.

.

.

.

.

.

.

.

.

.

.

.

.

.

.

.

.

.

.

.

.

.

.

.

.

.

.

.

.

.

.

.

.

.

.

.

.

.

.

.

.

.

.

.

.

.

.

.

.

.

.

.

.

.

.

.

.

.

.

.

.

.

.

.

.

.

.

.

.

.

.

.

.

.

.

.

.

.

.

.

.

.

.

.

.

.

.

.

.

.

.

.

.

.

.

.

.

.

.

.

.

.

.

.

.

.

.

.

.

.

.

.

.

.

.

.

.

.

.

.

.

.

.

.

.

.

.

.

.

.

.

.

.

.

.

.

.

.

.

.

.

.

.

.

.

.

.

.

.

.

.

.

.

.

.

.

.

.

.

.

.

.

.

.

.

.

.

.

.

.

.

.

.

.

.

.

.

.

.

.

.

.

.

.

.

.

.

.

.

.

.

.

.

.

.

.

.

.

.

.

.

.

.

.

.

.

.

.

.

.

.

.

.

.

.

.

.

.

.

.

.

.

.

.

.

.

.

.

.

.

.

.

.

.

.

.

.

.

.

.

.

.

.

.

.

.

.

.

.

.

.

.

.

.

.

.

.

.

.

.

.

.

.

.

.

.

.

.

.

.

.

.

.

.

.

.

.

.

.

.

.

.

.

.

.

.

.

.

.

.

.

.

.

.

.

.

.

.

.

.

.

.

.

.

.

.

.

.

.

.

.

.

.

.

.

.

.

.

.

.

.

.

.

.

.

.

.

.

.

.

.

.

.

.

.

.

.

.

.

.

.

.

.

.

.

.

.

.

.

.

.

.

.

.

.

.

.

.

.

.

.

.

.

.

.

.

.

.

.

.

.

.

.

.

.

.

.

.

.

.

.

.

.

.

.

.

.

.

.

.

.

.

.

.

.

.

.

.

.

.

.

.

.

.

.

.

.

.

.

.

.

.

.

.

.

.

.

.

.

.

.

.

.

.

.

.

.

.

.

.

.

.

.

.

.

.

.

.

.

.

.

.

.

.

.

.

.

.

.

.

.

.

.

.

.

.

.

.

.

.

.

.

.

.

.

.

.

.

.

.

.

.

.

.

.

.

.

.

.

.

.

.

.

.

.

.

.

.

.

.

.

.

.

.

.

.

.

.

.

.

.

.

.

.

.

.

.

.

.

.

.

.

.

.

.

.

.

.

.

.

.

.

.

.

.

.

.

.

.

.

.

.

.

.

.

.

.

.

.

.

.

.

.

.

.

.

.

.

.

.

.

.

.

.

.

.

.

.

.

.

.

.

.

.

.

.

.

.

.

.

.

.

.

.

.

.

.

.

.

.

.

.

.

.

.

.

.

.

.

.

.

.

.

.

.

.

.

.

.

.

.

.

.

.

.

.

.

.

.

.

.

.

.

.

.

.

.

.

.

.

.

.

.

.

.

.

.

.

.

.

.

.

.

.

.

.

.

.

.

.

.

.

.

.

.

.

.

.

.

.

.

.

.

.

.

.

.

.

.

.

.

.

.

.

.

.

.

.

.

.

.

.

.

.

.

.

.

.

.

.

.

.

.

.

.

.

.

.

.

.

.

.

.

.

.

.

.

.

.

.

.

.

.

.

.

.

.

.

.

.

.

.

.

.

.

.

.

.

.

.

.

.

.

.

.

.

.

.

.

.

.

.

.

.

.

.

.

.

.

.

.

.

.

.

.

.

.

.

.

.

.

.

.

.

.

.

.

.

.

.

.

.

.

.

.

.

.

.

.

.

.

.

.

.

.

.

.

.

.

.

.

.

.

.

.

.

.

.

.

.

.

.

.

.

.

.

.

.

.

.

.

.

.

.

.

.

.

.

.

.

.

.

.

.

.

.

.

.

.

.

.

.

.

.

.

.

.

.

.

.

.

.

.

.

.

.

.

.

.

.

.

.

.

.

.

.

.

.

.

.

.

.

.

.

.

.

.

.

.

.

.

.

.

.

.

.

.

.

.

.

.

.

.

.

.

.

.

.

.

.

.

.

.

.

.

.

.

.

.

.

.

.

.

.

.

.

.

.

.

.

.

.

.

.

.

.

.

.

.

.

.

.

.

.

.

.

.

.

.

.

.

.

.

.

.

.

.

.

.

.

.

.

.

.

.

.

.

.

.

.

.

.

.

.

.

.

.

.

.

.

.

.

.

.

.

.

.

.

.

.

.

.

.

.

.

.

.

.

.

.

.

.

.

.

.

.

.

........

.....

....

.....

....

.....

....

.....

....

.....

....

.....

....

.....

....

.....

....

.....

....

.....

.....

.....

....

.....

....

.....

....

.....

....

.....

....

.....

....

.....

....

.....

....

.....

......................................................................................................................................................................................................................................................................

Figure 5.1

Nous avons les réponses :

Question 01 : a) Fausse b) Vraie c) Fausse d) Fausse e) Fausse

Le coefficient directeur d’une droite est indépendant des valeurs numériques des coor-données de ses points, car il est possible de choisir pour définir une droite des pointsparticuliers, d’abscisses positives ou négatives par exemple.

Il faut bien lire, et c’est toute la difficulté des deux assertions proposées.

Nous avons besoin à la question 3 de l’équation cartésienne de D1 qui est de la formey = a x+ b ,

72 Questionnements Automatisables

où a et b vérifient le système ½0 = 4 a + b2 = 3 a + b ,

soit a = −2 et b = 8.D1 : y = − 2x+ 8 .

Le coefficient directeur est a = −2, donc l’assertion 2 a) est fausse. L’assertion 2 b)propose un coefficient directeur négatif, ce qui est vrai, mais la suite n’implique pas cetteproposition, donc nous écartons l’assertion 2 b).

Une équation cartésienne de D2 est de la formey = a x+ b ,

où a et b vérifient le système ½2 = 3 a + b1 = 0 + b ,

soit b = 1 et a =1

3.

D2 : y =1

3x+ 1 ⇔ −x+ 3 (y − 1) = 0 . (2)

Nous vérifions en particulier que les coefficient directeurs des droites D1 et D2 ont unproduit non égal à −1, ce qui re-confirme que ces droites ne sont pas perpendiculaires.D’après la deuxième expression donnée en (2) pour une équation de D2, nous pouvonsrejeter l’assertion 2 c). Pour 2 d), il faut lire la totalité de ce qui est proposé, ce qui estcorrect, donc nous acceptons cette assertion.

Nous avons les réponses :

Question 02 : a) Fausse b) Fausse c) Fausse d) Vraie e) Fausse

L’aire hachurée est comprise entre les droites D2, D1 et l’axe des abscisses (cf figure 5.1),nous pouvons l’écrire, en unité d’aire, sous la forme

A =

Z 3

0

µ1

3x+ 1

¶dx+

Z 4

3

(− 2x+ 8) dx (3)

=

·x2

6+ x

¸30

+h− x2 + 8x

i43

=11

2.

Ce calcul peut être plus rapide, en calculant l’aire du trapèze OABE et du triangleEBC,soit

Aire (OABE) =2 + 1

23 =

9

2

Aire (EBC) =4− 32

2 = 1 .

Maintenant, il faut décoder les assertions proposées.

L’assertion 3 a) est vraie d’après l’expression donnée en (3) , faire attention au signe −,c’est souvent un piège. Nous rejetons ainsi l’assertion 3 b).

L’assertion 3 c) est fantaisiste car sa valeur est nulle.

Pour l’assertion 3 d), il est préférable de faire le calcul, sait-on jamais ce qui se passe dansla tête d’un examinateur...

Corrigés 73

1

3

Z 3

0

(x+ 3) + 1 =1

3

·x2

2+ 3x

¸30

+ 1

=11

2.

Vous pouvez constater le côté vicieux de cette assertion. Dur - dur, ... les Question-nements Automatisables, il faut tout examiner dans le détail.

Nous avons les réponses :

Question 03 : a) Vraie b) Fausse c) Fausse d) Vraie e) Fausse

06

Une lecture attentive des premières questions nous suggère de faire un figure très précise,sans quoi, nous aurons beaucoup de problèmes... Il est fréquent dans les problèmes degéométrie en QCM que la figure apporte des renseignements. Toutefois, nous allonstravailler de manière plus classique, afin que ceux qui préparent des sujets non QCMpuissent en tirer profit.

ATTENTION : Nous obtenons avec une figure seulement des conjectures et non descertitudes, cf figure 6.1.

Quelques constations, utiles pour la suite.

Les triangles ABC et ADB sont équilatéraux, donc leurs orthocentres I et J sontégalement : centre du cercle circonscrit, point de concours des médianes (centre de gravité)et point de concours des bissectrices (centre du cercle inscrit).

La longueur du segment AI est égale au2

3de la longueur d’une médiane, soit

AI =2

3

"AB

√3

2

#=

l√3.

Cette valeur est voisine de celle proposée à l’assertion 1 a), mais ... fausse. Donc l’assertion1 a) est à rejeter.

Nous avons

IJ = AI =l√3.

Ce qui prouve que l’assertion 1 d) est fausse.

Le quadrilatère AIBJ est un losange car les côtés sont tous égaux[1](cf ci-après), mais cen’est pas un carré. En effet l’angle dIAJ = 60.[1] Nous pouvons aussi dire que les diagonales sont perpendiculaires et se coupent en leur milieu.

74 Questionnements Automatisables

...............

...............

l...............................................................................................................................................................

.................

..........................

.................................................................................................................................................................

..........................

A B

C

D

F

E

I

J

K

•L

....................................................................................................................................................................................................................................................................................................................................................................................................................................................................................................................................................................................................................................................................................................................................................................................................................................................................................................................................................................................................................................................................................................................................................................................................................................................................................................................................................................................................................................................................................

.......................................................................................................................................................................................................................................................................................................................................................................................................................................................................................................................................................................................................................................................................................................................................................................................................................................................................................................................................................................................................................................................................................................................................................................................................................................................................................................................................................................................................................................................................

................................................................................................................................................................................................................................................................................................................................................................................................................................................................................................................................................................................................................................................................................................................................................................................................................................................................................................................................................................................................................................................................................................................................................................

....................................................................................................................................................................................................................................................................................................................................................................................................................................................................................................................................................................................................................................................................................................................................................................................................................................................................................................................................................................................................................................................

..............................

..............................

..............................

..............................

..............................

.....

.............................................................................................................................................................................

................................................................................................................................................................

.........................................................................................................................................................

................

............................................................................................................................................ ..................

.....................................................................................................................................................................

.. .... .... .... .... .... .... .... ....

..........................................................................................

............................................................................................... .... .... .... .... .... .... .... ....

.........................................................................................................................................................................................................................

Figure 6.1

Nous avons les réponses :

Question 01 : a) Fausse b) Vraie c) Fausse d) Fausse e) Fausse

Il est évident que les deux quadrilatères ADBC et AIBJ sont seulement des losanges(Diagonales non égales, AB = l et CD = l

√3 pour le losange ADBC, AB = l et

IJ =l√3pour le losange AIBJ) et non des carrés, donc leurs sommets ne sont pas

cocycliques.Les deux assertions 2 a) et 2 b) sont donc fausses.

Les diagonales CJ et AK du quadrilatère AJKC se coupent en leur milieu I, car I estle centre de gravité du triangle ABC et AI = IK par symétrie. Il en résulte que lequadrilatère AJKC est un rectangle. Ce ne peut être un carré car les longueurs des deux

côtés consécutifs AC = l et AJ =l√3ne sont pas égales, donc l’assertion 2 c) est fausse

et l’assertion 2 d) est vraie.

Nous avons les réponses :

Question 02 : a) Fausse b) Fausse c) Fausse d) Vraie e) Fausse

AJ = AI =l√3, vous remarquerez les questions liées. Si vous ne répondez pas la même

chose aux assertions 1 a) et 3 a), il faudra vous attendre à des pénalités maxi...C’est à la fois le bon et le mauvais côté des Questionnements Automatisables lesréponses au hasard sont exclues, sous peine de sanction.

Ainsi l’assertion 3 a) est fausse.

D’après ce qui précède IJ = IB = IK = JB, donc le triangle IJB est équilatéral.L’assertion 3 c) est donc fausse. De même, pour le losange IJBK, les diagonales (IB) et(JB) sont perpendiculaires, donc l’assertion 3 d) est fausse et BK = IJ, donc l’assertion

Corrigés 75

3 b) est vraie

Nous avons les réponses :

Question 03 : a) Fausse b) Vraie c) Fausse d) Fausse e) Fausse

Pour les deux questions suivantes, il n’est pas utile de déterminer de manière explicite fet g. Il suffit de déterminer chaque étape à la ...main...

Nous avonsf (A) = r r0 (A) = r (D) = B ,

ce qui prouve que f n’est pas l’identité et que A n’est pas un point invariant, ainsi lesassertions 4 a) et 4 b) sont fausses.

La relation précédente prouve qu’une partie des assertions 4 c) et 4 d) est vraie.Contrôlons les deuxièmes parties.

Nous avonsf (B) = r r0 (B) = r (B) = C ,

ce qui prouve que l’assertion 4 c) est fausse et que l’assertion 4 d) est vraie.

Nous avons les réponses :

Question 04 : a) Fausse b) Fausse c) Fausse d) Vraie e) Fausse

Nous opérons de manière analogue avec g.

Nous avonsg (B) = r0 r (B) = r0 (C) = A ,

ce qui prouve que toutes les assertions proposées à la question 5 sont fausses, nous devonsaccepter l’assertion 5 e). C’est une des règles de ce type de QCM, c’est l’origine du nomQuestionnements Automatisables.

Nous avons les réponses :

Question 05 : a) Fausse b) Fausse c) Fausse d) Fausse e) Vraie

f étant la composée de deux rotations dont la somme des angles est 2 θ =2π

3est donc

une rotation d’angle2π

3, donc les assertions 6 a) et 6 b) sont fausses. Il en est de même

de l’assertion 6 c), qui est de plus fantaisiste.

Nous pouvons accepter l’assertion 6 d), car nous savons que la composée de deux rotationsdont la somme des angles est un multiple de 2π est une translation, éventuellement devecteur nul.

Il n’est pas inutile de rechercher le centre de cette rotation, pour le plaisir certes, maisutile pour les questions 8 et 9, nous pouvons constater que le point I est invariant par fet ainsi

f = rotµI,2π

3

¶. (1)

Remarquer que l’assertion 6 a) est presque vraie ... faire attention à ce type d’assertion.

76 Questionnements Automatisables

Nous avons les réponses :

Question 06 : a) Fausse b) Fausse c) Fausse d) Vraie e) Fausse

L’application g étant une rotation d’angle2π

3est bijective, donc l’assertion 7 a) est fausse.

Or g−1 est une rotation d’angle opposé à l’angle de la rotation g, soit −2π3.

Nous pouvons constater que le point J est invariant par g.

Ainsi g est la rotation de centre J et g−1 la rotation de centre J et d’angle −2π3.

g = rotµJ,2π

3

¶et g−1 = rot

µJ, −2π

3

¶. (2)

Ce qui prouve que g−1 6= g et que l’assertion 7 c) est fausse.

Il faut être prudent lorsqu’on détermine l’inverse de la composée de plusieurs applications,il faut inverser l’ordre, comme le montre le diagramme ci-dessous

A A

D

....................................................................................................................................................................................................... ..........................

..........................................................................................................................................................................................

..........................

.............................................................................................................................................................................................................................................................................................................

AA

D

....................................................................................................................................................................................................... ..........................

........

........

........

........

........

........

........

........

........

........

........

........

........

........

........

........

........

........

........

..................................

..........................

...................................................................................................................................................................................................................................................................................

..........................

r r−1

r0 (r0)−1g g−1

g = r o r0 g−1 = (r0)−1 o r−1

Figure 6.2

donc l’assertion 7 d) est vraie et l’assertion 7 b) est fausse.

Nous avons les réponses :

Question 07 : a) Fausse b) Fausse c) Fausse d) Vraie e) Fausse

f et g−1 étant des rotations d’angles opposés alors f o g−1 est une translation car la sommedes angles est nulle.

Nous déterminons le vecteur translation à l’aide d’un point particulier.¡f o g−1

¢(J) = f o

£¡g−1¢(J)¤= f (J) = K .

Ce qui prouve que les assertions 8 a) et 8 b) sont fausses et que les assertions 8 c) et 8 d)sont vraies. Remarquer le libellé de l’assertion 8 d), pas évident si nous avons une lecturetrop hâtive,... dur - dur.

Nous avons les réponses :

Question 08 : a) Fausse b) Fausse c) Vraie d) Vraie e) Fausse

D’après (1) et (2) , la composée de f et de g est une rotation d’angle4π

3(somme des

angles des deux rotations). Donc les assertions 9 a) et 9 b) sont fausses.

Nous remarquerons que(g o f) (A) = g (B) = A ,

donc g o f = rot

µA,4π

3

¶.

Corrigés 77

A priori les applications f o g−1 et g−1 o f sont différentes, ne pas surtout dire qu’ellessont inverses l’une de l’autre.

Nous savons que g−1 o f est une translation, donc l’assertion 9 d) est fausse. Pour rejeteraussi l’assertion 9 c), il faut vérifier que le vecteur translation de g−1 o f n’est pas nul.¡

g−1 o f¢(I) =

¡g−1¢(I) = L ,

où L est le symétrique du point J par rapport à la droite (DB) .

Nous sommes dans cette question dans le cas ou toutes les assertions proposées sontfausses, nous devons accepter l’assertion 9 e). C’est la règle des QuestionnementsAutomatisables.

Nous avons les réponses :

Question 09 : a) Fausse b) Fausse c) Fausse d) Fausse e) Vraie

07

L’homothétie hom (P, k1) de centre P et de rapport k1 =PO

PAqui transforme A en O,

C en C 0 et B en B0 implique que les droites AC et OC 0 et les droites AB et OB0 sontparallèles. Ce qui permet de construire les points B0 et C 0, cf figure 7.1 où nous avonschoisi O intérieur au triangle ABC. Faire une figure avec O extérieur au triangle ABC etconstater que les propriétés sont conservées.

L’assertion 1 a) est donc vraie. Les autres assertions ne peuvent être vraies que pour desfigures particulières.

B

A

CP

Q

R

B0 C 0

O

..................................................................................................................................................................................................................................................................................................................................................................................................................................................................................................................................................................................................................................................................................................................................................................................................................................................................................................................................................................................................................................................................................................................................................................................................................................................................................................................................................................................................................................................................................................................................................................................................................

.......................................................................................

........................................................................................

.......................................................................................

.......................................................................................

.......................................................................................

.......................................................................................

......

............................................................................................................................................................................................................................................................................................................. ......................................

......................................

......................................

......................................

......................................

......................................

......................................

......................................

......................................

......................................

................................

............................................................................

..................................................

Figure 7.1

Par exemple, si P, Q et R sont les milieux respectifs des côtés BC, AC et AB alors O estle point de concours des médianes, appelé aussi centre de gravité. Les droites RP et ACsont parallèles ce qui exclu que RB0 soit parallèle à AC et de même pour QC 0 parallèle àAB et RQ parallèle à BC.

Il faut être prudent avec les assertions précédentes car une figure hâtive peut tout rendrefaux et induire en erreur de bonnes idées. C’est un piège de la géométrie exploité par lesQuestionnements Automatisables.

78 Questionnements Automatisables

Nous avons les réponses :

Question 01 : a) Vraie b) Fausse c) Fausse d) Fausse e) Fausse

L’homothétie hom (P, k1) permet d’écrire l’égalité des rapports

PO

PA= k1 =

PB0

PB=

PC 0

PC. (1)

Dans une suite de rapports égaux, nous pouvons obtenir un rapport encore égal parcombinaison linéaire de numérateurs et de dénominateurs [1] soit par exemple

k1 =PB0

PB=

PC 0

PC=−PB0 + PC 0

−PB + PC=

B0C 0

BC, soit X = B0C 0 . (2)

Ce qui prouve que l’assertion 2 a) est vraie.

Nous ne pouvions à priori éliminer les autres propositions car les segments BB0, BP etCC 0 sont colinéaires au segment BC. Seule l’assertion 2 d) est obligatoirement fausse, carle vecteur

−−→CC 0 est de sens contraire à ceux proposés par les trois autres assertions.

Nous avons les réponses :

Question 02 : a) Fausse b) Vraie c) Fausse d) Fausse e) Fausse

Les droites RB et OB0 étant parallèles, nous considérons l’homothétie hom (C, k2) de

centre C et de rapport k2 =CO

CRqui transforme R en O et B en B0.

Nous pouvons écrireRO

RC=

BB0

BCsoit Y = BB0 . (3)

Ces deux rapports sont de manière générale différents de k2. Nous obtenons la valeur durapport précédent en fonction de k2 en écrivant

RO

RC=

RC + CO

RC= 1 +

CO

RC= 1− k2 .

Nous en déduisons que toutes les assertions de question 03 sont fausses, nous devonschoisir l’assertion 3 e).

Nous avons les réponses :

Question 03 : a) Fausse b) Fausse c) Fausse d) Fausse e) Vraie

De manière analogue à la question précédente, nous considérons les droites parallèles QC

et OC 0 et l’homothétie hom (B, k3) de centre B et de rapport k3 =BO

BQqui transforme Q

en O et C en C 0.

Nous pouvons écrireQO

QB=

CC 0

CB=

C 0CBC

soit Z = C 0C . (4)

Ces deux rapports sont aussi de manière générale différents de k3. Nous obtenons la valeurdu rapport précédent en fonction de k3 en écrivant

QO

QB=

QB +BO

QB= 1 +

BO

QB= 1− k3 .

[1] ATTENTION : C’est la même combinaison linéaire pour le numérateur et le dénominateur.

Corrigés 79

Nous en déduisons que l’assertion 4 c) est vraie, les trois autres étant fausses.

Nous avons les réponses :

Question 04 : a) Fausse b) Fausse c) Vraie d) Fausse e) Fausse

Il résulte de (2) , (3) et (4) que

PO

PA+

RO

RC+

QO

QB=

X

BC+

Y

BC+

Z

BC= 1

car B0C 0 +BB0 + C 0C = BC.

Nous en déduisons que l’assertion 5 c) est vraie, les trois autres étant fausses.

Nous avons les réponses :

Question 05 : a) Fausse b) Fausse c) Vraie d) Fausse e) Fausse

Remarque : Les assertions 5 b) et 5 d) sont obligatoirement fausses car les rapportsPO

PA,

RO

RCet

QO

QBsont positifs donc leur somme ne peut être nulle ou égale à −1. Il faut parfois

avec un peu de bon sens pour éliminer certaines assertions.

Si les trois rapportsPO

PA,RO

RCet

QO

QBsont égaux alors d’après la question 5 leur valeur

commune est1

3.

Nous déduisons de la relation (1) que

PO

PA=1

3=

PB0

PB=

PC 0

PC.

Ce qui exprime P est le milieu du segment [B0C 0] donc le milieu du segment [BC] .

Ainsi la droite AP est la médiane du triangle ABC relative au côté BC et O est le centrede gravité[1] (ou point de concours des médianes).

Ce qui prouve que l’assertion 6 a) est vraie, les autres sont fausses car le centre du cerclecirconscrit au triangle ABC est le point de concours des médiatrices, le centre du cercleinscrit est le point de concours des bissectrices intérieures et l’orthocentre le point deconcours des hauteurs.

Nous avons les réponses :

Question 06 : a) Vraie b) Fausse c) Fausse d) Fausse e) Fausse

Dans le cas où les points A, B et C ne sont pas alignés, nous savons que tout point O duplan contenant ces trois points peut être considéré comme le barycentre des points A, Bet C affectés des coefficients α, β et γ vérifiant la condition supplémentaire α+β+γ = 1.Nous dirons que (α, β, γ) sont les coordonnées barycentriques du point O dans le repèrebarycentrique ABC.

Si M est un point quelconque du plan, nous pouvons écrire−−→MO = α

−−→MA+ β

−−→MB + γ

−−→MC . (5)

[1] Nous dirons, par abus de langage, centre de gravité, ce qui suppose que le triangle ABC est forméd’une plaque homogène de densité uniforme.

80 Questionnements Automatisables

Ainsi en remplaçant M par P dans (5) nous obtenons−→PO = α

−→PA+ β

−−→PB + γ

−→PC .

Ce que nous pouvons écrire³−→PO − α

−→PA´−³β−−→PB + γ

−→PC

´=−→0

et qui exprime que ( −→PO − α

−→PA =

−→0

β−−→PB + γ

−→PC =

−→0 .

(6)

Ce résultat est obtenu en remarquant qu’une somme de deux vecteurs non colinéaires nepeut être nulle que si et seulement si ces deux vecteurs sont nuls.

Il résulte de la première relation de (6) que l’assertion 7 a) est vraie et que les autresassertions de cette question sont fausses.

Nous avons les réponses :

Question 07 : a) Vraie b) Fausse c) Fausse d) Fausse e) Fausse

De manière analogue, si nous replaçons M par R dans (5) alors nous obtenons−→RO = α

−→RA+ β

−→RB + γ

−→RC .

Ce que nous pouvons écrire³−→RO − γ

−→RC

´−³β−→RB + α

−→RA´=−→0

et qui exprime que ( −→RO − γ

−→RC =

−→0

β−→RB + α

−→RA =

−→0 .

(7)

Il résulte de la première relation de (7) que l’assertion 8 b) est vraie et que les autresassertions de cette question sont fausses. Vous remarquerez le piège qui nous est tenduavec l’assertion 8 b), la bonne réponse étant γ.

Nous avons les réponses :

Question 08 : a) Fausse b) Vraie c) Fausse d) Fausse e) Fausse

Enfin, si nous remplaçons M par Q dans (5) alors nous obtenons−→QO = α

−→QA+ β

−−→QB + γ

−→QC .

Ce que nous pouvons écrire³−→QO − β

−−→QB

´−³α−→QA+ γ

−→QC

´=−→0

et qui exprime que ( −→QO − β

−−→QB =

−→0

α−→QA+ γ

−→QC =

−→0 .

(8)

Il résulte de la première relation de (8) que toutes les assertions de la question 9 sontfausses. Nous devons choisir l’assertion 9 e).

Le cas ou toutes les assertions proposées sont fausses est le plus délicat, car ondoute toujours... il faut être sûr de ses résultats, c’est cela les QuestionnementsAutomatisables.

Corrigés 81

Nous avons les réponses :

Question 09 : a) Fausse b) Fausse c) Fausse d) Fausse e) Vraie

La dernière question est culturelle. Dans le cas d’un doute, s’abstenir. En effet lesQuestionnements Automatisables ne sont pas une loterie gratuite (ou chaque joueurpeut gagner sans dépenser un sous), donc toute réponse fausse est pénalisée (Chercher lesloteries gratuites, les vraies, et ...tenez moi au courant).

Pour notre culture personnelle, la formule de la question 5PO

PA+

RO

RC+

QO

QB= 1 ,

est connu sous le nom de théorème de Céva.

Théorème de Céva : A, B et C étant trois points non alignés, P, Q et R distincts despoints A, B et C et appartenant respectivement aux droites (BC) , (CA) et (AB) .

Une condition nécessaire et suffisante pour que les droites (AP ) , (AQ) et (AR) soientconcourantes est que

PB

PC

QC

QA

RA

RB= −1 .

La preuve est obtenue à partir des deuxièmes relations des expressions (6) , (7) et (8) .

Nous avons les réponses :

Question 10 : a) Fausse b) Fausse c) Vraie d) Fausse e) Fausse

Compléments :

L’exercice précédant peut-être complété par le théorème de Ménélaüs.

Théorème de Ménélaüs : A, B et C étant trois points non alignés, P 0, Q et R distinctsdes points A, B et C et appartenant respectivement aux droites (BC) , (CA) et (AB) .

Une condition nécessaire et suffisante pour que les P 0, Q et R soient alignés est que

P 0BP 0C

QC

QA

RA

RB= 1 .

Les points P et P 0 vérifiant la relationPB

PC= −P

0BP 0C

,

sont dits conjugués harmoniques par rapport aux points B et C, cf figure 7.2

B

A

CP

Q

R

P 0

O............................................................................................................................................................................................................................................................................................................................................................................................................................................................................

.........................................................................................................................................................................................................................................................................................................................................................................................................................................................................................................................................................................................................................................................................................

..........................................................................................

.........................................................................................

..........................................................................................

.........................................................................................

.........

................................................................................................................................................................................................................... ......................................

......................................

......................................

......................................

......................................

......................................

......................................

..............................

............................................................................................................................................................................................................................................................................................................................................................................................................................................................................................................................................................................................................................................................................................................................................................................

........................................................................

........................................................................

........................................................................

........................................................................

.......................................................................

.......................................................................

...................................................

..................................

..................................

..................................

..................................

..................................

..................................

..................................

..................................

..................................

..................................

..................................

..................................

..................................

...............

Figure 7.2

82 Questionnements Automatisables

08

Le plus simple pour répondre aux assertionsposées à la question 1 est de construire avecsoin une figure, cf figure 8.1.

Ce qui est souvent le cas dans les premièresquestions de géométrie. Cela demandetoutefois d’avoir une bonne maîtrise desoutils élémentaires de dessin, car il ne peutêtre question de faire un tracé approximatif.

Il faut savoir juger avec rapidité et pré-cision, c’est tout l’art de la géométrie enQuestionnements Automatisables.Une excellente figure et un bon esprit cri-tique forment le couple idéal.

Il est évident alors que les assertions 1 a) et1 c) sont vraies, donc inutile de contrôler les

A B

CD

M

N

P

QO

.......................................................................................................................................................................................................................................................................................................................................................................................................................................................................................................................................................................

........

........

........

........

........

........

........

........

........

........

........

........

........

........

........

........

........

........

........

........

........

.....

............................................................................................................................................................................................................................................................................................................................................................................................................................................................................................................................................................................................................................................................................................................................................................................................................................................................................................................................................................................................................................................................

........

........

........

........

........

........

........

........

........

........

........

........

........

........

........

........

........

........

........

........

........

........

........

........

........ ........ ........ ........ ........ ........ ........ ........ ........ ........ ........ ........ ........ ........ ........ ........ ........ ........ ........ ........ ........ ........ ........ ........

..................................................................................................................................................................

.. ... ...

. ... .... ... .... ...

..................................................................................................................................................................

.. ... ...

. ... .... ... .... ...

..................................................................................................................................................................

.. ... ...

. ... .... ... .... ...

..................................................................................................................................................................

.. ... ...

. ... .... ... .... ...

Figure 8.1

deux autres, car dans ce type de QCM deux assertions au plus sont exactes. Ainsi lesassertions 1 b) et 1 d) sont à déclarer fausses. Contrôlons toutefois ces deux assertions.

Le carré ABCD est inscrit sur le carré MNPQ donc ne peut lui être égal, ce qui confirmeque l’assertion 1 d) est fausse et de même le cercle circonscrit au carré ABCD ne peutpasser par les points M, N, P et Q.

Nous répondons :

Question 01 : a) Vraie b) Fausse c) Vraie d) Fausse e) Fausse

Par hypothèse les triangles ABD et BCD sont équilatéraux ce qui implique queAB = BC = CD = DA = BD. Donc les diagonales AC et BD du quadrilatère ABCDne peuvent être égales, plus précisément AC =

√3 BD.

Ce qui prouve que l’assertion 2 a) est fausse. Pour cette question, une figure parfaiteinduit la bonne réponse à cette question, cf figure 8.2.

La figure est symétrique par rapport aux droites (BD) et (AC) . Ainsi en particulier lesdiagonales des deux quadrilatères ABCD et MNPQ sont concourantes en O. Il en étaitde même pour la question 1.

Les deux triangles AMB et ABO sont rectangles aux sommets respectifsM et O. Ainsi lesquatre points A, M, B et O sont sur un cercle centré au milieu de l’hypoténuse commune(AB) de ces deux triangles.

Ce qui prouve que l’assertion 2 b) est vraie.

Les angles inscrits \AMO et [ABO interceptent le même arc AO donc sont égaux ousupplémentaires. La figure nous indique plutôt l’égalité. Ainsi \AMO = [ABO = 60 cequi prouve que l’assertion 2 c) est vraie et l’assertion 2 d) est fausse. Ceci résulte aussi dufait que la valeur de l’angle \OAM est de 45 + 30 = 75, prouvant ainsi que le triangleOAM ne peut être équilatéral.

Corrigés 83

Nous avons les réponses :

Question 02 : a) Fausse b) Vraie c) Vraie d) Fausse e) Fausse

Les angles inscrits \AOM et \ABM interceptent le même arc AM donc sont égaux ousupplémentaires. La figure nous indique plutôt l’égalité. Ainsi \AOM = \ABM = 45. Etpar symétrie [AOQ = 45, ce qui prouve que les diagonales MP et NQ sont orthogonaleset de plus égales par symétrie.

Donc le quadrilatère MNPQ est un carré. L’assertion 3 a) est donc vraie et l’assertion 3d) fausse car le triangle OMQ est rectangle en O.

A

B

C

D

M N

PQ

O.................................................................................................................................................................................................................................................................................................................................................................................................................................................................................................................................................................................................................................................................................................................................................................................................................................................................................................................................................................................................................................................................................................................................................................

................................

...............................

................................

................................

................................

................................

................................

...............................

................................

................................

.......................................................................................................................................................................................................................................................................................................................................................................................................................................................................................................................................................................

..........................................................

..........................................................

..........................................................

.............................................................................................................................................................................................................................................................................................................................................................................................................................................................................................................................................................................................................................................

..........................................................

..........................................................

..........................................................

.....................................................................................................................................................................................................................................................................................................................................................................................................................................................................................................................................................................................................................................................................................................................................................................................................................................................................................

........

........

........

........

........

........

........

........

........

........

........

........

........

........

........

........

........

........

........

........

........

.....

...........................................................................................................................................................................................................................................................................................................................................................................................................................................................................................................................................................................................................................................................................................

.......................................

..................................................................................................................................................................................................................................................................................................................................................................................................................................................

....................................................................................................................................................................................................................................................................................................................................................................................................................................................................................................................................................................................................................................................................................................................................................................................................................................................................

.....................................................

........................................

.....................................................................................................................................................

........................................

..................................................................................................................................................................

..........................

..................................................................................................................................................................

...........................

..................................................................................................................................................................

...........................

Figure 8.2

Si les droites (AB) et (NQ) étaient parallèles alors d’après ce qui précède les deux droites(AB) et (MO) seraient perpendiculaires et ainsi le quadrilatère AMBO serait un carré(losange ayant deux angles droits). Ce qui est faux. Donc l’assertion 3 b) est fausse.

Il est clair que les points A, M et Q ne peuvent être alignés car cette hypothèse exigeraitque l’angle\OAM soit droit, or sa valeur calculée précédemment est de 75.Ainsi l’assertion3 c) est fausse.

Nous avons les réponses :

Question 03 : a) Vraie b) Fausse c) Fausse d) Fausse e) Fausse

Nous savons que le produit de l’homothétie de centre Ω et de rapport k =1√2, que nous

notons homµΩ,

1√2

¶, et de la rotation de centre Ω et d’angle θ =

π

4, que nous notons

rot³Ω,

π

4

´, est commutatif. En clair nous avons

ϕ = hom

µΩ,

1√2

¶ rot

³Ω,

π

4

´= rot

³Ω,

π

4

´ hom

µΩ,

1√2

¶.

ATTENTION : Le produit précédent est commutatif car les deux transformations ontmême centre. Il faut être prudent et vérifier la commutativité, qui nous le savons n’estpas à priori vraie... Prudence ...

84 Questionnements Automatisables

Toutefois [1] nous allons le vérifier.

Si ω est l’affixe du point Ω alors l’homothétie de centre Ω et de rapport k =1√2est définie

par la relation vectorielle −−→ΩM 0 = k

−−→ΩM .

Dans C, au point M d’affixe z nous associons le point M 0 d’affixe z0

z0 − ω = k (z − ω) . (1)

De même, la rotation de centre Ω et d’angle θ =π

4est définie par les relations

ΩM 0 = ΩM et³−−→ΩM ,

−−→ΩM 0

´= θ .

Dans C, au point M d’affixe z nous associons le point M 0 d’affixe z0

z0 − ω = ei θ (z − ω) . (2)

Si nous composons (1) et (2) , c’est-à-dire rot³Ω,

π

4

´ hom

µΩ,

1√2

¶, nous obtenons

Z = ei θ ([k (z − ω) + ω]− ω) + ω

= k ei θ (z − ω) + ω. (3)

Ce qui est identique à la composition de (2) et de (1), à vérifier...sait-on jamais !!!

La relation (3) prouve que l’assertion 4 a) est vraie car nous avons

α = k ei θ et β = ω¡1− k ei θ

¢.

En utilisant les valeurs de k et de θ nous obtenons

α = k ei θ

=1√2ei π / 4

=1 + i

2.

et

β = ω¡1− k ei θ

¢= ω

1− i

2.

En conclusion la similitude ϕ est définie par

Z =1 + i

2z + ω

1− i

2. (4)

Ce qui prouve que l’assertion 4 c) est fausse et que l’assertion 4 d) est vraie.

L’assertion 4 b) est obligatoirement fausse, toutefois son contrôle exige quelques commen-taires ... ATTENTION : de telles assertions ne sont là que pour encourager les erreurs.

Si l’image de M par ϕ est B alors

ΩB =1√2ΩM et

³−−→ΩM ,

−→ΩB´=

π

4

et de même si l’image de C par ϕ est M alors

ΩM =1√2ΩC et

³−−→ΩC ,

−−→ΩM

´=

π

4.

[1] Ceci est un test de validité. Dans toute réalisation, la validation est le stade primordial.

Imaginez un avion non validé ....

C’est souvent la procédure de plus grand coût....

Corrigés 85

Ce qui implique

ΩB =ΩC

2et

³−−→ΩC ,

−→ΩB´=³−−→ΩC ,

−−→ΩM

´+³−−→ΩM ,

−→ΩB´=

π

2,

qui peut-être vrai car cela indique que Ω est sur le cercle de diamètre [CB] .Or pour l’instant Ω est un point quelconque. Donc cette assertion est peut-être variepeut-être fausse ... selon les circonstances ... donc fausse.

Nous avons les réponses :

Question 04 : a) Vraie b) Fausse c) Fausse d) Vraie e) Fausse

Nous représentons les points A, B, C, D, M, N, P et Q sur la figure ci-dessous.

.......................................................................................................................................................................................................................................................................................................................................................................................................................................................................................................................................................................................................................... ..........................x

........

........

........

........

........

........

........

........

........

........

........

........

........

........

........

........

........

........

........

........

........

........

........

........

........

........

........

........

........

........

........

........

........

........

........

........

........

........

........

........

........

........

........

........

........

........

........

........

........

........

........

........

........

........

........

........

........

........

........

........

........

........

........

........

........

........

........

........

........

........

........

..................................

..........................

y

A

B

C

D

M

N

P

Q

1

5

−3

7

−5

..............................................................................

..............................................................................

..............................................................................

..........................................................................................................................................................................................................................................................................................................................................................................................................

........................................................

........................................................

........................................................

........................................................

........................................................

.........................................................................................................................................................................................................................................................................................................................................................................................................................................................................................................................................................................................................................................................................................................................................................................................................................................................................................................................................................................................................................................................................................................................................................................................................................................................................................................

............................................................................................................................................................................................................................................................................................................................................................................................................................................................................... ........

........

........

........................................................................................................................................

........

........

........

........

........

........

........

........

........

........

........

........

........

........

........

........

................................................................................................................................................................................................

............................................................................................................................................................................................

............................................................................................................................................................................................

............................................................................................................................................................................................

..................................................................................................................................................................

.. ..

..... ....... ....... ...

Figure 8.3

Le point M est l’image par la similitude ϕ de centre A du point B, donc en utilisant (4)nous obtenons

zM =1 + i

2zB + zA

1− i

2

=1 + i

2(5 + i) + 0

1− i

2= 2 + 3 i. (5)

Ce qui prouve que l’assertion 5 a) est fausse.

De manière identique, le point N est l’image par la similitude ϕ de centre B du point C,donc en utilisant (4) nous obtenons

zN =1 + i

2zC + zB

1− i

2

=1 + i

2(7− 5 i) + (5 + i)

1− i

2= 9− i. (6)

Ce qui prouve que l’assertion 5 b) est fausse.

86 Questionnements Automatisables

De manière identique, le point P est l’image par la similitude ϕ de centre C du point D,donc en utilisant (4) nous obtenons

zP =1 + i

2zD + zC

1− i

2

=1 + i

2(−3 i) + (7− 5 i) 1− i

2

=5

2− 152i. (7)

Ce qui prouve que l’assertion 5 c) est fausse.

Enfin de manière identique, le point Q est l’image par la similitude ϕ de centre D dupoint A, donc en utilisant (4) nous obtenons

zQ =1 + i

2zA + zD

1− i

2

=1 + i

20 + (−3 i) 1− i

2

= −32− 32i. (8)

Ce qui prouve que l’assertion 5 d) est fausse, donc nous devons accepter la case e). C’esttrès dur dans ces cas là, car tout faux c’est contraire à la logique humaine, mais c’estcela les QCM, il faut en accepter les règles, qui ne sont par ailleurs que celles de la viequotidienne ...

Il est évident que l’observation de la figure 8.3 permet d’éliminer toutes les assertionsproposées et que les calculs précédents ne sont pas utiles.

Nous avons les réponses :

Question 05 : a) Fausse b) Fausse c) Fausse d) Fausse e) Vraie

D’après (5) et (7) une équation de la droite (MP ) est y = αx+ β avec α et β vérifiant

3 = 2α+ β

−152

=5

2α+ β ,

soit α = −21 et β = 45, ce qui donne pour une équation de (MP )

y = −21x+ 45. (9)

Ce qui prouve que l’assertion 6 a) est vraie et que l’assertion 6 b) est fausse.

Les deux autres assertions de cette question sont évidement fausse, ce qui est logiqued’après la figure. Un des avantages des QCM est que l’utilisation d’une figure peut donnerdes résultats, à condition toutefois que celle ci soit parfaite. Il faut utiliser une règle etun compas, sinon il y a des dégâts [1] ...Nous avons les réponses :

Question 06 : a) Vraie b) Fausse c) Fausse d) Fausse e) Fausse

Nous continuons, l’équation de la droite (NQ) est obtenue avec (6) et (8) .

[1] Note de l’auteur : Traduire ceci en pénalités ... c’est cher le QCM.

Corrigés 87

Une équation de la droite (NQ) est y = αx+ β [1] avec α et β vérifiant

−1 = 9α+ β

−32= −3

2α+ β

soit α =1

21et β = −10

7, ce qui donne pour une équation de (NQ)

y =1

21x− 10

7. (10)

Ce qui prouve que les assertions 7 a) et 7 b) sont fausses.

Contrairement à la question précédente où les assertions c) et d) étaient fantaisistes, ici,nous devons les examiner de près, même de très près ...

A priori l’assertion 7 c) est acceptable, en fait rien ne l’élimine. Donc nous la marquonsvraie.

Pour l’assertion 7 d), une équation de la droite (AB) est y =1

5x, ce qui permet de la

rejeter.

Nous avons les réponses :

Question 07 : a) Fausse b) Fausse c) Vraie d) Fausse e) Fausse

Une équation de la droite (AC) est

y = −57x.

Ce qui prouve que l’assertion 8 a) est fausse.

Les résultats (9) et (10) prouvent que les droites (MP ) et (NQ) sont perpendiculaires doncl’assertion 8 c) est vraie; par ailleurs leur intersection est le point O dont les coordonnéesvérifient le système

y = −21x+ 45

y =1

21x− 10

7.

Ce qui donne Oµ75

34, −4534

¶et prouve que le point O n’est pas situé sur la droite (AC) .

Ce n’est pas très éloigné, c’est ici que nous constatons qu’une bonne figure peut aider etqu’une mauvaise peut nuire.

Nous en déduisons que l’assertion 8 b) est fausse.

Nous pouvons, soit mesurer sur la figure les longueurs des segments [MP ] et [NQ] , soitcalculer à l’aide des relations (5) , (6) , (7) et (8) les valeurs de ces segments. Cette dernièreest plus longue et plus certaine. C’est dur de faire des mathématiques, mais c’est beau ...

Calculons la longueur du segment [MP ]

MP 2 =

µ5

2− 2¶2+

µ−152− 3¶2

=211

2

[1] L’auteur signale aux adeptes de la secte de ceux qui utilise une calculatrice sans réfléchir, que cescalculatrices sont aux normes américaines. En particulier les droites ont pour équation y = α + β x, cequi est différent de la notation française. Attention aux dégâts ...

88 Questionnements Automatisables

et celle du segment [NQ]

NQ2 =

µ−32− 9¶2+

µ−32+ 1

¶2=

211

2.

Ce qui nous permet de conclure que l’assertion 8 d) est vraie.

Nous avons les réponses :

Question 08 : a) Fausse b) Fausse c) Vraie d) Vraie e) Fausse

A l’aide d’un logiciel de calcul formel, par exemple la TI 92, la TI 89 ou sur ordinateuravec MAPLE, il est aisé de vérifier que les deux assertions 8 c) et 8 d) sont vraies pourtout quadrilatère ABCD convexe.

Nous l’avons vérifié avec MAPLE [1].

Les instructions avec MAPLE [2]sont les suivantes :> k := 1/sqrt(2) : theta := Pi/4 :

Le : à la fin permet de ne pas afficher le résultat à l’écran.

> Z := z -> k*exp(I*theta)*(z-omega)+omega ;

Z := k ei θ (z − ω) + ω

Nous supposons que les points A, B, C et D ont respectivement pour coordonnées(0, 0) , (a, b) , (c, d) et (0, e) .

Nous prenons Ω successivement en A, B, C et D pour obtenir les affixes des pointsM, N, P et Q.

> omega := 0 : M := evalc( Z(a+I*b) ) ;

M :=1

2a− 1

2b+ I

µ1

2a+

1

2b

¶> omega := a+I*b : N := evalc( Z(c+I*d) ) ;

N :=1

2a+

1

2b+

1

2c− 1

2d+ I

µ−12a+

1

2b+

1

2c+

1

2d

¶> omega := c+I*d : P := evalc( Z(I*e) ) ;

P :=1

2c+

1

2d− 1

2e+ I

µ−12c+

1

2d+

1

2e

¶> omega := I*e : Q := evalc( Z(0) ) ;

Q :=1

2e+

1

2I e

Ce qui permet de déterminer une équation du type y = αx + β de la droite (MP ) enrésolvant le système :

> equ1 := 1/2*b+1/2*a = (1/2*a-1/2*b)*alpha+beta :

[1] Il est possible d’obtenir actuellement une version Student de MAPLE V pour une somme raisonnable.

La version dite Education est dans un nombre important d’établissements scolaires.

Les logiciels formels seront la base de l’enseignement du 21 siècle.[2] MAPLE veut dire Erable en anglais, arbre symbolique du Canada.

Sur Internet :http://www.drummond.com/erable/erable/recettes.html

donne des recettes de gateaux au sirop d’érable. A consommer avec modération ...

Corrigés 89

> equ2 := 1/2*e+1/2*d-1/2*c =(1/2*c-1/2*e+1/2*d)*alpha+beta :

> solve( equ1, equ2, alpha, beta ) ;

α =a+ b+ c− d− e

a− b− c− d+ e, β =

−a c+ a e− b d

a− b− c− d+ e(11)

Et de manière analogue une équation du type y = αx+ β de la droite (NQ) en résolvantle système :

> equ1 := 1/2*d+1/2*b+1/2*c-1/2*a =(1/2*c+1/2*a-1/2*d+1/2*b)*alpha+beta :

> equ2 := 1/2*e = 1/2*e*alpha+beta :

> solve( equ1, equ2, alpha, beta ) ;

α =a− b− c− d+ e

−a− b− c+ d+ e, β =

(a− d) e

−a− b− c+ d+ e(12)

Ce qui permet de conclure dans le cas général que le produit des valeurs des deuxcoefficients directeurs α déterminés en (11) et (12) étant égal à −1, les deux droites(MP ) et (NQ) sont perpendiculaires.

Il en est de même pour les longueurs des segments [MP ] et [NQ] .

> evalc( abs(M-P)^2 )-evalc( abs(M-P)^2 ) ;

0

Vous remarquerez la puissance et la facilité d’utilisation de ce logiciel .... BRAVO...

09

Il est souvent utile de faire une figure correcte, elle permet de donner des indicationssur les réponses aux questions posées. ATTENTION : Une figure peut prouver qu’uneproposition est fausse, mais ne prouve pas qu’une proposition est vraie.

L’homothétie h de centre B et de rapport 2 transforme le segment [AM ] en segment[A0C] .

Ah7−→ h(A) = A0

Mh7−→ h(M) = C

Donc l’assertion 1 a) est vraie. Attention à l’assertion 1 b) qui est en partie fausse.En Questionnements Automatisables, en partie faux est entièrement faux.

Il est clair que les droites (AM) et (A0C) sont parallèles, donc les droites (AM) et (A0C 0)ne peuvent être parallèles, l’assertion 1 c) est donc fausse.

90 Questionnements Automatisables

A

B0

B

C

M

C 0

A0

C1×

..........................................................................................................................................................................................................................................................................................................................................................................................................................................................................................................................................

........

........

........

........

........

........

........

........

........

........

........

........

........

........

........

........

........

........

........

........

........

........

........

........

.........

....................................................................................................................................................................................................................................................................................................................................................................................................................................................................................................................................................................................................................................................................................................................................................................................................................................................................................................................................................................................................................................................................

.............................................................................................................................................................................................................................................................................................

...................................................................................... ........ ........ ........ ....

........ ................ ........

........ ................ ........

........ ................ ........

........ ................ ........

.

................................................................................................................................................................................................................................................................. ...................................................................................................................... ................................................................................................................

Figure 9.1

La figure 9.1 prouve que les droites (AM) et (A0C 0) ne peuvent être perpendiculaires (sauffigure particulière), donc l’assertion 1 d) est fausse.

Nous avons les réponses :

Question 01 : a) Vraie b) Fausse c) Fausse d) Fausse e) Fausse

La transformation s est la composée d’une homothétie et d’une rotation. C’est donc unesimilitude. Le seul problème que nous pouvons examiner (pour la culture personnelle, carcela n’est pas demandé) est de déterminer si l’ordre de composition est à respecter. Eneffet de manière générale le produit de deux applications n’est pas commutatif.

Pour cela examinons un point particulier.

s = r h : A h7−→ h(A) = A0 r7−→ r(A0) = r h (A) = B0 (1)

s0 = h r : A r7−→ r(A) = Ah7−→ h(A) = h r (A) = A0

En conclusion, d’après (1) , le produit n’est pas commutatif.

Examinons les images par s des points B et M.

s : Bh7−→ h(B) = B

r7−→ r(B) = C1

s : Mh7−→ h(M) = C

r7−→ r(C) = C 0 (2)

Le point C1 étant le symétrique du point B0 par rapport à A (cf figure 9.1)

Nous acceptons, d’après (1) et (2) , l’assertion 2 b) et nous rejetons l’assertion 2 a).

Il est clair sur la figure 9.1, que les droites (AM) et (B0C 0) ne sont pas parallèles, doncl’assertion 2 c) est fausse. Mais d’après (1) et (2) l’image du segment [AM ] est lesegment [B0C 0], ce qui prouve que les droites (AM) et (B0C 0) sont perpendiculaires et

que°°°−−→B0C 0

°°° = 2 °°°−−→AM°°° . Ainsi l’assertion 2 d) est vraie.Nous avons les réponses :

Question 02 : a) Fausse b) Vraie c) Fausse d) Vraie e) Fausse

Nous pouvons maintenant construire la figure dans le plan complexe, ce qui donnera desinformations sur les questions suivantes.

Corrigés 91

Nous avons immédiatement les affixes suivantes :

B0 (−i) , C 0 ¡i r ei ϕ¢[1] , M µ1 + r ei ϕ

2

¶et A0 (−1) . (3)

Ainsi toutes les assertions de la question 3 sont fausses et nous devons choisir la case e.C’est toujours le cas le plus délicat. Il est recommandé alors de re-contrôler ses résultats.

........

........

........

........

........

........

........

........

........

........

........

........

........

........

........

........

........

........

........

........

........

........

........

........

........

........

........

........

........

........

........

........

........

........

........

........

........

........

........

........

........

........

........

........

........

........

........

........

........

........

........

........

........

........

........

........

........

........

........

........

........

........

........

........

........

........

........

..................................

..........................iy

.......................................................................................................................................................................................................................................................................................................................................................................................................................................................................................................................................................................................... ..........................x

A

B0(−i)

B (1)

C (r ei ϕ)

M (1+r ei ϕ

2)

C 0(i r ei ϕ)

A0 (−1).................................................................................................................................................................................................................................................................................................................................................................................................................................................................................................................................................................................................................................................................................................................................................................................................................................................................................................................................................................................................................................................................................................................................................................................................................................................................................................................................................................................................................................................................................................................................................................................................................................................................................................................................................................................................................................................................................................................................................

............................................................................................................................................................................................... ........ ................ ........ ................ ........ ................ ........ ................ ........ ................ ........

................................................................................................................................................................................................................................................................................

Figure 9.2

Nous avons les réponses :

Question 03 : a) Fausse b) Fausse c) Fausse d) Fausse e) Vraie

D’après les affixes données en (3) , la mesure complexe deh−−→AM

iest

α = zM − zA =1 + r ei ϕ

2− 0 = 1 + r ei ϕ

2(4)

et celle deh−−→B0C 0

iest

β = zC0 − zB0 = i r ei ϕ + i = i¡1 + r ei ϕ

¢= 2 i α. (5)

Ce qui permet de dire que l’assertion 4 d) est vraie et que les trois autres sont fausses.

Nous avons les réponses :

Question 04 : a) Fausse b) Fausse c) Fausse d) Vraie e) Fausse

Nous déduisons de (5) que seule l’assertion 5 d) est vraie, les trois autres sont fausses.L’assertion 5 d) n’est pas nouvelle, en effet la même question a été posée au 2 d). Attentionà ceux qui répondent au hasard, sinon bonjour les pénalités ... Il est fréquent que lesexaminateurs contrôlent l’honnêteté des candidats. La vie peut réserver des surprises auxindélicats ...

[1] Nous pourrions aussi écrire i r ei ϕ = r ei ϕ+

π

2 .

92 Questionnements Automatisables

Nous avons les réponses :

Question 05 : a) Fausse b) Fausse c) Fausse d) Vraie e) Fausse

Morale de l’exercice : Faire une bonne, une très bonne, figure.

10

Cet exercice propose de déterminer les racines de l’équation du quatrième degré

z4 −√2 z3 − 4

√2 z − 16 = 0 , (1)

puis d’étudier la représentation géométrique de ces racines dans le plan complexe et enfinde contrôler les racines réelles (éventuelles).

Les deux assertions a) et b) de la question 1 sont évidemment fausses. En effet les solutionsde l’équation

z4 − 1 = 0 ,sont −1, +1, −i et +i, ce qui contredit ces deux assertions.Il ne faut jamais oublier qu’un exemple n’est qu’une contre preuve.

Si nous développons l’expression¡z2 + 4

¢ ¡z2 + α z + β

¢, (2)

nous obtenonsz4 + α z3 + (β + 4) z2 + 4α z + 4β ,

ce qui par identification avec l’équation (1) donne le système de quatre équations à deuxinconnues

α = −√2β + 4 = 0

4α = −4√24β = −16 .

Il faut déterminer α et β à l’aide de deux équations, par exemple les deux premières, puisvérifier pour les deux suivantes. ½

α = −√2β = −4 .

Ainsi l’expression (2) donne l’équation¡z2 + 4

¢ ³z2 −

√2 z − 4

´= 0 . (3)

Nous en déduisons que les deux assertions 1 c) et 1 d) sont fausses.

Nous sommes dans le cas ou toutes les assertions proposées sont fausses, cas difficile caril faut re-contrôler ses réponses. Que c’est dur - dur ... les QCM.

Nous avons les réponses :

Question 01 : a) Fausse b) Fausse c) Fausse d) Fausse e) Vraie

L’équation (3) donne immédiatement les racines de (1) .

Corrigés 93

Pour l’équation du second degré

z2 −√2 z − 4 = 0 ,le discriminant est

∆ =³√2´2+ 4 ∗ 4 = 18 =

³3√2´2

et prouve que ces racines sont réelles.

En conclusion, nous avons les quatre racines

2 i, −2 i, −√2 et 2

√2 .

Nous en déduisons que les assertions 2 b) et 2 d) sont vraies et que les autres sont fausses.

Nous avons les réponses :

Question 02 : a) Fausse b) Vraie c) Fausse d) Vraie e) Fausse

Nous représentons dans le plan complexe les points A, B, C et D d’affixes respectives2 i, −2 i, −√2 et 2√2.

........

........

........

........

........

........

........

........

........

........

........

........

........

........

........

........

........

........

........

........

........

........

........

........

........

........

........

........

........

........

........

........

........

........

........

........

........

........

........

........

........

........

........

........

........

........

........

........

........

........

........

........

........

........

........

........

........

........

........

........

........

........

........

.................................

..........................iy

................................................................................................................................................................................................................................................................................................................................................................................................................................................................................................................................................................... ..........................x

B (−2 i)

A (2 i)

C (−√2)D (2

√2)Ω (−√2/2)

(Γ)..................................................................................................................................................................................................................... *******************************************************************************************************************************************************************

************

**************

*****************

**********************

**********************************************************************************************************************************************************************************************************************************************************************************************************************************************************************************************************************************************************************************************************************************************************************************************************************

***********************

*****************************************************************************************************************************************************************************************************************

Figure 10.1

Cela nous permet de rejeter sans problème les assertions 3 a) et 3 d).

Nous savons que si les quatre points A, B, C et D sont cocycliques[1] alors le centre Ω ducercle Γ passant par ces quatre points est en particulier à l’intersection des médiatricesdes segments [AB] et [CD] , donc ne peut être que le milieu du segment [CD] .

Ω

Ã−√2 + 2√2

2=

√2

2, 0

!Le dessin ne prouvant rien, malgré de fortes présomptions, il faut par le calcul vérifier queΩA = ΩB = ΩC = ΩD.

En raison des symétries, il suffit de contrôler que

AΩ = CΩ,

ce qui n’est autre que le rayon du cercle Γ.

[1] Cela veut dire sur un même cercle.

94 Questionnements Automatisables

Pour AΩ nous utilisons le théorème de Pythagore

AΩ2 = OΩ2 +OA2 =

Ã√2

2

!2+ 22 =

9

2.

ATTENTION de ne pas faire (2 i)2 au lieu de 22.

CΩ2 =

Ã√2

2+√2

!2=9

2=

µ3√2

¶2.

Nous pouvons accepter les assertions 3 b) et 3 d).Nous avons les réponses :

Question 03 : a) Fausse b) Vraie c) Fausse d) Vraie e) Fausse

La fonction numériqueg(x) = x4 −

√2x3 − 4

√2x− 16 ,

est définie sur R, car c’est une fonction polynôme.Elle est continue et dérivable, une infinité de fois, en termes mathématiques nous disonsde classe C∞(dire de classe C infinie). Donc nous acceptons l’assertion 4 a).

La dérivée g0 de g estg0(x) = 4x3 − 3

√2x2 − 4

√2 .

Nous devons déterminer les zéros de cette expression du troisième degré. Nous essayonsquelques valeurs simples, sans résultat. Nous devons donc étudier la fonction g0.

Elle est dérivable et admet pour dérivée

g00(x) = 12x2 − 6√2x = 12x

Ãx−√2

2

!. (4)

Ce qui donne le tableau des variations de g0

......................................................................................................................................................................................................................................................................................................................................................................................................................................................

......................................................................................................................................................................................................................................................................................................................................................................................................................................................

........

........

........

........

........

........

........

........

........

........

........

........

........

........

........

........

........

........

........

........

........

........

........

........

........

........

........

........

.......

x −∞ 0√2/2 +∞

−∞

−4√2

−9√2/2

+∞g00

g0

+ +−0 0

.....................................................................................................................

...........................................................................................................................

.......................................................................................................................

qui prouve que g0 n’admet qu’une seule racine réelle supérieure à

√2

2, les deux autres

étant complexes. Il est possible d’obtenir avec une calculatrice une valeur approchée decette racine x0 = 1.6078.

Nous acceptons l’assertion 4 c).

Nous pourrions ne pas examiner les deux autres assertions, car une des règles de ce typede QCM, est au plus deux assertions exactes.

Toutefois, il n’en est pas ainsi pour tous les QCM, donc poursuivons notre examen.

Par définition, les points d’inflexion de C sont les points où il y a un changement deconcavité. Donc en particulier les points qui annulent g00 [2]. D’après l’expression (4) , iln’y a au plus que deux points d’inflexion. Nous rejetons l’assertion 4 b).

[2] g00 nulle est une condition nécessaire, mais pas suffisante. Examiner, par exemple, la fonctionh(x) = x4.

Corrigés 95

Nous avons le tableau des variations de g

......................................................................................................................................................................................................................................................................................................................................................................................................................................................

......................................................................................................................................................................................................................................................................................................................................................................................................................................................

........

........

........

........

........

........

........

........

........

........

........

........

........

........

........

........

........

........

........

........

........

........

........

........

........

........

........

........

.......

x −∞ 0 x0 +∞

+∞−16

g(x0)

+∞g0

g

− − +

0

0

................................................................................................ .....................

..................................................................................... .....................

.......................................................................................................................

Une valeur approchée de g (x0) est −24.290. Le tableau confirme que l’équation (1)n’admet que deux racines réelles. Attention dans les Questionnements Automati-sables les réponses sont croisées (ce qui veut dire liées entre elles).

Le tableau ci-dessus permet de rejeter l’assertion 4 d).

Nous avons les réponses :

Question 04 : a) Vraie b) Fausse c) Vraie d) Fausse e) Fausse

Nous pouvons résumer certains résultats en traçant la courbe représentative de g. Ilfaut remarquer que les points d’inflexion sont situés dans une zone peut lisible. C’estle propre de tous les nouveaux problèmes qui ne veulent pas privilégier les calculatricesperformantes.[3]

........

........

........

........

........

........

........

........

........

........

........

........

........

........

........

........

........

........

........

........

........

........

........

........

........

........

........

........

........

........

........

........

........

........

........

........

........

........

........

........

........

........

........

........

........

........

........

........

........

........

........

........

........

........

........

........

........

........

........

........

........

........

........

........

........

........

........

........

........

........

........

........

........

........

............................

..........................

y

................................................................................................................................................................................................................................................................................................................................................................................................................................................................................................................................................................................................................................................... ..........................xO−√2 2

√2•

••

•..........................

10.....................

1

................................................................................................................................

..........................

.....................................................................................................................................

..........................

Pts d’inflexion

*************************************************************************************************************************************************************************************************************************************************************************************************************************************************************************************************************************************************************************************************************************************************************************************************************************************************************************************************************************************************************************************************************************************************************************************************************************************************************************************************

Figure 10.2

[3] Note très personnelle qui n’engage que l’auteur : Une calculatrice trop performante aide celui quin’a pas besoin de l’être et est un handicap supplémentaire pour celui qui en aurait besoin. Ces outilspermettent de mieux comprendre, mais il faut avoir les bases mathématiques nécessaires et une maitriseparfaite de la calculatrice ... ce qui exige beaucoup de temps.

96 Questionnements Automatisables

11

Nous commençons par un peu de trigonométrie, il faut savoir que les candidats auxconcours QCM n’ont pas à leur disposition les formulaires données pour le baccalauréat.

Il faut apprendre (par coeur) quelques formules de base.

Ainsi, à partir des relations fondamentales

cos2 x+ sin2 x = 1

cos2 x− sin2 x = cos 2x

nous obtenons, par addition et par soustraction

1 + cos 2x = 2 cos2 x

1− cos 2x = 2 sin2 x

formules que nous retenons en remplaçant x parx

2.

1 + cosx = 2 cos2x

2(1)

1− cosx = 2 sin2x

2(2)

D’après (2), l’assertion 1 a) serait vraie si au lieu de sin x il y avait cosx, elle est doncfausse et d’après (1) l’assertion 1 b) est vraie.

Si dans (1) nous remplaçons x par x− π

2, nous obtenons

1 + cos³x− π

2

´= 2 cos2

³x2− π

4

´(3)

Ce qui prouve que l’assertion 1 c) est vraie car cos³x− π

2

´= sin x.

Il est en principe inutile de contrôler l’assertion 1 d). En effet une des règles de ce typede QCM est au plus deux assertions exactes, vérifions le toutefois.

D’après (2) et la proposition de l’assertion 1 d), nous devrions avoir

sin2x

2= cos2

³x2− π

4

´.

Ce qui confirme que l’assertion 1 d) est fausse. ATTENTION : l’égalité précédente estvraie si on remplace

π

4par

π

2.

Nous avons les réponses :

Question 01 : a) Fausse b) Vraie c) Vraie d) Fausse e) Fausse

Nous développons l’expression proposée

(1 + sin x)2 + cos2 x = 1 + 2 sin x+ sin2 x+ cos2 x

= 2 (1 + sin x)

Ce qui prouve que l’assertion 2 a) est fausse et que l’assertion 2 b) est vraie, et d’après(3) , l’assertion 2 c) est aussi vraie.

L’assertion 2 d) est donc fausse, ce qui est confirmé en faisant dans cette assertion x =π

2.

Corrigés 97

Nous avons les réponses :

Question 02 : a) Fausse b) Vraie c) Vraie d) Fausse e) Fausse

Il faut être très attentif au passage des inégalités strictes pour α compris entre 0 et π, auxpropositions des assertions 3 a) et 3 b) qui sont au sens large. Prudence, prudence.

Nous partons de 0 < α < π. Nous divisons par 2 (ce qui ne change pas le sens desinégalités) et nous retranchons aux trois termes

π

4(ce qui est toujours possible)

−π4<

α

2− π

4<

π

2− π

4=

π

4. (4)

Ce qui prouve que l’assertion 3 a) est fausse. Pour l’assertion 3 b), c’est évident qu’elleest fausse. Attention, ne jamais tomber dans de tels pièges, sinon bonjour les pénalités.

Ce qui précède nous a remis en mémoire quelques formules de trigonométrie, formules quidoivent nous servir pour les assertions suivantes.

Si nous transformons 1 + sin α en utilisant (3) , nous obtenons pour partie réelle de Z

2 cos2³α2− π

4

´. (5)

Ce qui nous invite à exprimer la partie imaginaire sous une forme analogue.

− cos α = sin³α− π

2

´= 2 sin

³α2− π

4

´cos

³α2− π

4

´(6)

où nous avons utilisé l’autre formule fondamentale

sin 2x = 2 sin x cos x .

Ainsi d’après (5) et (6) , nous pouvons écrire

Z = 2 cos³α2− π

4

´ hcos³α2− π

4

´+ i sin

³α2− π

4

´i.

D’après (4) , pour 0 < α < π nous avons −π4

2− π

4<

π

4, ce qui exprime que

cos³α2− π

4

´> 0, donc nous pouvons prendre ρ = 2 cos

³α2− π

4

´et θ =

α

2− π

4.

Ainsi les assertions 3 c), 4 a) et 4 c) sont vraies et les assertions 3 d), 4 b) et 4 d) fausses.

Nous avons les réponses :

Question 03 : a) Fausse b) Fausse c) Vraie d) Fausse e) Fausse

Question 04 : a) Vraie b) Fausse c) Vraie d) Fausse e) Fausse

12

L’équation du troisième degré

2 z3 − (1 + 2 i) z2 + (25 i− 1) z + 13 i = 0 (E)

98 Questionnements Automatisables

admet trois racines [1] dans l’ensemble C des nombres complexes (distinctes ou non).La première question nous donne des indications sur les racines éventuelles, car nous nesavons pas, encore, déterminer avec la technique de Cardan, les solutions exactes de cetteéquation.

A priori toutes les assertions de la question 1 sont plausibles. Il faut poursuivre plus loinnotre recherche. C’est souvent le cas dans les Questionnements Automatisables lesréponses ne sont “ éclairées ” que quelques questions plus loin... dur - dur.

Toutefois la racine α = −12, nous est proposée de manière trop voyante pour que cela

soit faux, nous prenons peut-être un risque, mais nous supposons que cela est vrai. Enprincipe dans les premières questions il n’y a pas de piège...mais sait-on jamais ?

Un simple calcul permet de contrôler que

2

µ−12

¶3− (1 + 2 i)

µ−12

¶2+ (25 i− 1)

µ−12

¶+ 13 i =

−14− (1 + 2 i) 1

4− (25 i− 1) 1

2+ 13 i = 0 ,

ce qui permet d’accepter l’assertion 1 b) et de rejeter l’assertion 1 a).

Si α, β et γ, vous avez lu tout le texte et découvert que les racines de (E) sont ainsiappelées, sont les racines de cette équation alors nous pouvons écrire que

(z − α) (z − β) (z − γ) = 0 ,

soit en développant

z3 − (α+ β + γ) z2 + (αβ + β γ + γ α) z − αβ γ = 0 .

Par identification, nous obtenons (ne pas oublier le coefficient 2 de z3 dans l’équation)

α+ β + γ =1

2+ i

α β γ = −132i ,

soit si nous prenons α = −12

β + γ = 1 + i et β γ = 13 i . (1)

Ce qui prouve que β et γ ont une partie imaginaire non nulle et donc que les assertions 1c) et 1 d) sont fausses.

Nous avons les réponses :

Question 01 : a) Fausse b) Vraie c) Fausse d) Vraie e) Fausse

Il nous est proposé d’écrire (E) sous la forme

2 (z − α)¡z2 + a z + b

¢= 0 ,

avec α = −12.

1ere méthode : β et γ sont les racines de l’équation z2 + a z + b = 0 donc β + γ = −a etβ γ = b. D’après la relation (1), nous avons

a = −1− i et b = 13 i .

[1] De manière générale, toute équation polynômiale de degré n > 1 admet exactement n racines dansC, distinctes ou non.

Corrigés 99

Nous allons développer, pour ceux qui n’auraient pas vu les astuces précédentes, deuxautres méthodes pour déterminer les valeurs de a et de b.

2eme méthode : Nous développons l’expression proposée

2 z3 + 2

µa+

1

2

¶z2 + 2

³b+

a

2

´z + b = 0

et nous identifions à (E) , ce qui nous donne le système2

µa+

1

2

¶= −1− 2 i

2³b+

a

2

´= −1 + 25 i

b = 13 i .

Nous obtenons de ce système, la même réponse que ci-dessus½a = −1− ib = 13 i .

3eme méthode : En effectuant la division euclidienne du polynôme

2 z3 − (1 + 2 i) z2 + (25 i− 1) z + 13 i ,

par z +1

2(Cette méthode est détaillée à l’exercice 22 du tome d’analyse)

2 z3 − (1 + 2 i) z2 + (−1 + 25 i) z + 13 i 2 z + 1−2 z3 − z2 z2

−2 (1 + i) z2 + (−1 + 25 i) z + 13 i − (1 + i) z2 (1 + i) z2 + (1 + i) z

26 i + 13 i 13 i−26 i − 13 i

0

et nous retrouvons encore le même résultat.

Nous en déduisons que toutes les assertion proposées à la question 02 sont fausses, nousdevons répondre vraie à l’assertion 2 e). De plus, seule l’assertion 3 d) est vraie, les autresétant fausses. Attention dans ces deux questions, il n’y a qu’une réponse vraie au plus.Si vous marquez deux réponses vraies, bonjour les pénalités.

Nous avons les réponses :

Question 02 : a) Fausse b) Fausse c) Fausse d) Fausse e) Vraie

Question 03 : a) Fausse b) Fausse c) Fausse d) Vraie e) Fausse

Le discriminant de l’équation

z2 − (1 + i) z + 13 i = 0 , (E’)

vaut ∆ = [− (1 + i)]2 − 4 × 13 i = −50 i. Il ne peut être que le carré d’un nombrecomplexe dont les parties imaginaire et réelle sont opposées, soit de la forme u (1− i),ceci afin d’éliminer en élevant au carré, le terme réel. Il reste alors 2u i = −50 i. Nouspouvons donc choisir l’assertion 4 b) qui propose ∆ = (−5 + 5 i)2 ou 4 c) qui propose∆ = (5− 5 i)2 . Les autres assertions proposées sont fausses. Attention, contrairement auxquestions précédentes, il y a ici deux réponses possibles...dur - dur lesQuestionnementsAutomatisables.

100 Questionnements Automatisables

Nous avons les réponses :

Question 04 : a) Fausse b) Vraie c) Vraie d) Fausse e) Fausse

Si nous notons δ = 5 − 5 i une des racines carrées du discriminant ∆ alors les solutionsde (E0) sont

1

2(−a+ δ) = 3− 2 i et 1

2(−a− δ) = −2 + 3 i .

Par hypothèse β a une partie réelle positive, d’où

β = 3− 2 i et γ = −2 + 3 i .Ainsi les deux assertions 5 a) et 5 d) sont fausses (à moitié faux, c’est faux, à moitiévrai, c’est faux). Il en est de même pour l’assertion 5 b). Contrôlons la proposition del’assertion 5 c)

3β + 2 γ = 3 (3− 2 i) + 2 (−2 + 3 i) = 5 ,ce qui prouve qu’elle est vraie.

Nous avons les réponses :

Question 05 : a) Fausse b) Fausse c) Vraie d) Fausse e) Fausse

Les racines α, β et γ sont distinctes, doncl’assertion 6 a) est fausse.Si nous représentons graphiquement lespoints A, B et C, alors il est clair que cespoints ne sont pas alignés donc que les as-sertions 6 b) et 6 c) sont fausses.Pour déterminer la nature du triangleABC,nous devons évaluer la longueur des côtésAC et AB.

AB2 =

µ−2 + 1

2

¶2+ 32 =

45

4

AC2 =

µ3 +

1

2

¶2+ (−2)2 = 65

4

et nous pouvons conclure que le triangleABC n’est pas isocèle.

........

........

........

........

........

........

........

........

........

........

........

........

........

........

........

........

........

........

........

........

........

........

........

........

........

........

........

........

........

........

........

........

........

........

........

........

........

........

........

........

........

........

........

........

........

........

........

........

........

........

........

........

...............................

..........................i y

............................................................................................................................................................................................................................................................................................................................................................................................................................................................................. ..........................xO

3 i

−2 i

−23

−12

A

B

C

........

........

........

........

........

........

........

........

........

........

........

........

........

........

........ ........ ........ ........ ........ ........ ........ ........ ........ ......

........ ........ ........ ........ ........ ........ ........ ........ ........ ........ ........ ........ ........ ........ ..............................................................................

********************************************************************************************************************************************************************************************************************************************************************************************************************************************************************************************************************************************************************************************************************************************************************************************************************************************************************************************************************************************************************************************************************************************************************************************************************************************************************************************************************************************************************************************************************************************************************************************************************************************************************************************************************************************************************************************************************************************************************************************************************************************************************************************************************************************************************************************************************************************************************************************************************************************************************************************************************************************************************************************************************************************************************************************************

Figure 12.1

Ainsi l’assertion 6 d) est fausse, ce qui entraîne que toutes les assertions proposées sontfausses, il faut accepter l’assertion e.

Nous avons les réponses :

Question 06 : a) Fausse b) Fausse c) Fausse d) Fausse e) Vraie

ATTENTION : Si vous avez fait une erreur dans le calcul des racines de (E) alors vouspourriez être dans le cas d’une réponse e à toutes les questions suivantes, ce qui, à priori,est passible de beaucoup de pénalités. Les Questionnements Automatisables laissentpeu de place à la fantaisie, il faut imaginer qu’en bicyclette on ne peut pas griller un stopdevant un camion ... sans prendre un risque.

Par définition, O est le barycentre des points A, B et C affectés des coefficients depondération respectifs 10, λ et µ, si nous avons

10−→OA+ λ

−−→OB + µ

−→OC =

−→0 et 10 + λ+ µ 6= 0 , (2)

Corrigés 101

ce qui s’exprime à l’aide de α, β et γ par

10α+ λβ + µγ = 0 ,

soit

10

µ−12

¶+ λ (3− 2 i) + µ (−2 + 3 i) = −5 + 3λ− 2µ+ (−2λ+ 3µ) i = 0

et donne la système ½3λ − 2µ = 5−2λ + 3µ = 0 .

Nous obtenons

λ = 3 et µ = 2 .

Les assertions 7 b) et 8 c) sont vraies et les six autres sont fausses.

Nous avons les réponses :

Question 07 : a) Fausse b) Vraie c) Fausse d) Fausse e) Fausse

Question 08 : a) Fausse b) Fausse c) Vraie d) Fausse e) Fausse

La relation (2) peut être écrite

10−→OA+ 3

−−→OB + 2

−→OC =

−→0 .

Ainsi O ne peut être le centre de gravité du triangle ABC, car cela exige que les coefficientsde pondérations soient égaux.

Pour les autres assertions, soit nous déterminons des coefficients de pondération [1] del’orthocentre, du centre du cercle circonscrit et du centre du cercle inscrit, soit ce qui estplus simple, nous utilisons la figure 12.1.

La première méthode n’est plus au programme des classes terminales depuis 30 ans, ce quiest regrettable car elle fait appel à des propriétés géométriques des points remarquablesd’un triangle , cf pages suivantes pour des compléments.

OA n’est pas perpendiculaire à BC, donc O ne peut être l’orthocentre. L’assertion 9 b)est donc fausse.

La longueur OA est très inférieure à celle de OB ou de OC, donc O ne peut être le centredu cercle circonscrit. L’assertion 9 c) est donc fausse.

Un mesure des angles montre que\BAO = 30 et que\OAC = 117 donc OA ne peut êtrela bissectrice de \BAC. L’assertion 9 d) est donc fausse.

[1] Nous rappelons que si α, β et γ sont des coefficients de pondération des points A, B et C alors lebarycentre G n’existe que si et seulement si α+ β + γ 6= 0 et par définition le tripletµ

α

α+ β + γ,

β

α+ β + γ,

γ

α+ β + γ

¶représente les coordonnées barycentriques de G par rapport aux points A, B et C.

102 Questionnements Automatisables

Nous avons les réponses :

Question 09 : a) Fausse b) Fausse c) Fausse d) Fausse e) Vraie

A titre de compléments, nous déterminons des coefficients de pondération des pointsremarquables suivants d’un triangle : centre de gravité G (point de concours desmédianes), orthocentre H (point de concours des hauteurs), centre du cercle circonscritO (point de concours des médiatrices) centre du cercle inscrit (point de concours desbissectrices intérieurs) I et centres des cercles exinscrits J, K et L (point de concoursd’une bissectrice intérieure et de deux bissectrices extérieures).

Approche du problème.

Dans ce qui suit nous notons a, b et c les longueurs respectives des côtés BC, CA et ABdu triangle ABC et bA, bB et bC les angles au sommet de ce triangle. Pour mémoire, nousrappelons que l’existence du triangle ABC exige en particulier que

|a− b| < c < a+ b : c’est la double inégalité triangulaire

et la relation bA+ bB + bC = π

M un point intérieur à un triangle ABC etA0 intersection des droites AM et BC (cffigure 12.2a), le point B1 (respectivementC1) projeté de B (respectivement C) sur ladroite AM.Le rapport des aires des triangles CMA etAMB est

Aire CMA

Aire AMB=

AM ∗ CC1 / 2AM ∗BB1 / 2 .

Les deux triangles CA0C1 et BA0B1 sont

homothétiques, donc les rapportsA0CA0B

et

CC1BB1

sont égaux.

A

B C

M

A0

C1

B1................................................................................................................................

..............................................................................................

..

................................

.........................................................................................................................................................................................................................................................................................................................................................................................................................................................................................................................................................................................................................................................................................................................................................................................................................................................................................................................................................................................................................................................................................................................................................................................................................................................................................................................................................................................................................................................................................................................................................................................................................................................

.....................................................................................................................................................................

..........................................................................................................................................................................................

........................................................

..........................

........ ........ ........ ........ ........ ........ ........ ........ ........ ........ ........ ........ ........ ........

Figure 12.2a

Nous en déduisonsAire CMA

Aire AMB=

CA0

BA0d’où l’égalité vectorielle

Aire CMA ∗ −−→BA0 +Aire AMB ∗ −−→CA0 = −→0exprimant que A0 est le barycentre des points B et C affectés des coefficients depondération respectifs : Aire CMA et Aire AMB.

Le théorème classique sur le barycentre partiel permet alors de conclure que M estle barycentre des points A, B et C affectés des coefficients de pondération respectifs :Aire BMC, Aire CMA et Aire AMB.

Aire BMC ∗ −−→MA+Aire CMA ∗ −−→MB +Aire AMB ∗ −−→MC =−→0 . (3)

Il est possible d’affecter aux aires précédentes un signe + ou − selon la position deM parrapport aux droites AB, BC et CA, et vérifier qu’alors la relation (3) est encore vraie.

Aire BMC ×−−→MA+Aire CMA×−−→MB +Aire AMB ×−−→MC =−→0 .

Par exemple, Aire BMC a un signe + si A etM sont dans le même demi-plan déterminépar la droite BC, sinon le signe est −.

Corrigés 103

Ainsi pour la figure 12.2b, nous avons

Aire BM1C = −Aire BM1C,

Aire CM1A = +Aire CM1A

et Aire AM1B = +Aire AM1B,

Aire BM2C = −Aire BM2C,

Aire CM2A = +Aire CM2A

et Aire AM2B = −Aire AM2B,

nous remarquons que dans tous les cas lasomme des aires algébriques est l’aire arith-métique du triangle ABC. Des coefficientsde pondération d’un point M quelconque

A

B C

M1

M2

.......................................................................................................................................................................................................................................................................................................................................................................................................................................................................................................................................................................................................................................................................................................................................................................................................................................................................................................

.....................................................................................................................................................................................................................................................................................................................................................................................................................................

Figure 12.2b

du plan du triangle ABC sont donc : M =¡Aire BMC, Aire CMA, Aire AMB

¢.

Dans la suite nous supposons M à l’intérieur du triangle ce qui implique que les anglesbA, bB et bC sont tous les trois aigus. Le cas où M est extérieur s’en déduira aisément encontrôlant les signes.

1er cas : G est le centre de gravité du triangle ABC.A0 étant le milieu du segment BC (cf figure 12.2c) alors les aires des triangles CGA etAGB sont égales, cela est bien connu. Des coefficients de pondération de G sont donc

G = (1, 1, 1) .

2eme cas : M est l’orthocentre H du triangle ABC.Si A1 est le pied de la hauteur relativeau sommet A alors les aires des trian-gles AHC et BHA sont respectivementAH ∗ CA1 / 2 et AH ∗ BA1 / 2. Nous avonscf figure 12.2c

CA1 = b cos bC et BA1 = c cos bB.Le rapport des aires peut être écrit

Aire CMA

Aire AMB=

CA1BA1

=

cos bCc

cos bBb

A

B C

O

A0A1

G

H

HA A0

..........................................................................................................................................................................................................................................................................................................................................................................................................................................................................................................................................................................................................................................................................................................................................................................................................................................................................................................................................................................................................................................................................................................................................................................................................................................................................................................................................................................................................................................................................................................................................................

.......................................................................................................................................................................................................................................................................................................................................................................................................................................................................

.......................................

.............................................................................................................................................................................................

..................................................................................................

..

......................................

..................................................................................................

..

......................................

........

........

........

........

........

........

........

........

........

........

........

........

........

........

........

........

........

........

........

........

........ ........ ........ ........ ........ ........ ........ ........ ........ ........ ........ ........ ........ ........ ........ ........ ................................................................................................................................................................................................................

........ ........ ........ ........ ........ ........ ........ ........ ........ ........ ........ ........ ........ ........ ........ ........ ........

..........................................................................................................................................................

........

........

........

........

........

........

........

....

**********************************************************************************************************************************************************************************************************************************************************************************

Figure 12.2c

Des coefficients de pondération de H sont donc

H =

Ãcos bAa

,cos bBb

,cos bCc

!.

3eme cas : O est le centre du cercle circonscrit du triangle ABC supposé non rectangle.

L’angle au centre \BOC et l’angle inscrit [BAC[2]interceptent le même arc BC, cf figure

12.2c, d’où\BOC = 2\BOA0 = 2 bA.

L’aire du triangle BOC est BC ∗ OA0 / 2 = aR cos bA / 2 où R est le rayon du cerclecirconscrit. Nous en déduisons par permutation circulaire le rapport

Aire COAAire AOB

=b cos bBc cos bC

[2] Par convention tous les angles sont orientés dans le sens trigonométrique.

104 Questionnements Automatisables

et donc que des coefficients de pondération de O sont

O =³a cos bA, b cos bB , c cos bC´ .

Remarquer que ce résultat est encore vrai si le triangle ABC est rectangle.

Droite d’Euler.

Les hauteurs HB et HC sont perpendiculaires aux côtés respectifs AC et AB donc lesangles \BHC et [BAC sont égaux. Il en résulte que si HA est le symétrique de H parrapport à la droite BC alors ce point est situé sur le cercle circonscrit au triangle ABCcar \BHAC = \BHC par hypothèse et \BHAC = [BAC car ils interceptent le même arc

BC.

Si A0 est le point diamétralement opposé à A alors A0 est le milieu du segment [A0H],cela résulte du fait que A0 est le centre d’une homothétie de rapport 2 qui transforme Oen A et donc A0 en H car les droites OA0 et AH sont parallèles.

En particulier nous avons−−→AH = 2

−−→OA0 =

−−→OB +

−→OC, d’où la relation fondamentale

−−→OH =

−→OA+

−−→OB +

−→OC

qui permet de conclure d’après la relation barycentrique 3−→OG =

−→OA+

−−→OB +

−→OC que

OH = 3−→OG

ce qui exprime, en particulier, que les trois points O,G et H sont alignés.

Si deux, au moins, des points O, G et H sont distincts alors la droite OGH est dite droited’Euler du triangle ABC, cf figure 12.2c.

4eme cas : I et J sont respectivement le centre inscrit et le centre exinscrit relatif à l’anglebA du triangle ABC, cf figure 12.2d.L’aire du triangle BIC est BC ∗ IA1 / 2soit a r / 2, où r est le rayon du cercleinscrit. Nous en déduisons par permutationcirculaire le rapport

Aire COAAire AOB

=b

c

et donc que des coefficients de pondérationde I sont I = (a, b, c) .

Pour le centre du cercle exinscrit J dansl’angle bA, il suffit de remarquer que lespoints A et J sont de part et d’autre de ladroite BC et qu’il suffit de changer le signedu coefficients de pondération a.Des coefficients de pondération de J [3] sont

A

B C

I

J

C1

C2

B1

B2

A1

A2

........................................................................................................................................................................................................................................................................................................................................................................................................................................................................................................................................................................................................................................................................................................................................................................................................................................................................................

............................................................................................................................................................................................................................................................................................................................................................................................................................................................................................................................

.............................................................................................................................................................................................................................................................................................................................................................................................................................................................................................................................................................................................................................................

....................................................................................................................................................................................................................................................................................................

...........................................................................................................................................................................

..............................................................................................................................................................................................................

........

........

........

........

........

..........................................................................................................................................................................................

................................

..................................................

...............................................................................................................................................................................................................................................................................................................................................................................................................

..................................................................................................

..................................................................................................

............................................................................. ....... ....... .......

............................................................................. ....... ....... .......

..........................................................................

..

...................... ..........................

........

................................................................

........

........

........

........

........ ........ ........

........ .

................................

........ ........ ........

........ ........ ........

........ ........ ........

........ ........ ........

........ ........ ........

........

........

........

........

........

........

........

........

........

........

........

........

........

........

........

........................................................................................................................

Figure 12.2dI = (−a, b, c)

et par permutation circulaire pour K et L

K = (a,− b, c) et L = (a, b, −c) .

[3] Remarquer que −a+ b+ c ne peut être nul que si les points A, B et C sont alignés, ce qui est exclu.

Corrigés 105

13

Une formule connu (ou à connaître) sin 2 θ = 2 cos θ sin θ permet d’écrire

z = 2 cos2 θ + i sin 2 θ

= 2 cos θ (cos θ + i sin θ) . (1)

Ce qui compte tenu de l’hypothèse −π2< θ <

π

2donne

|z| = 2 cos θ et arg z = θ,

ce qui prouve que les deux assertions 1 a) et 1 c) sont fausses.

D’après (1) , nous avons pour Z =1

z2

|Z| = 1

4 cos2 θet arg Z = −2 arg z = −2 θ,

ou encoreZ =

1

4 cos2 θ(cos 2 θ − i sin 2 θ) , (2)

ce qui prouve que l’assertion 1 b) est fausse car |z Z| = 1

2 cos θet que les assertions 1 d)

et 2 a) sont vraies.

Nous avons les réponses :

Question 01 : a) Fausse b) Fausse c) Fausse d) Vraie e) Fausse

Pour |θ| = π

3nous avons |z| = 1, donc l’assertion 2 b) est vraie. D’après la règle de ce

type de QCM, nous n’avons pas besoin d’examiner les deux autres assertions, car nousavons accepté deux assertions. Nous le contrôlons toutefois. Une erreur dans l’énoncé,sans conséquence car la correction peut accepter plus de deux réponses [1], est toujourspossible

|z|+ |Z| = 2 cos θ + 1

4 cos2 θ= 1

nous donne l’équation du troisième degré

8 cos3 θ − 4 cos2 θ + 1 = 0que nous ne savons pas résoudre car l’équation 8x3 − 4x2 + 1 = 0 n’admet pas de racinesimple évidente. Toutefois avec une calculatrice programmable nous pouvons résoudrecette équation et montrer que les solutions sont

x1 = −0.3774, x2 = 0.4387 + 0.3724 i et x3 = x2,

d’où il résulte que cos θ = −0.3774, ce qui donne une valeur de θ exclue par l’énoncé.Tout ceci est bien compliqué, il doit y avoir une solution plus simple. Réflexion ! !

L’hypothèse |z| + |Z| = 1 implique obligatoirement que |z| 6 1 et |Z| 6 1 ce qui est encontradiction avec l’énoncé qui nous donne |Z| |z|2 = 1. Ainsi l’assertion 2 c) est bienfausse.

Si θ = 0 alors z = 2 donc Z =1

4, ce qui confirme que l’assertion 2 d) est fausse.

[1] Dans le cas rarissime, où plus de deux réponses sont correctes, le logiciel de correction accepte toutesles combinaisons de deux réponses possibles, avec neutralisation possible des pénalités. Ceci peut résulterpar exemple d’une ambiguïté dans une question, ce qui est décelé par l’algorithme de correction.

106 Questionnements Automatisables

Nous avons les réponses :

Question 02 : a) Vraie b) Vraie c) Fausse d) Fausse e) Fausse

L’équation en ζ peut être écrite

ζ2 = z − 1= 2 cos2 θ + i sin 2 θ − 1= cos 2 θ + i sin 2 θ. (E)

Nous en déduisons que

|ζ|2 = 1 et arg ζ2 = 2 θ avec − π < 2 θ < π ,

soit pour solutions de (E)

|ζ| = 1 et arg ζ = θ ou arg ζ = θ + π .

Ce qui donne deux solutions distinctes, ce qui est attendu car (E) est une équation dusecond degré.

ζ1 = cos θ + i sin θ et ζ2 = −ζ1. (3)

Nous en déduisons que les assertions 3 a) et 3 d) sont fausses et que l’assertion 3 c) estvraie. Les solutions sont réelles si et seulement si θ = 0 et dans ce cas l’équation (E)admet une racine double 1, donc l’assertion 3 b) est vraie.

Nous avons les réponses :

Question 03 : a) Fausse b) Vraie c) Vraie d) Fausse e) Fausse

La relation ζ2 − 1 = 1

Zpeut s’écrire

ζ2 − 1 =½

z2 d’après l’énoncéz − 2 d’après (E) ,

ce qui n’est pas vérifié pour θ = 0 par exemple, donc l’assertion 4 a) est fausse.

D’après (3) nous avons

ζ1 − 1 = cos θ − 1 + i sin θ

= −2 sin2 θ/2 + 2 i sin θ/2 cos θ/2

= 2 sin θ/2 (− sin θ/2 + i cos θ/2)

et nous en déduisons que|ζ1 − 1| = 2 | sin θ/2|,

(le résultat est identique pour ζ2), ainsi l’assertion 4 b) est fausse.

D’après (E) et (2) nous avons

Z (z − 1) =1

4 cos2 θ(cos 2 θ − i sin 2 θ) (cos 2 θ + i sin 2 θ)

=1

4 cos2 θ= |Z|

donc l’assertion 4 c) est vraie.

D’après (E) nous avons1

ζ2= cos 2 θ − i sin 2 θ

Corrigés 107

et nous en déduisons en utilisant encore (E)

1

ζ2− (z − 1) = −2 i sin 2 θ

ce qui prouve que l’assertion 4 d) est fausse.

Cette question est longue à traiter car nous devons examiner chaque assertion, elle est àfaire en seconde lecture.

Nous avons les réponses :

Question 04 : a) Fausse b) Fausse c) Vraie d) Fausse e) Fausse

14

Nous représentons sur la figure 14.1 lespoints A et B d’affixes respectives 3 et 4 i,ce qui permet de contrôler certains calculset de visualiser certaines assertions, il fauttoujours prendre soin d’une figure.

Nous devons déterminer la partie réelle et lapartie imaginaire de ϕ (z) , pour cela nousposons z = x+ i y alors

ϕ (x+ i y) =x− 3 + i y

x+ i (y − 4) ,

d’où en multipliant et en divisant parl’expression conjuguée du dénominateur

x− i (y − 4) ,nous obtenons une expression réelle audénominateur

............................................................................................................................................................................................................................................................................................................................................................................................. ..........................x

........

........

........

........

........

........

........

........

........

........

........

........

........

........

........

........

........

........

........

........

........

........

........

........

........

........

........

........

........

........

........

........

........

........

........

........

........

........

........

........

........

........

........

........

........

........

........

........

........

........

........

........

...............................

..........................i y

O

A

3

B4 i

M

C

Ω•

...........................................................................................................................................................................................................................................................................................................................................................................................................................................................

.............................

.............................

.............................

.............................

.............................

.............................

.............................

.............................

.............................

.............................

.............................

.............................

...........................................................................................................................................................

.....................................................................................................................................................................................................................

..................................................................................................................................................................................................................................................................................................................................................................................................................................................................

................................

........................................................

.................................................................................................................................................................................................................................................................................................................................................................................................................................................................................................................................................................

............................................

......

........................................................................

Figure 14.1

ϕ (x+ i y) =(x− 3 + i y) ((x− i (y − 4)))(x+ i (y − 4)) (x− i (y − 4))

=x2 + y2 − 3x− 4 y

x2 + (y − 4)2 + i(4x+ 3 y − 12)x2 + (y − 4)2 . (1)

ϕ (z) est un imaginaire pur si et seulement si sa partie réelle est nulle.

Nous déduisons de (1) que C est l’ensemble des points vérifiant l’équationx2 + y2 − 3x− 4 y = 0

avec exclusion du point B.C est sur le cercle d’équation µ

x− 32

¶2+ (y − 2)2 = 13

4

dont le centre Ω a pour coordonnéesµ3

2, 2

¶et pour rayon

√13

2.

En particulier le cercle ci-dessus admet pour diamètre le segment [AB] .

108 Questionnements Automatisables

Nous en déduisons que les assertions 2 a) et 2 d) sont fausses. Attention aux propositionsdes assertions 2 b) et 2 c), il ne faut retenir que 2 c), l’assertion 2 b) est un piège quientraîne une pénalité élevée, elle sanctionne ceux qui ne savent pas lire.

Remarque : La droite AB privée du point B représente l’ensemble des points d’affixe ztels que ϕ (z) soit réel.

Ces résultats sont logiques dans la mesure où ϕ (z) n’est autre que le rapport des mesurescomplexes des vecteurs

−−→AM et

−−→BM. Un résultat bien connu exprime que le rapport des

mesures complexes de ces deux vecteurs est un nombre complexe dont le module est lerapport des longueurs des segments [AM ] et [BM ] et l’argument l’angle des vecteurs−−→AM et

−−→BM. Ainsi si ϕ (z) est réel alors les points A, M et B sont alignés et si ϕ (z)

est imaginaire pur alors l’angle \AMB est droit. Ceci est plus rapide pour répondre auxquestions posées. Encore une particularité des Questionnements Automatisables quiproposent une large éventail de possibilités. Il est moral de récompenser ceux qui ont plusde maturité mathématique.

Nous avons les réponses :

Question 02 : a) Fausse b) Fausse c) Vraie d) Fausse e) Fausse

Pour tout pointM de C, nous savons que l’angle \AMB est droit. Nous acceptons qu’il enest encore ainsi lorsque M est en A. Cela permet d’accepter l’assertion 1 b) et de rejeterl’assertion 1 a). Nous avons sur la figure 14.1 représenté un point M vérifiant l’assertion1 b).

Nous représentons sur la figure 14.2 lesymétriqueM du point O par rapport audiamètre AB. Ce qui prouve l’existence etl’unicité (ce qui n’est pas demandé) d’unpointM de C tel que les droites OM et ABsoient perpendiculaires. Nous acceptonsl’assertion 1 c).

En principe l’assertion 1 d) doit être fausse,c’est le principe de ce type de QCM, nouspouvons le vérifier rapidement en remar-quant qu’une condition nécessaire pour quele quadrilatère OAMB soit un carré est quele triangle OAB soit rectangle isocèle, cequi est faux.

............................................................................................................................................................................................................................................................................................................................................................................................. ..........................x

........

........

........

........

........

........

........

........

........

........

........

........

........

........

........

........

........

........

........

........

........

........

........

........

........

........

........

........

........

........

........

........

........

........

........

........

........

........

........

........

........

........

........

........

........

........

........

........

........

........

........

........

...............................

..........................i y

O

A

3

B4 i

M

C

Ω•

.............................................................................................................................................................................................................................................................................................................................................................................................................

.............................................................................................................................................................................................................................................................................................................................................................................................................................

.....................................................................................................................................................................................................................

...................................................................................................................................................................................................................................................................................................................................................................................................................................................................

.................................

..........................................................

.............................................................................................................................................................................................................................................................................................................................................................................................................................................................................................................................................................

..................................................

......

............

............

............

............

............

......

Figure 14.2

Nous avons les réponses :

Question 01 : a) Fausse b) Vraie c) Vraie d) Fausse e) Fausse

La troisième question peut être source d’une erreur, qualifié de grossière, si le candidat netient pas compte du fait que les Questionnements Automatisables n’ont de raisond’exister que s’ils décèlent les qualités scientifiques d’un candidat : raisonnement logique,rigueur dans l’enchaînement des questions et surtout du bon sens, celui des paysansdes siècles précédents qui n’ayant pas à leur disposition les prévisions de Météo-France,déduisaient de l’observation les dates de leurs travaux agricoles.

Le plus naturel, et là est l’erreur grossière, est de reprendre l’expression de ϕ (z) calculéeen (1) . C’est possible, mais long, pénible et pas facile.

Corrigés 109

Si nous reprenons au début alors l’équation

ϕ (z) = i

admet, immédiatement, une et une seule solution

ϕ (z) =

(i

z − 3z − 4 i

qui après réduction estα =

7

2+7

2i.

Il est clair quearg α =

π

4et |α| = 7√

2.

Nous en déduisons que l’assertion 3 a) est fausse et que l’assertion 3 b) est vraie.

De manière analogue, l’équationϕ (z) = −i

admet, immédiatement, une et une seule solution

ϕ (z) =

( −iz − 3z − 4 i

qui après réduction estβ = −1

2+1

2i.

Il est évident que l’assertion 3 d) est fausse, bien que très possible car α est solution deϕ (z) = i et β solution de ϕ (z) = −i. Pour l’assertion 3 c) nous devons faire le calcul,l’expression 4β4 + 1 donne bien 0 comme résultat, donc elle est vraie.

Nous avons les réponses :

Question 03 : a) Fausse b) Vraie c) Vraie d) Fausse e) Fausse

15

Le cercle C passe par le point O car l’angledIOJ est droit, donc l’assertion 1 a) estfausse et l’assertion 1 b) peut être acceptéecar C coupe l’axe Ox en deux points, cequi veut dire au moins un. Vous avez iciun contrôle de français, et oui, même enmathématiques ...La projection Ω du centre de C sur l’axe Oxadmet pour abscisse −1

4, donc l’assertion 1

c) est fausse.Le centre K de C a pour coordonnéesµ−14,1

2

¶et pour rayon R =

√5

4, ainsi son

équation cartésienne est

............................................................................................................................................................................................................................................................................................................................................................................................................................................................... ..........................x

........

........

........

........

........

........

........

........

........

........

........

........

........

........

........

........

........

........

........

........

........

........

........

........

........

........

........

........

........

........

........

........

........

........

........

........

........

........

........

........

........

........

........

........

........

........

........

........

........

........

........

........

...............................

..........................

y

O

J 1

−1/2I

P

α

Q

β

C

Ω

K

................................................................................................................................................................................................................................................................................................................................................

........

........

........

........

........

........

........

........

........

........

........

........

........

........

........

........

........

........

.......

........

........

........

........

........

........

........

........

........

........

........

........

........

........

........

........

........

........

........

........

........

........

........

........

........

........

........

........

........

........

........

........

........

........

........

........

........

........

........

........

........

........

........

........

........

........

........

........

........

........

........

........

.....

........

........

........

........

........

........

........

........

........

........

........

........

........

........

........

........

........

........

........

........

........

........

........

........

........

........

........

........

........

........

........

........

........

........

........

........

........

........

........

........

........

........

........

........

........

........

........

........

........

........

........

........

.....

...................................................................................................................................................................................................................................................................................................................................................................

...................................

........................................................................................................................................................................................................................................................................................................................................................................................................................................................................................................................................................................................................................................................................................

Figure 15.1µx+

1

4

¶2+

µy − 1

2

¶2=5

16,

110 Questionnements Automatisables

soit après réduction

x2 +1

2x+ y2 − y = 0. (3)

Ce qui n’est pas tout à fait l’expression proposée (1) , donc l’assertion 1 d) est fausse.Nous devons également rejeter l’assertion 2 a) pour la même raison. Il faut beaucoup deprudence dans la lecture des Questionnements Automatisables.

Nous avons les réponses :

Question 01 : a) Fausse b) Vraie c) Fausse d) Fausse e) Fausse

α et β sont les solutions du système obtenu avec (3)x2 +

1

2x+ y2 − y = 0

y =1

2

qui donne l’équation du second degré

x2 +1

2x+−1

4= 0. (4)

Le discriminant de (4) est ∆ =

µ1

2

¶2+ 1 =

5

4. Remarquer que ∆ = IJ2, ce qui est

attendu car PQ = IJ = 2R.

Nous en déduisons

α =−1 +√5

4et β =

−1−√54

. (5)

Ce qui permet de rejeter les assertions 2 c) et 2 d).

L’équation (4) nous permet d’avoir

α+ β = −12et αβ = −1

4.

Donc l’assertion 1 b) est fausse.

Toutes les assertions proposées à la question 2 sont fausses, nous devons alors marquer lacase e. Ce cas est toujours délicat. Il faut re-contrôler les calculs afin d’être sûr.

Nous avons les réponses :

Question 02 : a) Fausse b) Fausse c) Fausse d) Fausse e) Vraie

Une équation du quatrième degré dans C à coefficients réels peut être écrire sous la formez4 + a z3 + b z2 + c z + d = 0, (6)

avec a, b, c et d réels.

Il est immédiat qu’en prenant la conjuguée de cette équation

z4 + a z3 + b z2 + c z + d = 0,

si z est solution alors z est aussi solution. Ce qui veut dire que si une telle équationadmet une solution non réelle alors elle admet la solution conjuguée, en d’autres termesle nombre de solutions non réelles est pair.

Nous admettons, mais cela est vérifié sur des exemples, qu’une équation du quatrièmedegré dans C à coefficients complexes admet quatre racines distinctes ou non. Ceci résulted’un théorème appelé théorème fondamental de l’algèbre.

Examinons, maintenant, les différentes assertions de la question 3.

Corrigés 111

Pour a = b = c = 0 et d = 1 l’équation (6) devient z4 + 1 = 0 et admet pour solution [1]

par exemple

z =1 + i√2,

donc sa conjuguée aussi, d’après ce qui précède. L’assertion 3 a) est fausse.

Pour a = b = c = d = 0, l’équation (6) devient z4 = 0 et admet une solution quadruple 0.Les assertions 3 b) et 3 d) sont donc fausses.

Si d = 0 alors l’équation (6) devient z4 + a z3 + b z2 + c z = 0 et admet 0 pour racine, aumoins simple. Nous pouvons accepter l’assertion 3 d).

Nous avons les réponses :

Question 03 : a) Fausse b) Fausse c) Fausse d) Vraie e) Fausse

Si nous lisons le texte jusqu’au bout, ce qui est toujours recommandé, alors l’assertion 5a) nous propose de multiplier par z − 1 l’équation (2) , soit¡

z4 + z3 + z2 + z + 1¢(z − 1) = z5 − 1.

Remarque : Vous avez déjoué le piègegrossier de l’assertion 5 a). En effet z5k estégal à 1 et non à −1 ce qui prouve quel’assertion 5 a) est fausse.

Ainsi les quatre racines de (2) sont lesquatre racines non réelles de l’équationz5 − 1 = 0 soit

zk = e2 i k π/5 avec k = 1..4,

elles sont distinctes et non réelles doncl’assertion 4 a) est vraie et l’assertion 4 b)est fausse.Nous savons que les racines cinquième del’unité zk sont les affixes d’un pentagonerégulier dont une seule est réelle.

Si z1, z2, z3 et z4 sont les racines del’équation (2) alors nous avons en dévelop-pant la factorisation

.................................................................................................................................................................................................................................................................................................................................................................................................................................................................................................. ..........................x

........

........

........

........

........

........

........

........

........

........

........

........

........

........

........

........

........

........

........

........

........

........

........

........

........

........

........

........

........

........

........

........

........

........

........

........

........

........

........

........

........

........

........

........

........

........

........

........

........

........

.............................

..........................

y

O Pα

Q

β

A0 (z0)

A1 (z1)

A2 (z2)

A3 (z3)

A4 (z4)

..................................................................................................................................................................................................................................................................................................................................................................................................................................................................................................................................................................................................................................................................................................................................................................................................................................................................................................................................................................................................................................................................................................................................................................................................................................................................

..........................................

......................................................................................................................................................................................................................................................................................................................................................................................................................................................................................................................................................................................................................................................................................................................................................................................................................................................................................................

.........................................................................................................................................................................................................

Figure 15.2

(z − z1) (z − z2) (z − z3) (z − z4) = z4 − (z1 + z2 + z3 + z4) z3

+(z1z2 + z2z3 + z3z4 + z4z1) z2

− (z1z2z3 + z2z3z4 + z3z4z1 + z4z1z2) z + z1z2z3z4.

D’où par identification, nous obtenons

z1 + z2 + z3 + z4 = −1z1z2 + z2z3 + z3z4 + z4z1 = 1

z1z2z3 + z2z3z4 + z3z4z1 + z4z1z2 = −1z1z2z3z4 = 1

et nous en déduisons que l’assertion 4 c) est vraie et que l’assertion 4 d) est fausse.

Nous avons les réponses :

[1] Pour déterminer une solution de cette équation dite bicarrée, il suffit de poser Z = z2 et de résoudrealors Z2 +1 = 0, soit par exemple Z = i, puis de résoudre l’équation z2 = i en déterminant le module etl’argument d’une solution.

112 Questionnements Automatisables

Question 04 : a) Vraie b) Fausse c) Vraie d) Fausse e) Fausse

L’assertion 5 b) est fantaisiste car α et β sont réels alors que z2 et z4 sont non réels.

Remarque : La proposition z2 + z4 réel est aussi fausse (cf figure 15.2).

L’équation (2) est appelée équation réciproque car en divisant par z2, ce qui est possiblepuisque z = 0 n’en est pas solution, nous obtenons

z2 + z + 1 +1

z+1

z2= 0

et nous faisons apparaître Z = z +1

zet Z2 = z2 + 2 +

1

z2.

D’où le système équivalent à l’équation (2)(Z = z +

1

zZ2 + Z − 1 = 0

(7)

qui prouve que les assertions 5 c) et 5 d) sont fausses.

Toutes les assertions proposées à la question 5 sont fausses, nous devons alors marquer lacase e. Cas très délicat car nous avons déjà rencontré cette situation à la question 2.Dur - dur ... les Questionnements Automatisables.

Nous avons les réponses :

Question 05 : a) Fausse b) Fausse c) Fausse d) Fausse e) Vraie

Nous avons déterminé en (5) les valeurs exactes de α et de β, aussi nous pouvons avec

notre calculatrice calculer une valeur approchée de cos2 k π

5pour k = 1, 3, 4. Il faut être

prudent sur les résultats obtenus car valeur approchée n’est pas valeur exacte. Le risquetoutefois est faible, mais existe, les concepteurs desQuestionnements Automatisablessont parfois vicieux.

α =−1 +√5

2= 0.309 02 et β =

−1−√52

= −0.809 02

cos2π

5= 0.309 02, cos

5= −0.809 02 et cos

5= 0.309 02

Nous en déduisons que les assertions 6 a) et 6 b) sont vraies.

Nous pouvons résoudre le système (7) et déterminer la valeur exacte de cos2 k π

5et

sin2 k π

5pour k = 1.. 4 afin contrôler de manière exacte ce qui précède.

Le discriminant de Z2 + Z − 1 = 0 est ∆ = 5, d’où les solutions

Z1 =−1 +√5

2et Z2 =

−1−√52

.

Pour z +1

z= Z1, le discriminant de z2 − Z1 z + 1 = 0

∆ =

Ã−1 +√5

2

!2− 4 = −5 +

√5

2

est négatif, d’où les solutions sont complexes

z0 =−1 +√5

4+1

2

s5 +√5

2i et z00 =

−1 +√54

− 12

s5 +√5

2i.

Corrigés 113

Ce qui confirme que la partie réelle de z0 et de z00 est bien cos2π

5. Pour Z2, il suffit de

remplacer ci dessus√5 par −√5.

Il est en principe inutile de contrôler que les assertions 6 c) et 6 d) sont fausses, constatonsle toutefois sur la figure 15.2, c’est rapide.

Nous avons les réponses :

Question 06 : a) Vraie b) Vraie c) Fausse d) Fausse e) Fausse

Nous pouvons dorénavant construire un pentagone régulier, voir figure 15.3, avec une règleet un compas à partir de la figure 15.1.

A partir du cercle trigonométrique Γ, nous obtenons les point J et L.

Avec un compas, nous déterminons le milieu I du segment [OL] ; il suffit de construire lamédiatrice de ce segment.

Nous déterminons le milieu K du segment [IJ ] et nous traçons le cercle C de diamètre IJ.Les tangentes au cercle C coupent le cercle Γ aux sommets Ai, avec i = 1..4, du pentagone.

................................................................................................................................................................................................................................................................................................................................................................................................................................................................................................................................................................................................................................................... ..........................x

........

........

........

........

........

........

........

........

........

........

........

........

........

........

........

........

........

........

........

........

........

........

........

........

........

........

........

........

........

........

........

........

........

........

........

........

........

........

........

........

........

........

........

........

........

........

........

........

........

........

........

........

........

........

........

........

........

........

........

........

........

........

........

........

........

........

........

........

........

........

........

........

........

........

........

...........................

..........................

y

O................................................................................................................................................................................................................................................................

..................................

..................................................

..............................................................................................................................................................................................................................................................................................................................................................................................................................................................................................................................................................................................................................................................................................................................................................................................................................................................................................................................................................

.................................................................................................................................................................................................................................................................................................................

A0

A1

A2

A3

A4

J

L I

PQ

C

Γ

Ω

K

........................................................................................................................................................................................................................................................................................

........

........

........

........

........

........

........

........

........

........

........

........

........

........

........

......

........

........

........

........

........

........

........

........

........

........

........

........

........

........

........

........

........

........

........

........

........

........

........

........

........

........

........

........

........

........

........

........

........

........

........

........

........

........

........

........

........

........

........

........

........

........

........

........

........

........

........

........

........

........

........

........

........

........

........

........

........

........

........

........

........

........

........

........

........

........

........

........

........

........

........

.

........

........

........

........

........

........

........

........

........

........

........

........

........

........

........

........

........

........

........

........

........

........

........

........

........

........

........

........

........

........

........

........

........

........

........

........

........

........

........

........

........

........

........

........

........

........

........

........

........

........

........

........

........

........

........

........

........

........

........

........

........

........

........

........

........

........

........

........

........

........

........

........

........

........

........

.

.........................................................................................................................................................................................................................................................................................................

......................................

................................................................................................................................................................................................................................................................................................................................................................................................................................................................................................................................................................ ****

************************************************************************************************************************************************************************************************************************************************************************************************************************************************************************************************************************************************************************************************************************************************************************************************************************************************************************************************************************************************************************************************************************************************************************************************************************************************************************************************************************************************************************************************************************************************************************************************************************

Figure 15.3

114 Questionnements Automatisables

16

S’il existe un zéro réel pour le polynôme f alors α0 vérifie l’égalité

4α30 − 6 i√3α20 − 3

³3 + i

√3´α0 − 4 = 0 .

Nous pouvons séparer les parties réelles et imaginaires¡4α30 − 9α0 − 4

¢+ i

³−6√3α20 − 3

√3α0´= 0 .

Ce qui fournit le système ½4α30 − 9α0 − 4 = 0

−6√3α20 − 3√3α0 = 0 .

(1)

La seconde équation de (1) admet pour solution 0 et −12. Nous vérifions que 0 n’est pas

solution de la première équation de (1) mais que −12est solution de la première équation

de (1) .

Nous en déduisons que l’assertion 1 c) est vraie et que les autres sont fausses.

Nous avons les réponses :

Question 01 : a) Fausse b) Fausse c) Vraie d) Fausse e) Fausse

Nous pouvons déterminer de deux manières la factorisation proposée en préambule de laquestion 2 :

1ère méthode :

Nous développons l’expression

f(z) = 4

µz +

1

2

¶g (z) [1]

= 4

µz +

1

2

¶¡z2 +Az +B

¢= 4z3 + (4A+ 2) z2 + (2A+ 4B) z + 2B

et nous identifions. 4A+ 2 = −6 i√32A+ 4B = −3 ¡3 + i

√3¢

2B = −4 .Ce qui donne immédiatement

A = −1 + 3 i√3

2à l’aide de la première et

B = −2à l’aide de la troisième. Il ne faut pas oublier de vérifier ces valeurs trouvées dans laseconde. C’est un excellent moyen de contrôle. A ne pas négliger, sous peine de sanction.Les Questionnements Automatisables attribuent des points négatifs aux mauvaisesréponses. Si on perd au loto alors on perd son argent.

[1] Nous avons mis 4 en facteur dans f(z) afin de faire apparaître g(z).

Corrigés 115

2ème méthode :

−12étant un zéro de f, nous pouvons écrire

f(z) = 4 z3 − 6 i√3 z2 − 3

³3 + i

√3´z − 4

0 = 4

µ−12

¶3− 6 i

√3

µ−12

¶2− 3

³3 + i

√3´µ−12

¶− 4

soit en retranchant terme à terme

f(z) = 4

Ãz3 −

µ−12

¶3!− 6 i

√3

µz2 −

µ−12

¶2

¶− 3

³3 + i

√3´µ

z −µ−12

¶¶.

Les identités remarquables

a3 − b3 = (a− b)¡a2 + a b+ b2

¢a2 − b2 = (a− b) (a+ b)

nous permettent de mettre en facteur z+1

2dans f (z) et d’obtenir la factorisation demandée

f(z) = 4

µz +

1

2

¶Ãz2 − 1 + 3 i

√3

2z − 2

!.

Ce qui prouve que les assertions 2 c) et 3 b) sont vraies.

Les autres assertions de la question 3 sont fausses, pour celles de la question 2, nousdevons rendre réel le dénominateur de 2 b) et de 2 d).

Pour 2 b), nous obtenons

3√3− i

2 i=

¡3√3− i

¢(−i)

2=−1− 3 i√3

2,

ce qui prouve que cette assertion est exacte. Il est dans ce type de QCM inutile de contrôlerque 2 d) est fausse, car chaque question a au plus deux réponses exactes. Toutefois nousavons pour 2 d)

3√3 + i

2 i=

¡3√3 + i

¢(−i)

2=1− 3 i√3

2,

ce qui vérifie que cette assertion est fausse.

Nous avons les réponses :

Question 02 : a) Fausse b) Vraie c) Vraie d) Fausse e) Fausse

Question 03 : a) Fausse b) Vraie c) Fausse d) Fausse e) Fausse

Le discriminant de l’équation du second degré

z2 − 1 + 3 i√3

2z − 2 = 0

est

∆ =

Ã−1 + 3 i

√3

2

!2− 4 (−2)

=3 + 3 i

√3

2.

116 Questionnements Automatisables

La question 4 suggère que ∆ est le carré de l’une des quatre assertions qui sont proposées.Nous pouvons faire une vérification exhaustive, ce qui est long et fastidieux, ou biendéterminer a et b tels que

3 + 3 i√3

2= (a+ i b)2 . (2)

Développons le deuxième membre de (2)

3 + 3 i√3

2= a2 − b2 + 2 i a b.

et identifions les parties réelles et imaginairesa2 − b2 =

3

2

2 a b =3√3

2.

(3)

Il est souhaitable d’associer à (3) la relation des modules vérifiée par (2)¯¯3 + 3 i

√3

2

¯¯ = ¯(a+ i b)2

¯soit

a2 + b2 = 9.

Ce qui nous donne deux solutionsa1 =

3

2

b1 =

√3

2

et

a2 = −3

2

b2 = −√3

2.

Ce qui prouve que toutes les assertions proposées à la question 4 sont fausses.

Nous devons choisir la réponse 4 e).

Nous répondons:

Question 04 : a) Fausse b) Fausse c) Fausse d) Fausse e) Vraie

Nous pouvons écrire que3 + i

√3

2est une racine carrée de ∆.

Ainsi les deux zéros du polynôme g(z) sont

1 + 3 i√3

2+3 + i

√3

22

= 1 + i√3

et1 + 3 i

√3

2− 3 + i

√3

22

=−1 + i

√3

2.

Le texte propose que <e α1 < 0 donc

α1 =−1 + i

√3

2

α2 = 1 + i√3 .

Ce qui prouve que les assertions 5 c) et 6 d) sont vraies et que les autres assertions desquestions 5 et 6 sont fausses.

Corrigés 117

Nous répondons :

Question 05 : a) Fausse b) Fausse c) Vraie d) Fausse e) Fausse

Question 06 : a) Fausse b) Fausse c) Fausse d) Vraie e) Fausse

Si α0, α1 et α2 sont trois éléments consécutifs d’une suite géométrique alors les rapportsα1α0et

α2α1sont égaux à la raison q.

Ce qui est confirmé, car nous avons d’une part

α1α0=

−1 + i√3

2

−12

= 1− i√3

et d’autre partα2α1

=1 + i

√3

−1 + i√3

2

=³1 + i

√3´Ã−1− i

√3

2

!= 1− i

√3 .

En conclusion, la raison de cette suite géométrique est

q = 1− i√3 .

Ce qui prouve que l’assertion 7 d) est vraie et que les autres assertions de la question 7sont fausses.

Nous répondons :

Question 07 : a) Fausse b) Fausse c) Fausse d) Vraie e) Fausse

Nous en déduisons que le terme général de cette suite géométrique, dont le premier termeest α0, peut être écrit sous la forme

αn = α0³1− i

√3´n= −1

2

³1− i

√3´n

.

Soit en utilisant la forme trigonométrique de q¯1− i

√3¯= 2 et arg

³1− i

√3´= −π

3.

αn = −2n−1 e−i nπ / 3.En particulier

α3 = −22 e−i π = 4 et α4 = −23 e−i 4π / 3 = 4³1− i

√3´.

Ce qui prouve que les assertions 8 b) et 8 d) sont vraies et que les assertions 8 a) et 8 c)sont fausses.

Nous répondons :

Question 08 : a) Fausse b) Vraie c) Fausse d) Vraie e) Fausse

118 Questionnements Automatisables

17

Nous pouvons représenter les trois racines cubiques de l’unité sur le cercle trigonométrique,cf figure 17.1, elles forment les trois sommet d’un triangle équilatéral. Il est alors évident

que j = e2 i π / 3 =−1 + i

√3

2et donc que seule l’assertion 1 a) est vraie, les trois autres

étant fausses.

Nous avons les réponses :

Question 01 : a) Vraie b) Fausse c) Fausse d) Fausse e) Fausse

Nous avons

j2 = e4 i π / 3 = e−2 i π / 3 = j et j4 = j

et nous en déduisons les affixes des points A, B, C et D.

zA = −j2 = −ei π / 3, zB = 2

j2= 2 j, zC = − 2

j2= −2 j et zD =

4

j4= 4 j2. (1)

Remarque : Un excellente figure permet de conjecturer beaucoup de questions.

.....................................................................................

.....................................................................................

..................................................................................................................................................................................................................................................................................................................................................................................................................................................................................................................................................................................................................................................................................................... ..........................x

........

........

........

........

........

........

........

........

........

........

........

........

........

........

........

........

........

........

........

........

........

........

........

........

........

........

........

........

........

........

........

........

........

........

........

........

........

........

........

........

........

........

........

........

........

........

........

........

........

........

........

........

........

........

........

........

........

........

........

........

........

........

........

........

........

........

........

........

........

........

........

........

........

........

........

........

........

........

........

........

........

.............................

..........................i y

O..................................................................

........................................................................................................................................................................................................................................................................................................................ ......

......

......

................................................................................................

...................

..........................

........................................................................................................................................................................................................................................................................................................................................................................................................................................................................

......................................................................................................................................................................................................................................................................................................................................................................................................................................................................................................................................

............................................................

A(−j2)

B(2/j2)

C(−2/j2)

D(4/j2)

j

j2

1 2

−4 i

−4

γ

..............................................................

.............................................................

..............................................................

.............................................................

..............................................................

.......................................................................................................................................................................................................................................................................................................................................................................................................................................................................................................................................................................................................................................................................................................................................................................................................................................................................................................................................................................................................................................................................................................................................................................

................................................................................................................................................................................................................................................................................................................................................................................................................................................................................................................................................................................................................................................................................................................................................................................................................................................................................................................................................................................................................................................................................................................................................................................................................................................................................................................................................................................................................................................................................................................................................................................................................................................................................................................................................................................................................................................................................................................................................................................................................................................................................................................................................................................................................................................................................

................................................................................................................................................................................................................................................................................................................................................................................................................................................................................................................................................................................................................................................................................................................................................................................................................................

...........................................................................................................................................................................

....................................................................

...................................................................................................

......................................................................................................................................................................................................................................................................................................................................................................................................................................................................................................................................................................................................................................................................................................................................................................................................................................................................................................................................................................................................................................................................................................................................................................................................................................................................................................................................................................................................................................................................................................................................................................................................................................................................................................................................................................................................................................................................................................................................................................................................................................

.....................................................................................................................................................................................................................................................................................................................................................................................................................................................................................................................................................................................................................................................................................................................................................................................................................................................................................................................................................................................................................................................................

Figure 17.1

Corrigés 119

Si une équation du cercle circonscrit (Γ) au triangle ABC est

x2 + y2 + a x+ b y + c = 0

alors les valeurs de a, b et c sont déterminées en exprimant que les points A, B et C sontsur (Γ) .

Les coordonnées de ces points sont d’après (1)

A =

Ã1

2,

√3

2

!, B =

³−1,√3´et C =

³1, −√3´,

d’où le système linéaire de trois équations à trois inconnues

A ∈ (Γ)⇔ 1

2a+

√3

2b+ c = −1

B ∈ (Γ)⇔ −a+√3 b+ c = −4

C ∈ (Γ)⇔ a−√3 b+ c = −4 .

Par addition des deux dernières, nous obtenons c = −4.En portant dans les deux premières ce résultat et en multipliant la première par 2

a+√3 b = 6 et − a+

√3 b = 0 ,

nous obtenons par addition b =√3 et a = 3.

Nous en déduisons que les assertions 2 d), 3 b) et 4 b) sont vraies et que les autresassertions des question 2, 3 et 4 sont fausses.

Nous avons les réponses :

Question 02 : a) Fausse b) Fausse c) Fausse d) Vraie e) Fausse

Question 03 : a) Fausse b) Vraie c) Fausse d) Fausse e) Fausse

Question 04 : a) Fausse b) Vraie c) Fausse d) Fausse e) Fausse

L’équation du cercle circonscrit (Γ) au triangle ABC est donc

x2 + y2 + 3x+√3 y − 4 = 0 ,

ce qui implique qu’il ne passe pas par le point O, il faudrait pour cela que c = 0. Nous endéduisons que l’assertion 5 b) est fausse.

Le point D ayant pour coordonnées¡−2, −2√3¢ ne sera sur le cercle (Γ) que si

(−2)2 +³−2√3´2+ 3 (−2) +

√3³−2√3´− 4

est nul. Ce qui est bien vérifié, c’est ici qu’une bonne figure donne le résultat très vite.Toutefois, un contrôle par le calcul est plus sûr. Nous acceptons l’assertion 5 a).

Nous pouvons écrire l’équation du cercle (Γ) sous la forme réduiteµx+

3

2

¶2+

Ãy +

√3

2

!2= 7,

d’où le centre γ de (Γ) est le point de coordonnées

Ã−32, −√3

2

!qui est situé sur le

segment [BD], car les coordonnées de γ sont les demi-sommes des coordonnées de B etD, et donc pas sur le segment [AD] . L’assertion 5 c) est vraie et 5 d) fausse.

120 Questionnements Automatisables

Nous avons les réponses :

Question 05 : a) Vraie b) Fausse c) Vraie d) Fausse e) Fausse

La mesure complexe d’un vecteur est la différence des affixes de ses extrémités, d’oùd’après (1) h−−→

CBi= zB − zC = −1 + i

√3−

³1− i

√3´= −2 + 2 i

√3. (2)

Toutes les assertions proposées à la question 6 étant fausses, nous devons cocher la case6 e). Prudence, comme toujours dans ce cas là. Toutefois, B et C étant symétriques par

rapport à l’origine, il est évidenth−−→CB

i= 2 zB, ce qui donne le résultat plus rapidement.

Il faut souvent réfléchir, un peu, avant de faire les calculs.

Nous avons les réponses :

Question 06 : a) Fausse b) Fausse c) Fausse d) Fausse e) Vraie

De manière analogue, nous avons d’après (1)h−−→CD

i= zD − zC = −2− 2 i

√3−

³1− i

√3´= −3− i

√3 . (3)

Nous acceptons l’assertion 7 b) et nous rejetons les trois autres.

Nous avons les réponses :

Question 07 : a) Fausse b) Vraie c) Fausse d) Fausse e) Fausse

Nous continuons et nous avons d’après (1)h−→AB

i= zB − zA = −1 + i

√3−

Ã1

2+ i

√3

2

!= −3

2+ i

√3

2. (4)

Nous acceptons l’assertion 8 d) et nous rejetons les trois autres.

Nous avons les réponses :

Question 08 : a) Fausse b) Fausse c) Fausse d) Vraie e) Fausse

Enfin, nous avons d’après (1)h−−→AD

i= zD − zA = −2− 2 i

√3−

Ã1

2+ i

√3

2

!= −5

2− 5 i

√3

2. (5)

Nous acceptons l’assertion 9 b) et nous rejetons les trois autres.

Nous avons les réponses :

Question 09 : a) Fausse b) Vraie c) Fausse d) Fausse e) Fausse

D’après les relations (2) et (3) , nous avons

z0 =

h−−→CB

ih−−→CD

i = −2 + 2 i√3−3− i√3= 2

1− i√3

3 + i√3= −2

3i√3 .

La première expression est obtenue par factorisation et la dernière en multipliant et endivisant par la quantité conjuguée du dénominateur.

Corrigés 121

De même, d’après les relations (4) et (5) , nous avons

z00 =

h−→AB

ih−−→AD

i = −32 + i

√3

2

−52− 5 i

√3

2

=1

5

3− i√3

1 + i√3= −1

5i√3.

Le dernier résultat est obtenu comme précédemment.

Le rapport des mesures complexes de deux vecteurs est le nombre complexe ayant pourmodule le rapport des longueurs des vecteurs et pour argument l’angle orienté des vecteurs,soit pour z0

CB

CD=

¯−23i√3

¯=

2√3et

³−−→CD,

−−→CB

´= arg

µ−23i√3

¶= −π

2

et pour z00

AB

AD=

¯−15i√3

¯=

√3

5et

³−−→AD,

−→AB

´= arg

µ−15i√3

¶= −π

2.

Ce qui prouve que les angles inscrits\DCB et\DAB interceptent le même arc DB, doncles quatre points A, B, C et D sont cocycliques, ce qui a déjà été prouvé à la question 5.Cela permet d’éliminer les réponses au hasard. Nous acceptons l’assertion 10 c) et nousrejetons les trois autres.

Nous avons les réponses :

Question 10 : a) Fausse b) Fausse c) Vraie d) Fausse e) Fausse

Nous remarquons que les triangles DCB et DAB sont rectangles, d’où le centre γ ducercle (Γ) est bien le milieu du segment [BD] , cf question 5.Ces propriétés sont vérifiées sur la figure 17.1, encore une raison supplémentaire de faireune très bonne figure.

La rigueur et la précision sont toujours récompensées.

18

1. Il faut, en premier, faire une bonne figure, cf figure 18.1. Nous donnons des valeursnumériques à a et à λ et nous construisons les point B et I, puis la droite (BI) et lamédiatrice (∆) du segment [AI] ce qui détermine le point M.

a) Une équation de la droite (BI) est de la forme y = ux+ v où les valeurs de u et dev sont déterminées par les conditions

B ∈ (BI) ⇔ a = ua+ vI ∈ (BI) ⇔ λ = v ,

soit u =a− λ

a(existe car a est non nul par hypothèse) et v = λ.

Les coordonnées du point M vérifient le systèmey =

a− λ

ax+ λ

y =λ

2

122 Questionnements Automatisables

soit pour λ 6= a (hypothèse)

M =

µaλ

2 (λ− a),λ

2

¶. (1)

........

........

........

........

........

........

........

........

........

........

........

........

........

........

........

........

........

........

........

........

........

........

........

........

........

........

........

........

........

........

........

........

........

........

........

........

........

........

........

........

........

........

........

........

........

........

........

........

........

........

........

........

........

........

........

........

........

........

........

........

........

........

........

........

........

........

........

........

........

........

........

........

........

........

........

........

........

........

........

........

...........................

..........................

y

..................................................................................................................................................................................................................................................................................................................................................................................................................................................................................................................................................................................................................................................................................................... ..........................xA

a/2

a/2

a

a

I(λ)

B

M

(∆)

...........................................................................................................................................................................................

..........................................................................................................................................................................................................................................................................................................................................................................................................................................................................................................................................................................................................

..................................................................................................................................................................................................................................................................................................................................................................................................................

....

.....

....

.....

....

.....

.....

.....

.....

.....

.....

.....

.....

.....

....

.....

....

.....

....

.....

....

.....

....

.....

....

.....

....

.....

....

.....

.....

.....

.....

.....

.....

.....

....

.....

.....

.....

.....

.....

.....

....

.....

....

.....

....

.....

....

.....

....

.....

....

.....

....

.....

....

.....

.....

.....

.....

.....

.....

.....

.....

.....

....

.....

....

.....

....

.....

.

............................................................................................................................................................................................................................................................................................................................................................

***********************************************************************************************************************************************************************************************************************************************************************************************************************************************************************************************************************************************************************************************************************************************************************************************************************************************************************************************************

**********************************************************************************************************************************************************************************************************************************************************************************************************************************************************************************************************************************************************************************************************************************************************************************************************************************************************************************************************************************************************************************************************************************************************************************************************************************************************************************************************************************************************************************************************************************************************************************************************************************************************

****************************************************************************************************************************************************************************************************************************************************************************************************************************************************************************************************************************************************************************************************************************************************************************************************************************************************************************************************************************************************************************************************************************************************************************************************************************************************************************************************************************************************************************************************************************************************************************************************************************************************************************************************************************************************

Figure 18.1

b) La médiatrice (∆) du segment [AI] est aussi la bissectrice de l’angle [AMI. Ainsi(∆) est une des bissectrices de l’angle (MA, MB) , plus précisément si λ > 0 c’estla bissectrice intérieure et si λ < 0 c’est la bissectrice extérieure, cf figure 18.1.

c) Nous éliminons λ entre les coordonnées du point M dans la relation (1) et nousobtenons

x =a y

2 y − a⇔ y =

a x

2x− a. (2)

La fonction homographique h : x→ a x

2x− aest involutive car h = h−1.

La courbe représentative de h est une hyperbole équilatère passant par les pointsA, B et M et d’asymptotes les droites d’équation x =

a

2et y =

a

2.Nous avons

construit H sur la figure 18.1.

2. Nous pouvons écrire f (z) sous la forme canonique

z2 − 2 (4 + i) z + 15 = (z − (4 + i))2 + 15− (4 + i)2

= (z − 4− i)2 − 8 i .D’après ce qui précède, l’équation f (z) = 0 est

(z − 4− i)2 − 8 i = 0 ,soit en remarquant que 8 i = (2 + 2 i)2

(z − 4− i)2 = (2 + 2 i)2 ,

d’où α = 6 + 3 i et β = 2− i.

Remarque : Il est également possible d’obtenir α et β en calculant le discriminant del’équation du second degré f(z) = 0.

Corrigés 123

a)

b) Nous avons la factorisation

z0 = f (z) = (z − α) (z − β) ,

d’où en notanth−−→UM

ieth−−→VM

iles

mesures complexes respectives z − αet z − β, nous obtenons

|z0| = UM × VM

arg z0 =³−→Ax,−−→UM

´+³−→Ax,−−→VM

´.

En notant−→D i avec i = 1..4 une des

directions des bissectrices de l’angle(MU, MV ) , cf figure 18.2, nous pou-vons écrire

...................................................................................................................................................................................................................... ..........................O x

........

.....

M

V

U

D4

D2

D3

D1

............................................................................................................................................................................................................................................................

........

........

........

..

............................................................................................................................................................................................................................................................

..........................

........................

........................

........................

........................

....................................

.................................................................................................................... ................

....................................................................................

................

....................................................................................

................

Figure 18.2³−→Ax,−−→UM

´+³−→Ax,−−→VM

´=

³−→Ax,−−→MU

´+³−→Ax,−−→MV

´= 2

³−→Ax,−→D i

´+³−→D i,−−→MU

´+³−→D i,−−→MV

´= 2

³−→Ax,−→D i

´.

la première égalité est obtenu en remplaçant−−→UM et

−−→VM par

−−→MU et

−−→MV , ce qui

revient à ajouter π + π = 2π, la deuxième est la relation de Chasles sur les angleset la troisième par définition des bissectrices.En conclusion

|z0| = UM × VM et arg z0 = 2³−→Ax,−→D i

´= 2 (Ax, D) (mod 2π) . (3)

En effet, nous pouvons remplacer les vecteurs−→D1 par

−→D3 et

−→D2 par

−→D4 ce

qui revient à ajouter 2π et donc l’écrire pour une des bissectrices D de l’angle(MU, MV ) .

c) z0 réel si et seulement si arg z0 = 0 (mod π) , donc d’après (3)

2 (Ax, D) = 0 (mod π)⇔ (Ax, D) = 0³mod

π

2

´,

ce qui confirme que D est bien l’une des deux bissectrices.Nous en déduisons que z0 est réel si et seulement les bissectrices de l’angle(MU, MV ) sont parallèles aux axes, donc ont une direction fixe.

d) Pour rapprocher les deux questions, nous allons modifier les notations.Les points U et V de la question 2 sont les points A et B de la question 1, la valeurde a est 4.Si le points M décrit l’hyperbole H alors z0 est réel. Il suffit de déterminer lesparties de H telles que z0 soit strictement négatif, avec la relation (3) , nous avons

arg z0 =½

π2 (Ax, D)

(mod 2π) (4)

soit (Ax, D) =π

2(mod π), ce qui est logique en angle de droites. Le choix de la

bissectrice intérieure pour D partage H en une zone, marquée en gras, où D estverticale, c’est celle cherchée d’après la relation (4) , cf figure 18.3, et une zone oùD est horizontale, marquée en pointillé.

3. Pour z = x+ i y, nous avons dans z0 = f(z)

x0 + i y0 = (x+ i y)2 − 2 (4 + i) (x+ i y) + 15

= x2 − y2 − 8x+ 2 y + 15 + 2 i (x y − x− 4 y)

124 Questionnements Automatisables

et nous en déduisons que

½x0 = x2 − y2 − 8x+ 2 y + 15y0 = 2 (x y − x− 4 y) . (5)

L’ensemble E du 2. d) est donc d’après (5) défini par le système½

x2 − y2 − 8x+ 2 y + 15 < 0x y − x− 4 y = 0 . (6)

........

........

........

........

........

........

........

........

........

........

........

........

........

........

........

........

........

........

........

........

........

........

........

........

........

........

........

........

........

........

........

........

........

........

........

........

........

........

........

........

........

........

........

........

........

........

........

........

........

........

........

........

........

........

........

........

........

........

........

........

........

........

........

........

........

........

........

........

........

........

.................................

..........................i y

........................................................................................................................................................................................................................................................................................................................................................................................................................................................................................................................................................................................................................................................................................................... ..........................x

U

O6

2

3 i

−i

V

M

(D)

.................................................................................................................................................................................................................

...............................................................................................................................................................................................................................................................................................................................................................................................................................................................

....................................................................................................................................................................................................................................................................................................................................................................................................................................................................................................................................................................................................................................................................................................................................................

....

.....

.....

.....

....

.....

....

.....

....

.....

....

.....

....

.....

....

.....

....

.....

....

.....

....

.....

....

.....

....

.....

....

.....

.....

.....

....

.....

....

.....

....

.....

....

.....

....

.....

....

.....

....

.....

....

.....

....

.....

....

.....

....

.....

....

.....

....

.....

.....

.....

.....

.....

.....

....

.....

....

.....

....

...

................................................................................................................................................................................................................................................................................................................................................................

..............................................................................................................................................

............................................................................................

*********************************************************************************************************************************************************************************************

*********************************************************************************************************************************************************************************************************************************************************************************************************************************************************************************************************************************************************************************************************************************************************************************************************************************************************************************************************************************************************************************************************************************************************************************************************************************************************************************************************************************************************************************************************************************************************************************************************************************************************************************************************************************************************************************************************************************************************************************************************************************************************************************************************************************************************************************************************************************************************************************************

Figure 18.3

Nous retrouvons l’hyperbole H du 1et une inéquation à deux inconnuesdont les solutions forment un anglequi délimite l’ensemble E.L’équation

x2 − y2 − 8x+ 2 y + 15 = 0représente une courbe du second de-gré, pour l’étudier il faut l’écrire sousla forme canonique

(x− 4)2 − (y − 1)2 = 0miracle !, cela veut dire que l’angle estlimité par les deux droites (d1) et (d2)d’équation respectives x − y − 3 = 0et x + y − 5 = 0. La partie de l’anglevérifiant (6) est celle qui ne contientpas l’origine, cf figure 18.4.

........

........

........

........

........

........

........

........

........

........

........

........

........

........

........

........

........

........

........

........

........

........

........

........

........

........

........

........

........

........

........

........

........

........

........

........

........

........

........

........

........

........

........

........

........

........

........

........

........

........

........

........

........

........

........

........

........

........

...............................

..........................

y

.................................................................................................................................................................................................................................................................................................................................................................................................................................... ..........................x

UO

(d1)

(d2)

5

−3

5

3

V

.............................................................................................................................................................................................................................................................................................................................................................................................................................................................................................................................................................................

...................................................................................................................................................................................................................................................................................................................................................................................................................................................................................................................................................

.....

.....

....

.....

....

.....

....

.....

....

.....

....

.....

....

.....

....

.....

.....

.....

....

.....

.....

.....

.....

....

.....

....

.....

....

.....

....

.....

....

.....

....

.....

....

.....

....

.....

....

.....

....

.....

....

.....

.....

.....

.....

.....

.....

....

.....

....

.....

...

....................................................................................................................................................................................................................

........................................................................................

........................................................

*****************************************************************************************************************************************************************************************************************************************************************************************************

**************************************************************************************************************************************************************************************************************************************************************************************************************************************************************************************************************

Figure 18.4

Corrigés 125

Nous pouvons maintenant répondre à toutes les assertions des 12 questionsposées.

D’après la relation (1) , les assertions 1 b), 1 c) et 2 a) sont vraies et les cinq autres fausses.

Remarquer la méchanceté de l’assertion 1 c), elle s’adresse à ceux qui ne savent pas lire.

Nous avons les réponses :

Question 01 : a) Fausse b) Vraie c) Vraie d) Fausse e) Fausse

Question 02 : a) Vraie b) Fausse c) Fausse d) Fausse e) Fausse

La médiatrice (∆) du segment [AI] est telle queMA =MI. Or il n’y a pour tout I aucunlien commun avec la longueur MB, nous pouvons donc éliminer les assertions 3 a), 3 b)et 3 c).

Pour 3 d), c’est plus délicat, carMA 6=MB est bien vérifié (en général), mais est-ce vraipour tout I ?

Là est la question.

Raisonnons par l’absurde (ab absurdo pour les latinistes), si MA =MB alors M est surla médiatrice du segment [AB] .Ainsi il existe I tel que MA =MI =MB, donc nous ne pouvons pas accepter l’assertion3 d), nous choisissons de répondre vraie à l’assertion 3 e).

Nous avons les réponses :

Question 03 : a) Fausse b) Fausse c) Fausse d) Fausse e) Vraie

D’après la relation (2) et la figure 18.1, les assertions 4 a) et 4 c) sont vraies et les deuxautres sont fausses.

Nous avons les réponses :

Question 04 : a) Vraie b) Fausse c) Vraie d) Fausse e) Fausse

Les racines de l’équation f(z) = 0 sont

α = 6 + 3 i et β = 2− i.

Nous en déduisons que l’assertion 5 a) est vraie et les trois autres fausses.

ATTENTION : Ne pas confondre α et β, sinon il faut accepter 5 c). Dur - dur.

Nous avons les réponses :

Question 05 : a) Vraie b) Fausse c) Fausse d) Fausse e) Fausse

Toutes les assertions proposées à la question 6 étant fausses, nous devons répondre vraieà la case e.

Cas toujours délicat, à examiner et à ré-examiner. Sait-on jamais !Nous avons les réponses :

Question 06 : a) Fausse b) Fausse c) Fausse d) Fausse e) Vraie

La relation (3) prouve que les assertions 7 a) et 8 b) sont vraies et que les six autres sontfausses.

126 Questionnements Automatisables

ATTENTION : ne pas confondre à π près et à 2π près.

Nous avons les réponses :

Question 07 : a) Vraie b) Fausse c) Fausse d) Fausse e) Fausse

Question 08 : a) Fausse b) Vraie c) Fausse d) Fausse e) Fausse

Un examen de la figure 18.3 prouve que les deux assertions 9 a) et 9 b) sont fausses. Vousconstatez une nouvelle fois l’intérêt de faire une bonne figure.D’après la relation (3) , les bissectrices ont une direction fixe si et seulement si arg z0 estconstant, ce qui est le cas pour les deux assertions 9 c) et 9 d), qui sont donc vraies.

Nous avons les réponses :

Question 09 : a) Fausse b) Fausse c) Vraie d) Vraie e) Fausse

La relation (4) prouve que E est contenue dans une hyperbole et admet un centre desymétrie, cf figure 18.3. Les assertions 10 b) et 10 c) sont donc vraies et les deux autresfausses. Le seul cas douteux est la proposition de l’assertion 10 d). Il faut exclure lespoint U et V car la bissectrice n’est pas définie en ces points, il faudrait examiner ce quise passe à la limite.Nous avons les réponses :

Question 10 : a) Fausse b) Vraie c) Vraie d) Fausse e) Fausse

D’après la relation (5) , les assertions 11 a), 11 b) et 11 d) sont fausses. La factorisationcanonique de x0 prouve que l’assertion 11 c) est vraie. Merci à l’auteur de nous donnerun coup de pouce.

Nous avons les réponses :

Question 11 : a) Fausse b) Fausse c) Vraie d) Fausse e) Fausse

Un bon examen de la figure 18.4 prouve que seule l’assertion 12 d) est vraie, les troisautres étant fausses. Il faut bien voir l’inéquation (y + 1) (y − 3) > 0, c’est beau lesmathématiques et les Questionnements Automatisables.Nous avons les réponses :

Question 12 : a) Fausse b) Fausse c) Fausse d) Vraie e) Fausse

19

Nous avons Z = f(z) =i z − 1z − 2 i , d’où en exprimant z en fonction de ZZ (z − 2 i) = i z − 1z (Z − i) = 2 i Z − 1

z = f−1 (Z) =2 i Z − 1Z − i

,

Corrigés 127

nous obtenons l’application réciproque f−1 de f.

Ce qui prouve que f est une application bijective de D = C\2 i sur D0 = C\ i.Ce qui prouve que l’assertion 1 a) est fausse. D’autre part si f est une involution alorsf = f−1 ce qui confirme que l’assertion 1 a) est fausse même si le et est remplacé par ou.

Si z est réel alors Z = f (x) =i x− 1x− 2 i n’est ni réel, ni imaginaire pur, pour tout x réel.

Les assertions 1 b) et 1 d) sont donc fausses.

Si z est un imaginaire pur alors Z =i (i y)− 1i y − 2 i = i

y + 1

y − 2 est un imaginaire pur, pourtout y réel, l’assertion 1 c) est donc fausse.

Nous devons répondre vraie à la case e de la première question.

Le démarrage est dur - dur.

Nous avons les réponses :

Question 01 : a) Fausse b) Fausse c) Fausse d) Fausse e) Vraie

Nous posons Z = X + i Y et z = x+ i y, où X, Y, x et y sont réels et nous pouvons écrire

X + i Y =−y − 1 + i x

x+ i (y − 2) ,

soit en multipliant et en divisant par la quantité conjuguée du dénominateur x− i (y − 2)

X + i Y =(−y − 1 + i x) [x− i (y − 2)][x+ i (y − 2)] [x− i (y − 2)]

=−3x+ i (x2 + y2 − y − 2)

x2 + (y − 2)2 . (1)

Nous en déduisons

α = −3, β = 0, γ = 0, δ = 0, ε = −1 et ϕ = −2,

d’où seules les assertions 2 a), 3 c) et 4 c) sont vraies, toutes les autres étant fausses.

Remarque : Certaines assertions précédentes sont en partie vraie et en partie fausse, ellessont donc fausses, sauf si dans le texte le et est remplacé par ou.

Il faut bien lire les sujets de Questionnements Automatisables.

Nous avons les réponses :

Question 02 : a) Vraie b) Fausse c) Fausse d) Fausse e) Fausse

Question 03 : a) Fausse b) Fausse c) Vraie d) Fausse e) Fausse

Question 04 : a) Fausse b) Fausse c) Vraie d) Fausse e) Fausse

128 Questionnements Automatisables

Nous déduisons de (1) que Z est réel(respectivement imaginaire pur) si et seule-ment si la partie imaginaire de Z est nulle,soit : x2+y2−y−2 = 0 (respectivement si lapartie réelle de Z est nulle, soit : −3x = 0).Attention : il faut tenir compte de D pourpréciser les ensembles U et V.

U :

½x2 + y2 − y − 2 = 0

x+ i y 6= 2 i .

U est le cercle d’équation

x2 +

µy − 1

2

¶2=7

4,

dont le centre u à pour coordonnées (0, 1/2)et de rayon R =

√7/2, privé du point B.

Remarquer que ce cercle admet le segment[AB] pour diamètre.

........................................................................................................................................................................................................................................................................................................................................................................................................................................................................................................................................................... ..........................x

........

........

........

........

........

........

........

........

........

........

........

........

........

........

........

........

........

........

........

........

........

........

........

........

........

........

........

........

........

........

........

........

........

........

........

........

........

........

........

........

........

........

........

........

........

........

........

........

........

........

........

........

........

........

........

........

........

........

........

........

........

........

...............................

..........................i y

O

.......................................................................................................................................................................................................................................................................................................................................................................................................................................................................................................

....................................

................................................................................................................................................................................................................................................................................................................................................................................................................................................................................................................................................................................................................................................................................

−i

2 i

1/2

−1 2

A

B

u

C D

U

r(U)

.....................................................................................................................................................................................................................................................................................................................

.................................................................................................................................................................................................................................................................................................................................................................................................................................................................................

.....................

..............................................................................................................................................................................................................................

Figure 19.1

Nous pouvons accepter l’assertion 5 a), faire très attention au libellé “U est sur” car sansle sur l’assertion 5 a) est fausse et rejeter les autres assertions.Nous avons les réponses :

Question 05 : a) Vraie b) Fausse c) Fausse d) Fausse e) Fausse

V est la droite d’équation x = 0, soit l’axe imaginaire, privé du point B.

Nous pouvons accepter l’assertion 6 a), faire encore attention au libellé “V est sur”, etrejeter les autres assertions.

Nous avons les réponses :

Question 06 : a) Vraie b) Fausse c) Fausse d) Fausse e) Fausse

La rotation définie par r (z) = e−i π / 2 z est la rotation de centre O et d’angle −π2.

Elle transforme le cercle de diamètre [AB] en un cercle de diamètre [CD] où C et D ontpour affixes respectives −1 et 2, voir figure 19.1.r (U) est donc le cercle de diamètre [CD] privé du point D. Toutes les propositions desquatre assertions de la question 7 sont donc fausses, nous répondons vraie à la case e.Ceci est attendu car U étant un cercle privé d’un point, il en est de même de r (U) carcontrairement aux deux questions précédentes il n’y a pas de sur.

Nous avons les réponses :

Question 07 : a) Fausse b) Fausse c) Fausse d) Fausse e) Vraie

r (V ) est l’image de l’axe imaginaire (privé du point B) par la rotation, c’est donc l’axeréel (privé du point D).

Nous pouvons accepter l’assertion 8 c) à cause du sur et rejeter les trois autres.

Nous avons les réponses :

Question 08 : a) Fausse b) Fausse c) Vraie d) Fausse e) Fausse

Corrigés 129

Les points A et B ont pour coordonnées respectives (0, −1) et (0, 2) , donc une équationde la médiatrice ∆ du segment [AB] est y =

1

2, ce qui permet d’accepter l’assertion 9 b)

et de rejeter les trois autres.

Nous avons les réponses :

Question 09 : a) Fausse b) Vraie c) Fausse d) Fausse e) Fausse

Tout point de la droite ∆ admet pour affixe z = x+i

2, d’où

f

µx+

i

2

¶=

i

µx+

i

2

¶− 1µ

x+i

2

¶− 2 i

=i x− 3

2

x− 32i.

Nous cherchons x tel que fµx+

i

2

¶= −i, la seule solution est x = 0. L’assertion 10 c)

est donc vraie et les trois autres fausses.

Nous avons les réponses :

Question 10 : a) Fausse b) Fausse c) Vraie d) Fausse e) Fausse

20

Cet exercice est plus une épreuve de français que de mathématiques. Il s’agit de biencomprendre l’énoncé et de répondre correctement aux questions posées. Une lecture troprapide peut induire des erreurs. Les “étourdis” seront ainsi éliminés. Il est fréquent derencontrer des situations semblables. Le bon sens rien que le bon sens...

Nous pouvons représenter l’ensemble des 260 candidats à ce concours selon le baccalauréatobtenu en 2041 à l’aide de la représentation suivante :

. . . . . . . . . . . . . . . . . . . . . . . . . . . . . . . . . .. . . . . . . . . . . . . . . . . . . . . . . . . . . . . . . . . . .. . . . . . . . . . . . . . . . . . . . . . . . . . . . . . . . . .. . . . . . . . . . . . . . . . . . . . . . . . . . . . . . . . . . .. . . . . . . . . . . . . . . . . . . . . . . . . . . . . . . . . .. . . . . . . . . . . . . . . . . . . . . . . . . . . . . . . . . . .. . . . . . . . . . . . . . . . . . . . . . . . . . . . . . . . . .. . . . . . . . . . . . . . . . . . . . . . . . . . . . . . . . . . .. . . . . . . . . . . . . . . . . . . . . . . . . . . . . . . . . .. . . . . . . . . . . . . . . . . . . . . . . . . . . . . . . . . . .. . . . . . . . . . . . . . . . . . . . . . . . . . . . . . . . . .. . . . . . . . . . . . . . . . . . . . . . . . . . . . . . . . . . .. . . . . . . . . . . . . . . . . . . . . . . . . . . . . . . . . .. . . . . . . . . . . . . . . . . . . . . . . . . . . . . . . . . . .. . . . . . . . . . . . . . . . . . . . . . . . . . . . . . . . . .. . . . . . . . . . . . . . . . . . . . . . . . . . . . . . . . . . .. . . . . . . . . . . . . . . . . . . . . . . . . . . . . . . . . .. . . . . . . . . . . . . . . . . . . . . . . . . . . . . . . . . . .. . . . . . . . . . . . . . . . . . . . . . . . . . . . . . . . . .. . . . . . . . . . . . . . . . . . . . . . . . . . . . . . . . . . .. . . . . . . . . . . . . . . . . . . . . . . . . . . . . . . . . .. . . . . . . . . . . . . . . . . . . . . . . . . . . . . . . . . . .. . . . . . . . . . . . . . . . . . . . . . . . . . . . . . . . . .. . . . . . . . . . . . . . . . . . . . . . . . . . . . . . . . . . .

..

.

..

.

..

.

..

.

..

.

..

.

..

.

..

..

..

.

..

.

..

.

.

..

.

..

.

..

.

..

.

..

.

..

.

..

.

..

..

..

.

..

.

.

..

.

..

.

..

.

..

.

..

.

..

.

..

.

..

..

..

.

..

.

..

.

.

..

.

..

.

..

.

..

.

..

.

..

.

..

.

..

..

..

.

..

.

.

..

.

..

.

..

.

..

.

..

.

..

.

..

.

..

..

..

.

..

.

..

.

.

..

.

..

.

..

.

..

.

..

.

..

.

..

.

..

..

..

.

..

.

.

..

.

..

.

..

.

..

.

..

.

..

.

..

.

..

..

..

.

..

.

..

.

.

..

.

..

.

..

.

..

.

..

.

..

.

..

.

..

..

..

.

..

.

.

..

.

..

.

..

.

..

.

..

.

..

.

..

.

..

..

..

.

..

.

..

.

.

..

.

..

.

..

.

..

.

..

.

..

.

..

.

..

..

..

.

..

.

.

..

.

..

.

..

.

..

.

..

.

..

.

..

.

..

..

..

.

..

.

..

.

.

..

.

..

.

..

.

..

.

..

.

..

.

..

.

..

..

..

.

..

.

.

..

.

..

.

..

.

..

.

..

.

..

.

..

.

..

..

..

.

..

.

..

.

.

..

.

..

.

..

.

..

.

..

.

..

.

..

.

..

..

..

.

..

.

.

..

.

..

.

..

.

..

.

..

.

..

.

..

.

..

..

..

.

..

.

..

.

.

..

.

..

.

..

.

..

.

..

.

..

.

..

.

..

..

..

.

..

.

.

..

.

..

.

..

.

..

.

..

.

..

.

..

.

..

..

..

.

..

.

..

.

.

..

.

..

.

..

.

..

.

..

.

..

.

..

.

..

..

..

.

..

.

.

..

.

..

.

..

.

..

.

..

.

..

.

..

.

..

..

..

.

..

.

..

.

.

..

.

..

.

..

.

..

.

..

.

..

.

..

.

..

..

..

.

..

.

.

..

.

..

.

..

.

..

.

..

.

..

.

..

.

..

..

..

.

..

.

..

.

.

..

.

..

.

..

.

..

.

..

.

..

.

..

.

..

..

..

.

..

.

.

..

.

..

.

..

.

..

.

..

.

..

.

..

.

..

..

..

.

..

.

..

.

.

..

.

..

.

..

.

..

.

..

.

..

.

..

.

..

..

..

.

..

.

.

..

.

..

.

..

.

..

.

..

.

..

.

..

.

..

..

..

.

..

.

..

.

.

..

.

..

.

..

.

..

.

..

.

..

.

..

.

..

..

..

.

..

.

.

..

.

..

.

..

.

..

.

..

.

..

.

..

.

..

..

..

.

..

.

..

.

.

..

.

..

.

..

.

..

.

..

.

..

.

..

.

..

..

..

.

..

.

.

..

.

..

.

..

.

..

.

..

.

..

.

..

.

..

..

..

.

..

.

..

.

.

..

.

..

.

..

.

..

.

..

.

..

.

..

.

..

..

..

.

..

.

.

..

.

..

.

..

.

..

.

..

.

..

.

..

.

..

..

..

.

..

.

..

.

.

..

.

..

.

..

.

..

.

..

.

..

.

..

.

..

..

..

.

..

.

.

..

.

..

.

..

.

..

.

..

.

..

.

..

.

..

..

..

.

..

.

..

.

.

..

.

..

.

..

.

..

.

..

.

..

.

..

.

..

..

..

.

..

.

.

..

.

..

.

..

.

..

.

..

.

..

.

..

.

..

..

..

.

..

.

..

.

.

..

.

..

.

..

.

..

.

..

.

..

.

..

.

..

..

..

.

..

.

.

..

.

..

.

..

.

..

.

..

.

..

.

..

.

..

..

..

.

..

.

..

.

.

..

.

..

.

..

.

..

.

..

.

..

.

..

.

..

..

..

.

..

.

.

..

.

..

.

..

.

..

.

..

.

..

.

..

.

..

..

..

.

..

.

..

.

.

..

.

..

.

..

.

..

.

..

.

..

.

..

.

..

..

..

.

..

.

.

..

.

..

.

..

.

..

.

..

.

..

.

..

.

..

..

..

.

..

.

..

.

.

..

.

..

.

..

.

..

.

..

.

..

.

..

.

..

..

..

.

..

.

.

..

.

..

.

..

.

..

.

..

.

..

.

..

.

..

..

..

.

..

.

..

.

.

..

.

..

.

..

.

..

.

..

.

..

.

..

.

..

..

..

.

..

.

.

..

.

..

.

..

.

..

.

..

.

..

.

..

.

..

..

..

.

..

.

..

.

.

..

.

..

.

..

.

..

.

..

.

..

.

..

.

..

..

..

.

..

.

.

..

.

..

.

..

.

..

.

..

.

..

.

..

.

..

..

..

.

..

.

..

.

.

..

.

..

.

..

.

..

.

..

.

..

.

..

.

..

..

..

.

..

.

.

..

.

..

.

..

.

..

.

..

.

..

.

..

.

..

..

..

.

..

.

..

.

.

..

.

..

.

..

.

..

.

..

.

..

.

..

.

..

..

..

.

..

.

.

..

.

..

.

..

.

..

.

..

.

..

.

..

.

..

..

..

.

..

.

..

.

.

..

.

..

.

..

.

..

.

..

.

..

.

..

.

..

..

..

.

..

.

.

..

.

..

.

..

.

..

.

..

.

..

.

..

.

..

..

..

.

..

.

..

.

.

..

.

..

.

..

.

..

.

..

.

..

.

..

.

..

..

..

.

..

.

.

..

.

..

.

..

.

..

.

..

.

..

.

..

.

..

..

..

.

..

.

..

.

.

..

.

..

.

..

.

..

.

..

.

..

.

..

.

..

..

..

.

..

.

.

..

.

..

.

..

.

..

.

..

.

..

.

..

.

..

..

..

.

..

.

..

.

.

..

.

..

.

..

.

..

.

..

.

..

.

..

.

..

..

..

.

..

.

.

..

.

..

.

..

.

..

.

..

.

..

.

..

.

..

..

..

.

..

.

..

.

.

..

.

..

.

..

.

..

.

..

.

..

.

..

.

..

..

..

.

..

.

.

. . . . . . . . . . . . . . . . . . . . . . . . . . . .. . . . . . . . . . . . . . . . . . . . . . . . . . .. . . . . . . . . . . . . . . . . . . . . . . . . . . .. . . . . . . . . . . . . . . . . . . . . . . . . . .. . . . . . . . . . . . . . . . . . . . . . . . . . . .. . . . . . . . . . . . . . . . . . . . . . . . . . .. . . . . . . . . . . . . . . . . . . . . . . . . . . .. . . . . . . . . . . . . . . . . . . . . . . . . . .. . . . . . . . . . . . . . . . . . . . . . . . . . . .. . . . . . . . . . . . . . . . . . . . . . . . . . .. . . . . . . . . . . . . . . . . . . . . . . . . . . .. . . . . . . . . . . . . . . . . . . . . . . . . . .. . . . . . . . . . . . . . . . . . . . . . . . . . . .. . . . . . . . . . . . . . . . . . . . . . . . . . .. . . . . . . . . . . . . . . . . . . . . . . . . . . .. . . . . . . . . . . . . . . . . . . . . . . . . . .. . . . . . . . . . . . . . . . . . . . . . . . . . . .. . . . . . . . . . . . . . . . . . . . . . . . . . .. . . . . . . . . . . . . . . . . . . . . . . . . . . .

BacC (65)

BacD (91)

BacG (65)

Autres (39)

Figure 20.1

Nous notons dans la suite C, D, G et A les ensembles de candidats reçus respectivementaux baccalauréats BacC, BacD, BacG et BacAutres.

130 Questionnements Automatisables

Nous avons la répartition suivante :

Baccalauréat Nombre de reçus

C Card (C) = 65

D Card (D) = 91

G Card (G) = 65

A Card (A) = 39

Total 260

Ce qui nous permet d’écrire en appelant Ω l’univers correspondant aux reçus

Card (Ω) = Card (C) + Card (D) +Card (G) +Card (A)

= 65 + 91 + 65 + 39

= 260.

Il en résulte que l’assertion 1 b) est fausse.

Nous pouvons alors répondre aux autres assertions de la question 1.

Card (C ∪D) = Card (C) + Card (D)

= 65 + 91

= 156

car nous supposons qu’en 2041 il n’est pas possible de passer simultanément deuxbaccalauréat distincts [1].

Nous en déduisons que l’assertion 1 a) est vraie.

Il faut “décoder” les assertions 1 c) et 1 d).

Le groupe formé de ceux ayant le baccalauréat C, D ou G admet pour cardinal

Card (C ∪D ∪G) = Card (C) +Card (D) + Card (G)

= 65 + 91 + 65

= 221,

qui n’est autre que le complément à Ω de ceux ayant un baccalauréat Autres, soitCard (A) = 39.

Pour l’assertion 1 c)Card (C ∪D) et Card (G ∪A)

sont différents d’après ce qui précède et de même pour l’assertion 1 d)

Card (C ∪G) et Card (D ∪A)sont égaux.

Nous en déduisons que l’assertion 1 c) est fausse et que l’assertion 1 d) est vraie.

Nous répondons :

Question 01 : a) Vraie b) Fausse c) Fausse d) Vraie e) Fausse

Cette question considère les baccalauréats obtenus en 2041 et avant 2041.

L’énoncé donneCard (C ∩D) = 13 et Card (D ∩G) = 26

[1] Il était possible, certaines années, de concourir pour deux baccalauréats différents. Un scientifique etun littéraire. C’était le nec plus ultra. Le plaisir quoi.

Corrigés 131

ce qui donne la répartition suivante :

Baccalauréat Nombre de reçus en 2041 Reçus aux années antérieures

C Card (C) = 65 13 Baccalauréat D

D Card (D) = 91

G Card (G) = 65 26 Baccalauréat D

A Card (A) = 39

Nous pouvons accepter le diagramme de l’assertion 2 a) en marquant dans les partiescommunes le nombre des candidats ayant deux baccalauréats.

. . . . . . . . . . . . . . . . . . . . . . . . . . . . . . . . . . .. . . . . . . . . . . . . . . . . . . . . . . . . . . . . . . . . .. . . . . . . . . . . . . . . . . . . . . . . . . . . . . . . . . . .. . . . . . . . . . . . . . . . . . . . . . . . . . . . . . . . . .. . . . . . . . . . . . . . . . . . . . . . . . . . . . . . . . . . .. . . . . . . . . . . . . . . . . . . . . . . . . . . . . . . . . .. . . . . . . . . . . . . . . . . . . . . . . . . . . . . . . . . . .. . . . . . . . . . . . . . . . . . . . . . . . . . . . . . . . . .. . . . . . . . . . . . . . . . . . . . . . . . . . . . . . . . . . .. . . . . . . . . . . . . . . . . . . . . . . . . . . . . . . . . .. . . . . . . . . . . . . . . . . . . . . . . . . . . . . . . . . . .. . . . . . . . . . . . . . . . . . . . . . . . . . . . . . . . . .. . . . . . . . . . . . . . . . . . . . . . . . . . . . . . . . . . .. . . . . . . . . . . . . . . . . . . . . . . . . . . . . . . . . .. . . . . . . . . . . . . . . . . . . . . . . . . . . . . . . . . . .. . . . . . . . . . . . . . . . . . . . . . . . . . . . . . . . . .. . . . . . . . . . . . . . . . . . . . . . . . . . . . . . . . . . .. . . . . . . . . . . . . . . . . . . . . . . . . . . . . . . . . .. . . . . . . . . . . . . . . . . . . . . . . . . . . . . . . . . . .

. . . . . . . . . . . . . . . . . . . . . . . . . . . . . . . . . . . . . . . . . . . . . . . . . . . . . . . . . . . . . .. . . . . . . . . . . . . . . . . . . . . . . . . . . . . . . . . . . . . . . . . . . . . . . . . . . . . . . . . . . . . .. . . . . . . . . . . . . . . . . . . . . . . . . . . . . . . . . . . . . . . . . . . . . . . . . . . . . . . . . . . . . .. . . . . . . . . . . . . . . . . . . . . . . . . . . . . . . . . . . . . . . . . . . . . . . . . . . . . . . . . . . . . .. . . . . . . . . . . . . . . . . . . . . . . . . . . . . . . . . . . . . . . . . . . . . . . . . . . . . . . . . . . . . .. . . . . . . . . . . . . . . . . . . . . . . . . . . . . . . . . . . . . . . . . . . . . . . . . . . . . . . . . . . . . .. . . . . . . . . . . . . . . . . . . . . . . . . . . . . . . . . . . . . . . . . . . . . . . . . . . . . . . . . . . . . .. . . . . . . . . . . . . . . . . . . . . . . . . . . . . . . . . . . . . . . . . . . . . . . . . . . . . . . . . . . . . .. . . . . . . . . . . . . . . . . . . . . . . . . . . . . . . . . . . . . . . . . . . . . . . . . . . . . . . . . . . . . .. . . . . . . . . . . . . . . . . . . . . . . . . . . . . . . . . . . . . . . . . . . . . . . . . . . . . . . . . . . . . .. . . . . . . . . . . . . . . . . . . . . . . . . . . . . . . . . . . . . . . . . . . . . . . . . . . . . . . . . . . . . .. . . . . . . . . . . . . . . . . . . . . . . . . . . . . . . . . . . . . . . . . . . . . . . . . . . . . . . . . . . . . .. . . . . . . . . . . . . . . . . . . . . . . . . . . . . . . . . . . . . . . . . . . . . . . . . . . . . . . . . . . . . .. . . . . . . . . . . . . . . . . . . . . . . . . . . . . . . . . . . . . . . . . . . . . . . . . . . . . . . . . . . . . .. . . . . . . . . . . . . . . . . . . . . . . . . . . . . . . . . . . . . . . . . . . . . . . . . . . . . . . . . . . . . .. . . . . . . . . . . . . . . . . . . . . . . . . . . . . . . . . . . . . . . . . . . . . . . . . . . . . . . . . . . . . .. . . . . . . . . . . . . . . . . . . . . . . . . . . . . . . . . . . . . . . . . . . . . . . . . . . . . . . . . . . . . .. . . . . . . . . . . . . . . . . . . . . . . . . . . . . . . . . . . . . . . . . . . . . . . . . . . . . . . . . . . . . .. . . . . . . . . . . . . . . . . . . . . . . . . . . . . . . . . . . . . . . . . . . . . . . . . . . . . . . . . . . . . .. . . . . . . . . . . . . . . . . . . . . . . . . . . . . . . . . . . . . . . . . . . . . . . . . . . . . . . . . . . . . .. . . . . . . . . . . . . . . . . . . . . . . . . . . . . . . . . . . . . . . . . . . . . . . . . . . . . . . . . . . . . .. . . . . . . . . . . . . . . . . . . . . . . . . . . . . . . . . . . . . . . . . . . . . . . . . . . . . . . . . . . . . .. . . . . . . . . . . . . . . . . . . . . . . . . . . . . . . . . . . . . . . . . . . . . . . . . . . . . . . . . . . . . .. . . . . . . . . . . . . . . . . . . . . . . . . . . . . . . . . . . . . . . . . . . . . . . . . . . . . . . . . . . . . .. . . . . . . . . . . . . . . . . . . . . . . . . . . . . . . . . . . . . . . . . . . . . . . . . . . . . . . . . . . . . .. . . . . . . . . . . . . . . . . . . . . . . . . . . . . . . . . . . . . . . . . . . . . . . . . . . . . . . . . . . . . .. . . . . . . . . . . . . . . . . . . . . . . . . . . . . . . . . . . . . . . . . . . . . . . . . . . . . . . . . . . . . .. . . . . . . . . . . . . . . . . . . . . . . . . . . . . . . . . . . . . . . . . . . . . . . . . . . . . . . . . . . . . .. . . . . . . . . . . . . . . . . . . . . . . . . . . . . . . . . . . . . . . . . . . . . . . . . . . . . . . . . . . . . .. . . . . . . . . . . . . . . . . . . . . . . . . . . . . . . . . . . . . . . . . . . . . . . . . . . . . . . . . . . . . .. . . . . . . . . . . . . . . . . . . . . . . . . . . . . . . . . . . . . . . . . . . . . . . . . . . . . . . . . . . . . .. . . . . . . . . . . . . . . . . . . . . . . . . . . . . . . . . . . . . . . . . . . . . . . . . . . . . . . . . . . . . .. . . . . . . . . . . . . . . . . . . . . . . . . . . . . . . . . . . . . . . . . . . . . . . . . . . . . . . . . . . . . .. . . . . . . . . . . . . . . . . . . . . . . . . . . . . . . . . . . . . . . . . . . . . . . . . . . . . . . . . . . . . .. . . . . . . . . . . . . . . . . . . . . . . . . . . . . . . . . . . . . . . . . . . . . . . . . . . . . . . . . . . . . .. . . . . . . . . . . . . . . . . . . . . . . . . . . . . . . . . . . . . . . . . . . . . . . . . . . . . . . . . . . . . .. . . . . . . . . . . . . . . . . . . . . . . . . . . . . . . . . . . . . . . . . . . . . . . . . . . . . . . . . . . . . .

. . . . . . . . . . . . . . . . . . . . . . . . . . .. . . . . . . . . . . . . . . . . . . . . . . . . . . .. . . . . . . . . . . . . . . . . . . . . . . . . . .. . . . . . . . . . . . . . . . . . . . . . . . . . . .. . . . . . . . . . . . . . . . . . . . . . . . . . .. . . . . . . . . . . . . . . . . . . . . . . . . . . .. . . . . . . . . . . . . . . . . . . . . . . . . . .. . . . . . . . . . . . . . . . . . . . . . . . . . . .. . . . . . . . . . . . . . . . . . . . . . . . . . .. . . . . . . . . . . . . . . . . . . . . . . . . . . .. . . . . . . . . . . . . . . . . . . . . . . . . . .. . . . . . . . . . . . . . . . . . . . . . . . . . . .. . . . . . . . . . . . . . . . . . . . . . . . . . .. . . . . . . . . . . . . . . . . . . . . . . . . . . .. . . . . . . . . . . . . . . . . . . . . . . . . . .. . . . . . . . . . . . . . . . . . . . . . . . . . . .

BacC (65)

Bac C ∩D (13)BacD (91)

Bac G ∩D (26)

BacG (91)

Autres (39)

Figure 20.2

Ainsi l’assertion 2 a) est vraie et l’assertion 2 b) est fausse. En effet il n’est pas préciséque des candidats ont les baccalauréats C et G.

Les résultats illustrés par la figure 20.2 établissent que les assertions 2 c) et 2 d) sontfausses. C’est ici que la compréhension de la langue française prend de l’importance.ATTENTION les Questionnements Automatisables ne font aucun cadeau ...

Nous répondons :

Question 02 : a) Vraie b) Fausse c) Fausse d) Fausse e) Fausse

La probabilité pour un candidat d’être admis est donc p =100

260ce donne la répartition

suivante en fonction des baccalauréats obtenus en 2041

Baccalauréat de 2041 Nombre d’admis Selon Baccalauréat antérieur

C 65 ∗ 100 / 260 = 25 dont 5 Bac D

D 91 ∗ 100 / 260 = 35G 65 ∗ 100 / 260 = 25 dont 10 Bac D

A 39 ∗ 100 / 260 = 15

Total des reçus 100 dont 15 Bac D

ce qui permet de conclure que 20 reçus ont seulement le baccalauréat C, donc l’assertion3 a) est fausse.

Nous devons traduire pour les assertions 3 b) et 3 c) le au plus par un et un seul.

132 Questionnements Automatisables

15 reçus n’ont que le baccalauréat G donc l’assertion 3 b) est fausse, de même 35 reçusn’ont que le baccalauréat D donc l’assertion 3 c) est fausse. En fait il y a 50 reçus quiont le baccalauréat D au moins. Remarquer l’importance du français dans le libellé desassertions.Enfin il y a 10 reçus qui ont les baccalauréats D et G donc l’assertion 3 d) est fausse.

Toutes les assertions proposées étant fausses, nous devons répondre vraie à la case e.

Nous répondons :

Question 03 : a) Fausse b) Fausse c) Fausse d) Fausse e) Vraie

Nous pouvons représenter les reçus en fonction des baccalauréats obtenus :

Baccalauréats Nombre de reçus Démissions Promotion

C seul 20 20

D seul 35 35

G seul 15 15

A seul 15 10 5

C et D 5 5 0

G et D 10 5 5

Total des reçus 100 20 80

Il reste seulement 80 admis dans cette promotion. L’assertion 4 a) est fausse et l’assertion4 b) est vraie.

Il y a exactement (donc au plus) 5 reçus ayant deux baccalauréats et 75 reçus n’ayantqu’un seul baccalauréat. L’assertion 4 c) est donc vraie et l’assertion 4 d) est fausse.

Nous répondons :

Question 04 : a) Fausse b) Vraie c) Vraie d) Fausse e) Fausse

21

Nous pouvons représenter cette classe Ω de 40 élèves à l’aide de deux partitions.

La première en séparant filles et garçons :

F et F avec CardF = 8 et Card F = 32

et la seconde en séparant reçus (ou reçues) et collés (ou collées) :

R et R.Par hypothèse nous avons Prob (R |F ) = 0, 75 donc Card (R ∩ F ) = 8 ∗ 0, 75 = 6et Prob

¡R |F¢ = 0, 50 donc Card ¡R ∩ F¢ = 32 ∗ 0, 50 = 16. Ce qui peut être visualisé

sur la figure 21.1 page suivante.

Corrigés 133

. . . . . . . . . . . . . . . . . . . . . . . . . . . . . .. . . . . . . . . . . . . . . . . . . . . . . . . . . . .. . . . . . . . . . . . . . . . . . . . . . . . . . . . . .. . . . . . . . . . . . . . . . . . . . . . . . . . . . .. . . . . . . . . . . . . . . . . . . . . . . . . . . . . .. . . . . . . . . . . . . . . . . . . . . . . . . . . . .. . . . . . . . . . . . . . . . . . . . . . . . . . . . . .. . . . . . . . . . . . . . . . . . . . . . . . . . . . .. . . . . . . . . . . . . . . . . . . . . . . . . . . . . .. . . . . . . . . . . . . . . . . . . . . . . . . . . . .. . . . . . . . . . . . . . . . . . . . . . . . . . . . . .. . . . . . . . . . . . . . . . . . . . . . . . . . . . .. . . . . . . . . . . . . . . . . . . . . . . . . . . . . .. . . . . . . . . . . . . . . . . . . . . . . . . . . . .. . . . . . . . . . . . . . . . . . . . . . . . . . . . . .. . . . . . . . . . . . . . . . . . . . . . . . . . . . .. . . . . . . . . . . . . . . . . . . . . . . . . . . . . .. . . . . . . . . . . . . . . . . . . . . . . . . . . . .. . . . . . . . . . . . . . . . . . . . . . . . . . . . . .. . . . . . . . . . . . . . . . . . . . . . . . . . . . .. . . . . . . . . . . . . . . . . . . . . . . . . . . . . .. . . . . . . . . . . . . . . . . . . . . . . . . . . . .. . . . . . . . . . . . . . . . . . . . . . . . . . . . . .. . . . . . . . . . . . . . . . . . . . . . . . . . . . .. . . . . . . . . . . . . . . . . . . . . . . . . . . . . .. . . . . . . . . . . . . . . . . . . . . . . . . . . . .. . . . . . . . . . . . . . . . . . . . . . . . . . . . . .. . . . . . . . . . . . . . . . . . . . . . . . . . . . .. . . . . . . . . . . . . . . . . . . . . . . . . . . . . .. . . . . . . . . . . . . . . . . . . . . . . . . . . . .. . . . . . . . . . . . . . . . . . . . . . . . . . . . . .. . . . . . . . . . . . . . . . . . . . . . . . . . . . .. . . . . . . . . . . . . . . . . . . . . . . . . . . . . .. . . . . . . . . . . . . . . . . . . . . . . . . . . . .. . . . . . . . . . . . . . . . . . . . . . . . . . . . . .. . . . . . . . . . . . . . . . . . . . . . . . . . . . .. . . . . . . . . . . . . . . . . . . . . . . . . . . . . .. . . . . . . . . . . . . . . . . . . . . . . . . . . . .. . . . . . . . . . . . . . . . . . . . . . . . . . . . . .. . . . . . . . . . . . . . . . . . . . . . . . . . . . .. . . . . . . . . . . . . . . . . . . . . . . . . . . . . .. . . . . . . . . . . . . . . . . . . . . . . . . . . . .. . . . . . . . . . . . . . . . . . . . . . . . . . . . . .. . . . . . . . . . . . . . . . . . . . . . . . . . . . .. . . . . . . . . . . . . . . . . . . . . . . . . . . . . .. . . . . . . . . . . . . . . . . . . . . . . . . . . . .. . . . . . . . . . . . . . . . . . . . . . . . . . . . . .. . . . . . . . . . . . . . . . . . . . . . . . . . . . .. . . . . . . . . . . . . . . . . . . . . . . . . . . . . .. . . . . . . . . . . . . . . . . . . . . . . . . . . . .. . . . . . . . . . . . . . . . . . . . . . . . . . . . . .. . . . . . . . . . . . . . . . . . . . . . . . . . . . .

. . . . . . . . . . . . . . . . . . . . . . . . . . .. . . . . . . . . . . . . . . . . . . . . . . . . .

. . . . . . . . . . . . . . . . . . . . . . . . . . .. . . . . . . . . . . . . . . . . . . . . . . . . .. . . . . . . . . . . . . . . . . . . . . . . . . . .. . . . . . . . . . . . . . . . . . . . . . . . . .. . . . . . . . . . . . . . . . . . . . . . . . . . .. . . . . . . . . . . . . . . . . . . . . . . . . .. . . . . . . . . . . . . . . . . . . . . . . . . . .. . . . . . . . . . . . . . . . . . . . . . . . . .. . . . . . . . . . . . . . . . . . . . . . . . . . .. . . . . . . . . . . . . . . . . . . . . . . . . .. . . . . . . . . . . . . . . . . . . . . . . . . . .. . . . . . . . . . . . . . . . . . . . . . . . . .. . . . . . . . . . . . . . . . . . . . . . . . . . .. . . . . . . . . . . . . . . . . . . . . . . . . .. . . . . . . . . . . . . . . . . . . . . . . . . . .. . . . . . . . . . . . . . . . . . . . . . . . . .. . . . . . . . . . . . . . . . . . . . . . . . . . .. . . . . . . . . . . . . . . . . . . . . . . . . .. . . . . . . . . . . . . . . . . . . . . . . . . . .. . . . . . . . . . . . . . . . . . . . . . . . . .. . . . . . . . . . . . . . . . . . . . . . . . . . .. . . . . . . . . . . . . . . . . . . . . . . . . .. . . . . . . . . . . . . . . . . . . . . . . . . . .. . . . . . . . . . . . . . . . . . . . . . . . . .. . . . . . . . . . . . . . . . . . . . . . . . . . .. . . . . . . . . . . . . . . . . . . . . . . . . .. . . . . . . . . . . . . . . . . . . . . . . . . . .. . . . . . . . . . . . . . . . . . . . . . . . . .. . . . . . . . . . . . . . . . . . . . . . . . . . .. . . . . . . . . . . . . . . . . . . . . . . . . .. . . . . . . . . . . . . . . . . . . . . . . . . . .. . . . . . . . . . . . . . . . . . . . . . . . . .

. . . . . . . . . . . . . . . . . . . . . . . . . . . . . . . . . . . . . . . . . . . . . . . . . . . . . . . . . . . . . . . . . .. . . . . . . . . . . . . . . . . . . . . . . . . . . . . . . . . . . . . . . . . . . . . . . . . . . . . . . . . . . . . . . . . .. . . . . . . . . . . . . . . . . . . . . . . . . . . . . . . . . . . . . . . . . . . . . . . . . . . . . . . . . . . . . . . . . .. . . . . . . . . . . . . . . . . . . . . . . . . . . . . . . . . . . . . . . . . . . . . . . . . . . . . . . . . . . . . . . . . .. . . . . . . . . . . . . . . . . . . . . . . . . . . . . . . . . . . . . . . . . . . . . . . . . . . . . . . . . . . . . . . . . .. . . . . . . . . . . . . . . . . . . . . . . . . . . . . . . . . . . . . . . . . . . . . . . . . . . . . . . . . . . . . . . . . .. . . . . . . . . . . . . . . . . . . . . . . . . . . . . . . . . . . . . . . . . . . . . . . . . . . . . . . . . . . . . . . . . .. . . . . . . . . . . . . . . . . . . . . . . . . . . . . . . . . . . . . . . . . . . . . . . . . . . . . . . . . . . . . . . . . .. . . . . . . . . . . . . . . . . . . . . . . . . . . . . . . . . . . . . . . . . . . . . . . . . . . . . . . . . . . . . . . . . .. . . . . . . . . . . . . . . . . . . . . . . . . . . . . . . . . . . . . . . . . . . . . . . . . . . . . . . . . . . . . . . . . .. . . . . . . . . . . . . . . . . . . . . . . . . . . . . . . . . . . . . . . . . . . . . . . . . . . . . . . . . . . . . . . . . .. . . . . . . . . . . . . . . . . . . . . . . . . . . . . . . . . . . . . . . . . . . . . . . . . . . . . . . . . . . . . . . . . .. . . . . . . . . . . . . . . . . . . . . . . . . . . . . . . . . . . . . . . . . . . . . . . . . . . . . . . . . . . . . . . . . .. . . . . . . . . . . . . . . . . . . . . . . . . . . . . . . . . . . . . . . . . . . . . . . . . . . . . . . . . . . . . . . . . .. . . . . . . . . . . . . . . . . . . . . . . . . . . . . . . . . . . . . . . . . . . . . . . . . . . . . . . . . . . . . . . . . .. . . . . . . . . . . . . . . . . . . . . . . . . . . . . . . . . . . . . . . . . . . . . . . . . . . . . . . . . . . . . . . . . .. . . . . . . . . . . . . . . . . . . . . . . . . . . . . . . . . . . . . . . . . . . . . . . . . . . . . . . . . . . . . . . . . .. . . . . . . . . . . . . . . . . . . . . . . . . . . . . . . . . . . . . . . . . . . . . . . . . . . . . . . . . . . . . . . . . .. . . . . . . . . . . . . . . . . . . . . . . . . . . . . . . . . . . . . . . . . . . . . . . . . . . . . . . . . . . . . . . . . .. . . . . . . . . . . . . . . . . . . . . . . . . . . . . . . . . . . . . . . . . . . . . . . . . . . . . . . . . . . . . . . . . .. . . . . . . . . . . . . . . . . . . . . . . . . . . . . . . . . . . . . . . . . . . . . . . . . . . . . . . . . . . . . . . . . .. . . . . . . . . . . . . . . . . . . . . . . . . . . . . . . . . . . . . . . . . . . . . . . . . . . . . . . . . . . . . . . . . .. . . . . . . . . . . . . . . . . . . . . . . . . . . . . . . . . . . . . . . . . . . . . . . . . . . . . . . . . . . . . . . . . .. . . . . . . . . . . . . . . . . . . . . . . . . . . . . . . . . . . . . . . . . . . . . . . . . . . . . . . . . . . . . . . . . .. . . . . . . . . . . . . . . . . . . . . . . . . . . . . . . . . . . . . . . . . . . . . . . . . . . . . . . . . . . . . . . . . .. . . . . . . . . . . . . . . . . . . . . . . . . . . . . . . . . . . . . . . . . . . . . . . . . . . . . . . . . . . . . . . . . .. . . . . . . . . . . . . . . . . . . . . . . . . . . . . . . . . . . . . . . . . . . . . . . . . . . . . . . . . . . . . . . . . .. . . . . . . . . . . . . . . . . . . . . . . . . . . . . . . . . . . . . . . . . . . . . . . . . . . . . . . . . . . . . . . . . .. . . . . . . . . . . . . . . . . . . . . . . . . . . . . . . . . . . . . . . . . . . . . . . . . . . . . . . . . . . . . . . . . .. . . . . . . . . . . . . . . . . . . . . . . . . . . . . . . . . . . . . . . . . . . . . . . . . . . . . . . . . . . . . . . . . .. . . . . . . . . . . . . . . . . . . . . . . . . . . . . . . . . . . . . . . . . . . . . . . . . . . . . . . . . . . . . . . . . .. . . . . . . . . . . . . . . . . . . . . . . . . . . . . . . . . . . . . . . . . . . . . . . . . . . . . . . . . . . . . . . . . .. . . . . . . . . . . . . . . . . . . . . . . . . . . . . . . . . . . . . . . . . . . . . . . . . . . . . . . . . . . . . . . . . .. . . . . . . . . . . . . . . . . . . . . . . . . . . . . . . . . . . . . . . . . . . . . . . . . . . . . . . . . . . . . . . . . .. . . . . . . . . . . . . . . . . . . . . . . . . . . . . . . . . . . . . . . . . . . . . . . . . . . . . . . . . . . . . . . . . .. . . . . . . . . . . . . . . . . . . . . . . . . . . . . . . . . . . . . . . . . . . . . . . . . . . . . . . . . . . . . . . . . .. . . . . . . . . . . . . . . . . . . . . . . . . . . . . . . . . . . . . . . . . . . . . . . . . . . . . . . . . . . . . . . . . .. . . . . . . . . . . . . . . . . . . . . . . . . . . . . . . . . . . . . . . . . . . . . . . . . . . . . . . . . . . . . . . . . .. . . . . . . . . . . . . . . . . . . . . . . . . . . . . . . . . . . . . . . . . . . . . . . . . . . . . . . . . . . . . . . . . .. . . . . . . . . . . . . . . . . . . . . . . . . . . . . . . . . . . . . . . . . . . . . . . . . . . . . . . . . . . . . . . . . .. . . . . . . . . . . . . . . . . . . . . . . . . . . . . . . . . . . . . . . . . . . . . . . . . . . . . . . . . . . . . . . . . .. . . . . . . . . . . . . . . . . . . . . . . . . . . . . . . . . . . . . . . . . . . . . . . . . . . . . . . . . . . . . . . . . .. . . . . . . . . . . . . . . . . . . . . . . . . . . . . . . . . . . . . . . . . . . . . . . . . . . . . . . . . . . . . . . . . .. . . . . . . . . . . . . . . . . . . . . . . . . . . . . . . . . . . . . . . . . . . . . . . . . . . . . . . . . . . . . . . . . .. . . . . . . . . . . . . . . . . . . . . . . . . . . . . . . . . . . . . . . . . . . . . . . . . . . . . . . . . . . . . . . . . .. . . . . . . . . . . . . . . . . . . . . . . . . . . . . . . . . . . . . . . . . . . . . . . . . . . . . . . . . . . . . . . . . .. . . . . . . . . . . . . . . . . . . . . . . . . . . . . . . . . . . . . . . . . . . . . . . . . . . . . . . . . . . . . . . . . .. . . . . . . . . . . . . . . . . . . . . . . . . . . . . . . . . . . . . . . . . . . . . . . . . . . . . . . . . . . . . . . . . .. . . . . . . . . . . . . . . . . . . . . . . . . . . . . . . . . . . . . . . . . . . . . . . . . . . . . . . . . . . . . . . . . .

F (8) F (32)

R ∩ F (6) R ∩ F (16)

Figure 21.1

Nous en déduisons que

Prob (F ) =Card (F )Card (Ω)

=8

40= 0, 2.

Donc l’assertion 1 a) est vraie et l’assertion 1 b) est fausse.

Les événements F et F formant un partition de Ω, nous avons

Prob (F ) + Prob¡F¢= 1.

Donc l’assertion 1 c) est vraie et l’assertion 1 d) est fausse. Sans aucun calcul, nousaurions pu répondre fausse à cette dernière. En effet la probabilité F nulle exprime quecette classe ne comporte aucun garçon.

Nous répondons :

Question 01 : a) Vraie b) Fausse c) Vraie d) Fausse e) Fausse

Nous déduisons de la figure 21.1 que

Prob (R ∩ F ) = Card (R ∩ F )Card (Ω)

=6

40= 0, 15.

Donc l’assertion 2 a) est vraie et l’assertion 2 b) est fausse.

Les deux assertions suivantes sont un contrôle du cours. En effet il y a une confusionfréquente faite entre événements incompatibles et événements indépendants.

F et F étant incompatibles (ie : les ensembles correspondants sont disjoints), il en est demême de R ∩ F et de R ∩ F.Donc l’assertion 2 c) est vraie.

Il est clair que l’assertion 2 d) est fausse car R ∩ F = R ∪ F[1]ce qui est différent de

R ∩ F.

[1] Cette relation est une des deux lois de Morgan. L’autre loi est

R ∪ F = R ∩ F.

134 Questionnements Automatisables

Nous répondons :

Question 02 : a) Vraie b) Fausse c) Vraie d) Fausse e) Fausse

La figure 21.1 permet de répondre immédiatement aux assertions des question 3 et 4.

Card (R ∩ F ) = 6 donc Card ¡R ∩ F¢ =Card (F )− 6 = 2.Card

¡R ∩ F¢ = 16 donc Card ¡R ∩ F¢ =Card ¡F¢− 16 = 16.

Nous en déduisons que seule l’assertion 3 c) est vraie, les autres étant fausse (Parfois àdemi fausses).

Nous répondons :

Question 03 : a) Fausse b) Fausse c) Vraie d) Fausse e) Fausse

Par addition Card (R ∩ F )+Card ¡R ∩ F¢ =Card (R) = 6 + 16 = 22.Nous pouvons aussi obtenir ce résultat à l’aide du calcul proposé à l’assertion 4 a) qui estdonc vraie et par voie de conséquence l’assertion 4 b) est fausse.

Nous en déduisons aussi que l’assertion 4 d) est vraie.

L’assertion 4 c) est stupide, donc fausse.

Nous répondons :

Question 04 : a) Vraie b) Fausse c) Fausse d) Vraie e) Fausse

Les réponses aux assertions des questions 5 et 6 peuvent être obtenues directement avecles cardinaux des événements ou bien avec la formule de Bayes.

L’assertion 5 a) propose la définition d’une probabilité conditionnelle, elle donc acceptée.Nous avons

Prob (F |R) = Prob (F ∩R)Prob (R)

=Card (F ∩R)Card (R)

=6

22, (1)

ce qui permet de rejeter l’assertion 5 b).

L’assertion 5 c) ne donne pas une définition correcte d’une probabilité conditionnelle,toutefois nous pourrions avoir l’égalité.

D’une part, nous avons

Prob¡F ∩R¢

Prob¡R¢ =

Card¡F ∩R¢

Card¡R¢ =

2

18, (2)

et d’autre part

Prob¡F |R¢ = Prob

¡F ∩R¢

Prob (R)=Card

¡F ∩R¢

Card (R)=16

22. (3)

Ce qui permet de conclure que l’assertion 5 c) est fausse et que l’assertion 5 d) est vraie.

Nous constatons ici l’importance d’une figure correcte, elle permet de donner beaucoupde réponses. Il est conseillé d’examiner attentivement les exercices suivants.

Corrigés 135

Nous répondons :

Question 05 : a) Vraie b) Fausse c) Fausse d) Vraie e) Fausse

D’après les résultats (1) et (3) l’assertion 6 a) est vraie et l’assertion 6 b) est fausse.

Prob¡F |R¢ a été calculé en (2) et nous avons

Prob (F |R) + Prob ¡F |R¢ = 6

22+2

18=38

99ce qui établi que l’assertion 6 c) est fausse. De même

Prob¡F |R¢+ Prob ¡F |R¢ = 16

22+2

18=83

99,

ce qui prouve que l’assertion 6 d) est vraie.

Nous répondons :

Question 06 : a) Vraie b) Fausse c) Fausse d) Vraie e) Fausse

22

Il nous est possible de représenter l’ensemble des chaises et des tables en séparant les nondéfectueuses et les défectueuses à l’aide des graphiques suivants :

.

.

.

.

.

.

.

.

.

.

.

.

.

.

.

.

.

.

.

.

.

.

.

.

.

.

.

.

.

.

.

.

.

.

.

.

.

.

.

.

.

.

.

.

.

.

.

.

.

.

.

.

.

.

.

.

.

.

.

.

.

.

.

.

.

.

.

.

.

.

.

.

.

.

.

.

.

.

.

.

.

.

.

.

.

.

.

.

.

.

.

.

.

.

.

.

.

.

.

.

.

.

.

.

.

.

.

.

.

.

.

.

.

.

.

.

.

.

.

.

.

.

.

.

.

.

.

.

.

.

.

.

.

.

.

.

.

.

.

.

.

.

.

.

.

.

.

.

.

.

.

.

.

.

.

.

.

.

.

.

.

.

.

.

.

.

.

.

.

.

.

.

.

.

.

.

.

.

.

.

.

.

.

.

.

.

.

.

.

.

.

.

.

.

.

.

.

.

.

.

.

.

.

.

.

.

.

.

.

.

.

.

.

.

.

.

.

.

.

.

.

.

.

.

.

.

.

.

.

.

.

.

.

.

.

.

.

.

.

.

.

.

.

.

.

.

.

.

.

.

.

.

.

.

.

.

.

.

.

.

.

.

.

.

.

.

.

.

.

.

.

.

.

.

.

.

.

.

.

.

.

.

.

.

.

.

.

.

.

.

.

.

.

.

.

.

.

.

.

.

.

.

.

.

.

.

.

.

.

.

.

.

.

.

.

.

.

.

.

.

.

.

.

.

.

.

.

.

.

.

.

.

.

.

.

.

.

.

.

.

.

.

.

.

.

.

.

.

.

.

.

.

.

.

.

.

.

.

.

.

.

.

.

.

.

.

.

.

.

.

.

.

.

.

.

.

.

.

.

.

.

.

.

.

.

.

.

.

.

.

.

.

.

.

.

.

.

.

.

.

.

.

.

.

.

.

.

.

.

.

.

.

.

.

.

.

.

.

.

.

.

.

.

.

.

.

.

.

.

.

.

.

.

.

.

.

.

.

.

.

.

.

.

.

.

.

.

.

.

.

.

.

.

.

.

.

.

.

.

.

.

.

.

.

.

.

.

.

.

.

.

.

.

.

.

.

.

.

.

.

.

.

.

.

.

.

.

.

.

.

.

.

.

.

.

.

.

.

.

.

.

.

.

.

.

.

.

.

.

.

.

.

.

.

.

.

.

.

.

.

.

.

.

.

.

.

.

.

.

.

.

.

.

.

.

.

.

.

.

.

.

.

.

.

.

.

.

.

.

.

.

.

.

.

.

.

.

.

.

.

.

.

.

.

.

.

.

.

.

.

.

.

.

.

.

.

.

.

.

.

.

.

.

.

.

.

.

.

.

.

.

.

.

.

.

.

.

.

.

.

.

.

.

.

.

.

.

.

.

.

.

.

.

.

.

.

.

.

.

.

.

.

.

.

.

.

.

.

.

.

.

.

.

.

.

.

.

.

.

.

.

.

.

.

.

.

.

.

.

.

.

.

.

.

.

.

.

.

.

.

.

.

.

.

.

.

.

.

.

.

.

.

.

.

.

.

.

.

.

.

.

.

.

.

.

.

.

.

.

.

.

.

.

.

.

.

.

.

.

.

.

.

.

.

.

.

.

.

.

.

.

.

.

.

.

.

.

.

.

.

.

.

.

.

.

.

.

.

.

.

.

.

.

.

.

.

.

.

.

.

.

.

.

.

.

.

.

.

.

.

.

.

.

.

.

.

.

.

.

.

.

.

.

.

.

.

.

.

.

.

.

.

.

.

.

.

.

.

.

.

.

.

.

.

.

.

.

.

.

.

.

.

.

.

.

.

.

.

.

.

.

.

.

.

.

.

.

.

.

.

.

.

.

.

.

.

.

.

.

.

.

.

.

.

.

.

.

.

.

.

.

.

.

.

.

.

.

.

.

.

.

.

.

.

.

.

.

.

.

.

.

.

.

.

.

.

.

.

.

.

.

.

.

.

.

.

.

.

.

.

.

.

.

.

.

.

.

.

.

.

.

.

.

.

.

.

.

.

.

.

.

.

.

.

.

.

.

.

.

.

.

.

.

.

.

.

.

.

.

.

.

.

.

.

.

.

.

.

.

.

.

.

.

.

.

.

.

.

.

.

.

.

.

.

.

.

.

.

.

.

.

.

.

.

.

.

.

.

.

.

.

.

.

.

.

.

.

.

.

.

.

.

.

.

.

.

.

.

.

.

.

.

.

.

.

.

.

.

.

.

.

.

.

.

.

.

.

.

.

.

.

.

.

.

.

.

.

.

.

.

.

.

.

.

.

.

.

.

.

.

.

.

.

.

.

.

.

.

.

.

.

.

.

.

.

.

.

.

.

.

.

.

.

.

.

.

.

.

.

.

.

.

.

.

.

.

.

C (30) C (10)

Chaises..............................

..

..

.

..

.

..

.

..

.

..

.

..

.

..

.

..

.

..

.

.

..

.

..

.

..

.

..

.

..

.

..

.

..

.

..

.

..

..

.

..

..

.

..

.

..

.

..

.

..

.

..

.

..

.

..

.

..

.

.

..

.

..

.

..

.

..

.

..

.

..

.

..

.

..

.

..

..

.

..

..

.

..

.

..

.

..

.

..

.

..

.

..

.

..

.

..

.

.

..

.

..

.

..

.

..

.

..

.

..

.

..

.

..

.

..

..

.

..

..

.

..

.

..

.

..

.

..

.

..

.

..

.

..

.

..

.

.

..

.

..

.

..

.

..

.

..

.

..

.

..

.

..

.

..

..

.

..

..

.

..

.

..

.

..

.

..

.

..

.

..

.

..

.

..

.

.

..

.

..

.

..

.

..

.

..

.

..

.

..

.

..

.

..

..

.

..

..

.

..

.

..

.

..

.

..

.

..

.

..

.

..

.

..

.

.

..

.

..

.

..

.

..

.

..

.

..

.

..

.

..

.

..

..

.

..

..

.

..

.

..

.

..

.

..

.

..

.

..

.

..

.

..

.

.

..

.

..

.

..

.

..

.

..

.

..

.

..

.

..

.

..

..

.

..

..

.

..

.

..

.

..

.

..

.

..

.

..

.

..

.

..

.

.

..

.

..

.

..

.

..

.

..

.

..

.

..

.

..

.

..

..

.

..

..

.

..

.

..

.

..

.

..

.

..

.

..

.

..

.

..

.

.

..

.

..

.

..

.

..

.

..

.

..

.

..

.

..

.

..

..

.

..

..

.

..

.

..

.

..

.

..

.

..

.

..

.

..

.

..

.

.

..

.

..

.

..

.

..

.

..

.

..

.

..

.

..

.

..

..

.

..

..

.

..

.

..

.

..

.

..

.

..

.

..

.

..

.

..

.

.

..

.

..

.

..

.

..

.

..

.

..

.

..

.

..

.

..

..

.

..

..

.

..

.

..

.

..

.

..

.

..

.

..

.

..

.

..

.

.

..

.

..

.

..

.

..

.

..

.

..

.

..

.

..

.

..

..

.

..

..

.

..

.

..

.

..

.

..

.

..

.

..

.

..

.

..

.

.

..

.

..

.

..

.

..

.

..

.

..

.

..

.

..

.

..

..

.

..

..

.

..

.

..

.

..

.

..

.

..

.

..

.

..

.

..

.

.

..

.

..

.

..

.

..

.

..

.

..

.

..

.

..

.

..

..

.

..

..

.

..

.

..

.

..

.

..

.

..

.

..

.

..

.

..

.

.

..

.

..

.

..

.

..

.

..

.

..

.

..

.

..

.

..

..

.

..

..

.

..

.

..

.

..

.

..

.

..

.

..

.

..

.

..

.

.

..

.

..

.

..

.

..

.

..

.

..

.

..

.

..

.

..

..

.

..

..

.

..

.

..

.

..

.

..

.

..

.

..

.

..

.

..

.

.

..

.

..

.

..

.

..

.

..

.

..

.

..

.

..

.

..

..

.

..

..

.

..

.

..

.

..

.

..

.

..

.

..

.

..

.

..

.

.

..

.

..

.

..

.

..

.

..

.

..

.

..

.

..

.

..

..

.

..

..

.

..

.

..

.

..

.

..

.

..

.

..

.

..

.

..

.

.

..

.

..

.

..

.

..

.

..

.

..

.

..

.

..

.

..

..

.

..

..

.

..

.

..

.

..

.

..

.

..

.

..

.

..

.

..

.

.

..

.

..

.

..

.

..

.

..

.

..

.

..

.

..

.

..

..

.

..

..

.

..

.

..

.

..

.

..

.

..

.

..

.

..

.

..

.

.

..

.

..

.

..

.

..

.

..

.

..

.

..

.

..

.

..

..

.

..

..

.

..

.

..

.

..

.

..

.

..

.

..

.

..

.

..

.

.

..

.

..

.

..

.

..

.

..

.

..

.

..

.

..

.

..

..

.

..

..

.

..

.

..

.

..

.

..

.

..

.

..

.

..

.

..

.

.

..

.

..

.

..

.

..

.

..

.

..

.

..

.

..

.

..

..

.

..

..

.

..

.

..

.

..

.

..

.

..

.

..

.

..

.

..

.

.

..

.

..

.

..

.

..

.

..

.

..

.

..

.

..

.

..

..

.

..

..

.

..

.

..

.

..

.

..

.

..

.

..

.

..

.

..

.

.

..

.

..

.

..

.

..

.

..

.

..

.

..

.

..

.

..

..

.

..

..

.

..

.

..

.

..

.

..

.

..

.

..

.

..

.

..

.

.

.

.

.

.

.

.

.

.

.

.

.

.

.

.

.

.

.

.

.

.

.

.

.

.

.

.

.

.

.

.

.

.

.

.

.

.

.

.

.

.

.

.

.

.

.

.

.

.

.

.

.

.

.

.

.

.

.

.

.

.

.

.

.

.

.

.

.

.

.

.

.

.

.

.

.

.

.

.

.

.

.

.

.

.

.

.

.

.

.

.

.

.

.

.

.

.

.

.

.

.

.

.

.

.

.

.

.

.

.

.

.

.

.

.

.

.

.

.

.

.

.

.

.

.

.

.

.

.

.

.

.

.

.

.

.

.

.

.

.

.

.

.

.

.

.

.

.

.

.

.

.

.

.

.

.

.

.

.

.

.

.

.

.

.

.

.

.

.

.

.

.

.

.

.

.

.

.

.

.

.

.

.

.

.

.

.

.

.

.

.

.

.

.

.

.

.

.

.

.

.

.

.

.

.

.

.

.

.

.

.

.

.

.

.

.

.

.

.

.

.

.

.

.

.

.

.

.

.

.

.

.

.

.

.

.

.

.

.

.

.

.

.

.

.

.

.

.

.

.

.

.

.

.

.

.

.

.

.

.

.

.

.

.

.

.

.

.

.

.

.

.

.

.

.

.

.

.

.

.

.

.

.

.

.

.

.

.

.

.

.

.

.

.

.

.

.

.

.

.

.

.

.

.

.

.

.

.

.

.

.

.

.

.

.

.

.

.

.

.

.

.

.

.

.

.

.

.

.

.

.

.

.

.

.

.

.

.

.

.

.

.

.

.

.

.

.

.

.

.

.

.

.

.

.

.

.

.

.

.

.

.

.

.

.

.

.

.

.

.

.

.

.

.

.

.

.

.

.

.

.

.

.

.

.

.

.

.

.

.

.

.

.

.

.

.

.

.

.

.

.

.

.

.

.

.

.

.

.

.

.

.

.

.

.

.

.

.

T (8) T (2)

Tables

Figure 22.1

Nous en déduisons

Prob (C) =30

40=3

4et Prob

¡C¢=10

40=1

4(1)

Prob (T ) =8

10=4

5et Prob

¡T¢=2

10=1

5. (2)

Ce qui permet de conclure que toutes les assertions proposées à la question 1 sont fausses,donc nous devons répondre vraie à la case 1 e).

Nous répondons :

Question 01 : a) Fausse b) Fausse c) Fausse d) Fausse e) Vraie

La question 2 est une question de cours. Nous acceptons les assertions 2 a) et 2 b).Remarquer la présentation de cette dernière assertion, il faut savoir lire ... pour répondrecorrectement aux Questionnements Automatisables.

136 Questionnements Automatisables

Nous pouvons rejeter les assertions 2 c) et 2 d) car ce type de QCM admet au plus deuxréponses exactes, toutefois les assertions 2 c) et 2 d) sont “impossibles”.

Nous répondons :

Question 02 : a) Vraie b) Vraie c) Fausse d) Fausse e) Fausse

Les événements C ∩ T et C ∩ T forment une partition de C ce qui implique que

Prob (C ∩ T ) + Prob ¡C ∩ T¢ = Prob (C) = 3

4,

d’après le résultat du (1) . Ainsi les assertions 3 a) et 3 b) sont fausses. Remarquer qu’uneinversion de C et de C aurait transformé les assertions fausses en vraies. Dur -dur ...pourles étourdis.

De manière analogue, les événements C ∩ T et C ∩ T forment une partition de T ce quiimplique que

Prob¡C ∩ T¢+ Prob (C ∩ T ) = Prob (T ) = 4

5,

d’après le résultat du (2) . Ainsi l’assertion 3 c) est fausse et l’assertion 3 d) vraie.

Nous aurions pu d’une autre manière obtenir ces résultats.

En effet les événements C et T, C et T, C et T et C et T sont indépendants, c’est uneévidence car il n’y a aucune incidence entre les choix d’une table et d’une chaise.

Nous en déduisons que les probabilités des conjonctions de ces événements est le produitdes probabilités des événements.

Ainsi par exemple :

Prob (C ∩ T ) + Prob ¡C ∩ T¢ = Prob (C) ∗ Prob (T ) + Prob (C) ∗ Prob ¡T¢=

3

4∗ 45+3

4∗ 15=3

4,

et de même pour Prob (T ) .

Nous répondons :

Question 03 : a) Fausse b) Fausse c) Fausse d) Vraie e) Fausse

Nous pouvons représenter les quatre situations possibles

C ∩ T : La chaise et la table sont toutes deux non défectueuses.

C ∩ T : La chaise et la table sont toutes deux défectueuses.

C ∩ T : La chaise est défectueuse et la table non défectueuse.

C ∩ T : La chaise est non défectueuse et la table défectueuse.sous la forme de la figure suivante, dans la partie de gauche en faisant une partitionde la fabrication journalière et dans la partie de droite à l’aide d’un arbre stochastiquereprésentant les différentes situations.

Nous avons entouré d’un cercle le chemin correspondant à C ∩ T dont la probabilité estnotée p2.

Corrigés 137

. . . . . . . . . . . . . . .. . . . . . . . . . . . . .. . . . . . . . . . . . . . .. . . . . . . . . . . . . .. . . . . . . . . . . . . . .. . . . . . . . . . . . . .. . . . . . . . . . . . . . .. . . . . . . . . . . . . .. . . . . . . . . . . . . . .. . . . . . . . . . . . . .. . . . . . . . . . . . . . .. . . . . . . . . . . . . .. . . . . . . . . . . . . . .. . . . . . . . . . . . . .. . . . . . . . . . . . . . .. . . . . . . . . . . . . .. . . . . . . . . . . . . . .. . . . . . . . . . . . . .. . . . . . . . . . . . . . .. . . . . . . . . . . . . .. . . . . . . . . . . . . . .. . . . . . . . . . . . . .. . . . . . . . . . . . . . .. . . . . . . . . . . . . .. . . . . . . . . . . . . . .. . . . . . . . . . . . . .. . . . . . . . . . . . . . .. . . . . . . . . . . . . .. . . . . . . . . . . . . . .

. . . . . . . . . .. . . . . . . . .. . . . . . . . . .. . . . . . . . .. . . . . . . . . .. . . . . . . . .. . . . . . . . . .. . . . . . . . .. . . . . . . . . .. . . . . . . . .. . . . . . . . . .. . . . . . . . .. . . . . . . . . .. . . . . . . . .. . . . . . . . . .. . . . . . . . .. . . . . . . . . .. . . . . . . . .. . . . . . . . . .

.............................................................................................................................................................................................................................................................................................................................................................................................................................................................................................................................................................................................................................................................................................................................................................................................................................................................................................................................................................................................................................................................................................................................................................................................................................................................................................................................................................................................................................................................................................................................................................................................................................................................................................................................................................................................................

. . . . . . . . . . . . . . . . . . . .. . . . . . . . . . . . . . . . . . .. . . . . . . . . . . . . . . . . . . .. . . . . . . . . . . . . . . . . . .. . . . . . . . . . . . . . . . . . . .. . . . . . . . . . . . . . . . . . .. . . . . . . . . . . . . . . . . . . .. . . . . . . . . . . . . . . . . . .. . . . . . . . . . . . . . . . . . . .. . . . . . . . . . . . . . . . . . .. . . . . . . . . . . . . . . . . . . .. . . . . . . . . . . . . . . . . . .. . . . . . . . . . . . . . . . . . . .. . . . . . . . . . . . . . . . . . .. . . . . . . . . . . . . . . . . . . .. . . . . . . . . . . . . . . . . . .. . . . . . . . . . . . . . . . . . . .. . . . . . . . . . . . . . . . . . .. . . . . . . . . . . . . . . . . . . .. . . . . . . . . . . . . . . . . . .. . . . . . . . . . . . . . . . . . . .. . . . . . . . . . . . . . . . . . .. . . . . . . . . . . . . . . . . . . .. . . . . . . . . . . . . . . . . . .. . . . . . . . . . . . . . . . . . . .. . . . . . . . . . . . . . . . . . .. . . . . . . . . . . . . . . . . . . .. . . . . . . . . . . . . . . . . . .. . . . . . . . . . . . . . . . . . . .. . . . . . . . . . . . . . . . . . .. . . . . . . . . . . . . . . . . . . .. . . . . . . . . . . . . . . . . . .. . . . . . . . . . . . . . . . . . . .. . . . . . . . . . . . . . . . . . .. . . . . . . . . . . . . . . . . . . .. . . . . . . . . . . . . . . . . . .. . . . . . . . . . . . . . . . . . . .. . . . . . . . . . . . . . . . . . .. . . . . . . . . . . . . . . . . . . .

C ∩ T (12) C ∩ T (4)

C ∩ T (3) C ∩ T (1)

C

T

T

C

T

T

..............................................................................................................................................................................................

........................................................

........................................................

.................................................................

....................................................................................................................................................... ..........................

.................................................................................................................................................................... ..........................

....................................................................................................................................................... ..........................

........................................................

........................................................

.................................................................

...........................................................................................................

...........................................................................................................

3/41/5

Figure 22.2

p3 correspond à l’événement C ∩ T et p1 correspond à la réunion des deux événementsC ∩ T et C ∩ T.Nous avons donc

p2 = Prob¡C ∩ T¢ = Prob (C) ∗ Prob ¡T¢ = 3

4

1

5=3

20

p3 = Prob¡C ∩ T¢ = Prob ¡C¢ ∗ Prob (T ) = 1

4

4

5=4

20

p1 = Prob¡C ∩ T¢+ Prob ¡C ∩ T¢ = p2 + p3 =

7

20

ce qui permet de répondre à toutes les assertions des questions 04, 05 et 06.

Les assertions 4 a), 4 d), 5 d), 6 a) et 6 b) sont vraies et les autres fausses.

Nous répondons :

Question 04 : a) Vraie b) Fausse c) Fausse d) Vraie e) Fausse

Question 05 : a) Fausse b) Fausse c) Fausse d) Vraie e) Fausse

Question 06 : a) Vraie b) Vraie c) Fausse d) Fausse e) Fausse

23

Nous pouvons représenter sur la figure 23.1, une boîte contenant les k + 3 boules enséparant à l’aide de pointillé les 3 boules blanches notées par un rond ° et les k boulesrouges notées par un rond pointé de noir °• .Nous avons représenté une éventualité possible formée d’une boule blanche et d’une boulerouge.

138 Questionnements Automatisables

............................................................................................................................................................................................................................................................................................................Une éventualité

1 blanche et 1 rouge

................................................................................................................................................................

°

°

°

°

°

°

°

°

°

...

...

...

3 blanches k rouges

......................................................................................................................................................

..................................

....................................................................................

Figure 23.1

Nous examinons les deux cas de tirages possibles : les tirages avec remise ou tiragesindépendants et les tirages sans remise ou tirages exhaustifs.

Tirages indépendants :

Ce qualificatif est utilisé car si après tirage d’une boule nous remettons cette boule dansla boîte alors le deuxième tirage est réalisé dans les mêmes conditions que le premier.

Un tirage est une 2-liste ou encore un couple formé d’une première boule choisie au hasardparmi les k + 3 boules de la boîte et d’une deuxième boules choisie également au hasardparmi les k + 3 boules de la boîte.

Le nombre des éventualités, ou nombre des tirages possibles, ou cardinal de l’univers estdonc de

(k + 3)2 .

Nous en déduisons que seule l’assertion 1 d) est vraie, les autres étant fausses.

ATTENTION : Les autres propositions sont attractives si nous n’avons pas de rigueur.Il faut bien comprendre l’énoncé en probabilité et surtout ne pas introduire d’idéesétrangères. En un mot il faut savoir lire, écrire, compter et comprendre ... c’est bienle moins pour un bachelier.

Nous répondons :

Question 01 : a) Fausse b) Fausse c) Fausse d) Vraie e) Fausse

Un tirage sera bicolore si la première boule tirée est rouge et la seconde blanche ouinversement si la première boule tirée est blanche et la seconde rouge.

La cardinal de cet événement E est

Card (E) = 2 ∗ 3 k = 6 ket la probabilité de cet événement est

pk = Prob (E) =6 k

(k + 3)2. (1)

Nous en déduisons que limk→+∞Prob (E) = 0 ce qui est logique car si k est infini laprobabilité de choisir une boule blanche est nulle par rapport à celle de choisir une boulerouge. L’assertion 2 a) est donc fausse.

Nous étudions la fonction de la variable réelle x définie par

f(x) =6x

(x+ 3)2,

qui admet pour dérivée

f 0(x) = −6 x− 3(x+ 3)3

et pour tableau des variations

Corrigés 139

..............................................................................................................................................................................................................................................................................................................................................................................................................................................................................................................................................

..............................................................................................................................................................................................................................................................................................................................................................................................................................................................................................................................................

........

........

........

........

........

........

........

........

........

........

........

........

........

........

........

........

........

........

........

........

........

........

........

........

........

........

........

........

........

........

........

...

x 0 3 +∞

2 / 3 0

0

1 / 2

0

f 0

f

+ −

..................................................................................................................................................................................................................................................

..................................................................................................................................................................................................................................................

Ce qui prouve que les assertions 2 b) et 2 d) sont fausse car pour tout k > 2 nous avonsProb (E) 6 1

2et que l’assertion 2 c) est vraie car il existe une et une seule valeur k = 3

telle Prob (E) =1

2.

Nous répondons :

Question 02 : a) Fausse b) Fausse c) Vraie d) Fausse e) Fausse

Tirages exhaustifs :

Nous avons dans ce cas ce que nous appelons un modèle hypergéométrique. Soit le choixaléatoire de n = 2 boules prises simultanément dans un boîte contenant N = k+3 boules.Si nous considérons la variable aléatoire X égale au nombre de boules blanches alors X

suit la loi HµN = k + 3, n = 2, p =

3

k + 3

¶.

L’événement E n’est autre que l’événement (X = 1) .

Le nombre d’éventualités est le nombre de choix possibles de 2 boules parmi les k+3 soit

Card (Ω) = C2k+3 =

k + 3

2

k + 2

1.

Ce qui prouve que l’assertion 3 a) est vraie.

Il est aisé d’établir que le produit de deux entiers consécutifs est pair car il peut être écrit

2n (2n− 1) ou (2n+ 1) 2n

ce qui prouve que l’assertion 3 b) est fausse. Il faut remarquer que dans cette assertion, il

est précisé que(k + 3) (k + 2)

2est un entier pour certaines valeurs de k. Cette propriété

est vraie si nous donnons à k des valeurs particulières, ce qui ne prouve rien pour kquelconque. Il faut ce méfier des assertions qui sont en partie vraie. C’est pour de tellesréponses que les correcteurs jugent, ceux qui réfléchissent et les autres ...

Nous en déduisons aussi que les assertions 3 c) et 3 d) sont fausses. Il faut bien lire lelibellé de l’assertion 3 d), car elle serait acceptée si nous tenions compte de l’ordre dutirage ...

Nous répondons :

Question 03 : a) Vraie b) Fausse c) Fausse d) Fausse e) Fausse

Pour k quelconque le nombre d’éventualités correspondant au tirage d’une blanche etd’une rouge est

Card (E) = C13 C

1k = 3 k

car nous devons choisir une blanche parmi les 3 et une rouge parmi les k.

140 Questionnements Automatisables

Nous en déduisons [1] que

pk =Card (E)Card (Ω)

=6 k

(k + 3) (k + 2).

Si k = 5 alors nous avons

p =15

28.

Ce qui prouve que seule l’assertion 4 d) est vraie. Il faut remarquer ici les pièges duconcepteur du sujet. Que c’est dur dur les Questionnements Automatisables pourles étourdis. Il savoir éliminer les assertions douteuses.

Nous répondons :

Question 04 : a) Fausse b) Fausse c) Fausse d) Vraie e) Fausse

24

Nous pouvons représenter l’univers Ω des étudiants à l’aide du système completd’événements suivant :

A ∩B : Etudiants pratiquant que le sport A

A ∩B : Etudiants pratiquant que le sport B (1)

A ∩B : Etudiants pratiquant les deux sports A et B

A ∪B : Etudiants ne pratiquant aucun sport.

Les hypothèses donnent

Prob (A) = 0, 40; Prob (B) = 0, 25 et Prob (A ∩B) = 0, 10. (2)

Nous déduisons de la formule des probabilités totales [2] et de (2) que

Prob (A ∪B) = Prob (A) + Prob (B)− Prob (A ∩B)= 0, 40 + 0, 25− 0, 10= 0, 55 = 1− Prob ¡A ∪B¢ , (3)

ce qui donne les probabilités des événements du système complet d’événements (1) :

Prob¡A ∩B¢ = 0, 30; Prob

¡A ∩B¢ = 0, 15; Prob (A ∩B) = 0, 10 et

Prob¡A ∪B¢ = 0, 45 (4)

et nous obtenons ainsi la figure 24.1.

[1] Remarquer la différence de cette probabilté avec celle de la relation (1) .[2] Cette formule est un cas particulier de la formule du crible de Poincaré qui permet de calculer laprobabilité de la réunion d’événements. Par exemple pour trois événements A, B et C

Prob (A ∪B ∪ C) = Prob (A) + Prob (B) + Prob (C)

−Prob (A ∩B)− Prob (B ∩ C)− Prob (A ∩ C)+Prob (A ∩B ∩ C)

Corrigés 141

. . . . . . . . . . . . . . . . . . . . . .. . . . . . . . . . . . . . . . . . . . . .. . . . . . . . . . . . . . . . . . . . . .. . . . . . . . . . . . . . . . . . . . . .. . . . . . . . . . . . . . . . . . . . . .. . . . . . . . . . . . . . . . . . . . . .. . . . . . . . . . . . . . . . . . . . . .. . . . . . . . . . . . . . . . . . . . . .. . . . . . . . . . . . . . . . . . . . . .. . . . . . . . . . . . . . . . . . . . . .. . . . . . . . . . . . . . . . . . . . . .. . . . . . . . . . . . . . . . . . . . . .. . . . . . . . . . . . . . . . . . . . . .. . . . . . . . . . . . . . . . . . . . . .. . . . . . . . . . . . . . . . . . . . . .. . . . . . . . . . . . . . . . . . . . . .. . . . . . . . . . . . . . . . . . . . . .. . . . . . . . . . . . . . . . . . . . . .. . . . . . . . . . . . . . . . . . . . . .. . . . . . . . . . . . . . . . . . . . . .. . . . . . . . . . . . . . . . . . . . . .. . . . . . . . . . . . . . . . . . . . . .. . . . . . . . . . . . . . . . . . . . . .. . . . . . . . . . . . . . . . . . . . . .. . . . . . . . . . . . . . . . . . . . . .. . . . . . . . . . . . . . . . . . . . . .. . . . . . . . . . . . . . . . . . . . . .. . . . . . . . . . . . . . . . . . . . . .. . . . . . . . . . . . . . . . . . . . . .

. . . . . . . . . . . . . . .. . . . . . . . . . . . . .. . . . . . . . . . . . . . .. . . . . . . . . . . . . .. . . . . . . . . . . . . . .. . . . . . . . . . . . . .. . . . . . . . . . . . . . .. . . . . . . . . . . . . .. . . . . . . . . . . . . . .. . . . . . . . . . . . . .. . . . . . . . . . . . . . .. . . . . . . . . . . . . .. . . . . . . . . . . . . . .. . . . . . . . . . . . . .. . . . . . . . . . . . . . .. . . . . . . . . . . . . .. . . . . . . . . . . . . . .. . . . . . . . . . . . . .. . . . . . . . . . . . . . .

..................................................................................................................................................................................................................................................................................................................................................................................................................................................................................................................................................................................................................................................................................................................................................................................................................................................................................................................................................................................................................................................................................................................................................................................................................................................................................................................................................................................................................................................................................................................................................................................................................................................................................................................................................................................................................................................................................................................................................................................................................................................................................................................................................................................................................................................................................................................................................................................................................................................................................................................................................................................................................................................................................................................................

. . . . . . . . . . . . . . . . . . . . . .. . . . . . . . . . . . . . . . . . . . . .. . . . . . . . . . . . . . . . . . . . . .. . . . . . . . . . . . . . . . . . . . . .. . . . . . . . . . . . . . . . . . . . . .. . . . . . . . . . . . . . . . . . . . . .. . . . . . . . . . . . . . . . . . . . . .. . . . . . . . . . . . . . . . . . . . . .. . . . . . . . . . . . . . . . . . . . . .. . . . . . . . . . . . . . . . . . . . . .. . . . . . . . . . . . . . . . . . . . . .. . . . . . . . . . . . . . . . . . . . . .. . . . . . . . . . . . . . . . . . . . . .. . . . . . . . . . . . . . . . . . . . . .. . . . . . . . . . . . . . . . . . . . . .. . . . . . . . . . . . . . . . . . . . . .. . . . . . . . . . . . . . . . . . . . . .. . . . . . . . . . . . . . . . . . . . . .. . . . . . . . . . . . . . . . . . . . . .. . . . . . . . . . . . . . . . . . . . . .. . . . . . . . . . . . . . . . . . . . . .. . . . . . . . . . . . . . . . . . . . . .. . . . . . . . . . . . . . . . . . . . . .. . . . . . . . . . . . . . . . . . . . . .. . . . . . . . . . . . . . . . . . . . . .. . . . . . . . . . . . . . . . . . . . . .. . . . . . . . . . . . . . . . . . . . . .. . . . . . . . . . . . . . . . . . . . . .. . . . . . . . . . . . . . . . . . . . . .

A ∩B (30%) A ∩B (15%)

A ∩B (10%) A ∪B (45%)

Figure 24.1

Ainsi l’assertion 1 d) est vraie et les autres fausses. Vérifions le. La relation (2) prouveque l’assertion 1 a) est fausse. Les assertions 1 b) et 1 c) ont la même valeur car lesévénements

A ∪B et A ∩Bsont identiques (loi de DeMorgan). Nous déduisons encore de (2) que leur valeur communeest : 1− 0, 10 = 0, 90.Nous avons les réponses :

Question 01 : a) Fausse b) Fausse c) Fausse d) Vraie e) Fausse

La relation (3) donne la probabilité p1 pour que l’étudiant lambda pratique au moins undes deux sports A ou/et B. L’assertion 2 b) est exacte, les autres assertions sont sûrementfausses. ATTENTION : Si votre calculatrice est programmée en virgule flottante vous

pouvez obtenir5

9= 0, 55 ce qui donne une erreur.

Nous avons les réponses :

Question 02 : a) Fausse b) Vraie c) Fausse d) Fausse e) Fausse

La probabilité p2 pour que l’étudiant lambda ne pratique qu’un seul sport est

p2 = Prob¡A ∩B¢+ Prob ¡A ∩B¢

= 0, 30 + 0, 15

= 0, 45.

Nous en déduisons que l’assertion 3 b) est vraie et que toutes les assertions de la question4 sont fausses. Nous répondrons Vraie à la case 4 e).

Nous examinons les autres propositions de la question 3.

Prob¡A ∪B¢ = 0, 45 d’après (3)

donc l’assertion 3 a) est vraie. Dans ce type de QCM, il y a au plus deux assertionsexactes, il est en principe inutile de contrôler les autres assertions. Toutefois, pour leplaisir, nous allons le faire.

La formule du crible de Poincaré donne

Prob (A ∪B) + Prob (A ∩B) = Prob (A) + Prob (B) = 0, 65.Ce qui prouve que les assertions 3 c) et 3 d) sont fausses.

Nous répondons :

Question 03 : a) Vraie b) Vraie c) Fausse d) Fausse e) Fausse

142 Questionnements Automatisables

Question 04 : a) Fausse b) Fausse c) Fausse d) Fausse e) Vraie

La probabilité p3 pour que l’étudiant lambda ne pratique que le sport A est

p3 = Prob¡A ∩B¢

= Prob (A)− Prob (A ∩B)= 0, 30.

Nous en déduisons que les assertions 5 b) et 5 c) sont vraies et que les autres assertions5 a) et 5 d) sont fausses.

Nous répondons :

Question 05 : a) Fausse b) Vraie c) Vraie d) Fausse e) Fausse

Le calcul de la probabilité p4 est obtenu à partir de celui de p3 en échangeant A et B.

p4 = Prob¡A ∩B¢

= Prob (B)− Prob (A ∩B)= 0, 15.

L’assertion 6 d) est donc vraie et les autres assertions 6 a), 6 b) et 6 c) sont fausses.

Nous répondons :

Question 06 : a) Fausse b) Fausse c) Fausse d) Vraie e) Fausse

Si le nombre total des étudiants est de 360, alors en utilisant (4) nous avons la répartitionsuivante donnée par la figure 24.2.

. . . . . . . . . . . . . . . . . . . . . .. . . . . . . . . . . . . . . . . . . . . .. . . . . . . . . . . . . . . . . . . . . .. . . . . . . . . . . . . . . . . . . . . .. . . . . . . . . . . . . . . . . . . . . .. . . . . . . . . . . . . . . . . . . . . .. . . . . . . . . . . . . . . . . . . . . .. . . . . . . . . . . . . . . . . . . . . .. . . . . . . . . . . . . . . . . . . . . .. . . . . . . . . . . . . . . . . . . . . .. . . . . . . . . . . . . . . . . . . . . .. . . . . . . . . . . . . . . . . . . . . .. . . . . . . . . . . . . . . . . . . . . .. . . . . . . . . . . . . . . . . . . . . .. . . . . . . . . . . . . . . . . . . . . .. . . . . . . . . . . . . . . . . . . . . .. . . . . . . . . . . . . . . . . . . . . .. . . . . . . . . . . . . . . . . . . . . .. . . . . . . . . . . . . . . . . . . . . .. . . . . . . . . . . . . . . . . . . . . .. . . . . . . . . . . . . . . . . . . . . .. . . . . . . . . . . . . . . . . . . . . .. . . . . . . . . . . . . . . . . . . . . .. . . . . . . . . . . . . . . . . . . . . .. . . . . . . . . . . . . . . . . . . . . .. . . . . . . . . . . . . . . . . . . . . .. . . . . . . . . . . . . . . . . . . . . .. . . . . . . . . . . . . . . . . . . . . .. . . . . . . . . . . . . . . . . . . . . .

. . . . . . . . . . . . . . .. . . . . . . . . . . . . .. . . . . . . . . . . . . . .. . . . . . . . . . . . . .

. . . . . . . . . . . . . . .. . . . . . . . . . . . . .. . . . . . . . . . . . . . .. . . . . . . . . . . . . .. . . . . . . . . . . . . . .. . . . . . . . . . . . . .. . . . . . . . . . . . . . .. . . . . . . . . . . . . .. . . . . . . . . . . . . . .. . . . . . . . . . . . . .. . . . . . . . . . . . . . .. . . . . . . . . . . . . .. . . . . . . . . . . . . . .. . . . . . . . . . . . . .. . . . . . . . . . . . . . .

..................................................................................................................................................................................................................................................................................................................................................................................................................................................................................................................................................................................................................................................................................................................................................................................................................................................................................................................................................................................................................................................................................................................................................................................................................................................................................................................................................................................................................................................................................................................................................................................................................................................................................................................................................................................................................................................................................................................................................................................................................................................................................................................................................................................................................................................................................................................................................................................................................................................................................................................................................................................................................................................................................................................................

. . . . . . . . . . . . . . . . . . . . . .. . . . . . . . . . . . . . . . . . . . . .. . . . . . . . . . . . . . . . . . . . . .. . . . . . . . . . . . . . . . . . . . . .. . . . . . . . . . . . . . . . . . . . . .. . . . . . . . . . . . . . . . . . . . . .. . . . . . . . . . . . . . . . . . . . . .. . . . . . . . . . . . . . . . . . . . . .. . . . . . . . . . . . . . . . . . . . . .. . . . . . . . . . . . . . . . . . . . . .. . . . . . . . . . . . . . . . . . . . . .. . . . . . . . . . . . . . . . . . . . . .. . . . . . . . . . . . . . . . . . . . . .. . . . . . . . . . . . . . . . . . . . . .. . . . . . . . . . . . . . . . . . . . . .. . . . . . . . . . . . . . . . . . . . . .. . . . . . . . . . . . . . . . . . . . . .. . . . . . . . . . . . . . . . . . . . . .. . . . . . . . . . . . . . . . . . . . . .. . . . . . . . . . . . . . . . . . . . . .. . . . . . . . . . . . . . . . . . . . . .. . . . . . . . . . . . . . . . . . . . . .. . . . . . . . . . . . . . . . . . . . . .. . . . . . . . . . . . . . . . . . . . . .. . . . . . . . . . . . . . . . . . . . . .. . . . . . . . . . . . . . . . . . . . . .. . . . . . . . . . . . . . . . . . . . . .. . . . . . . . . . . . . . . . . . . . . .. . . . . . . . . . . . . . . . . . . . . .

A ∩B (108) A ∩B (54)

A ∩B (36) A ∪B (162)

Figure 24.2

Nous en déduisons que les assertions 7 b) et 7 c) sont vraies et que les assertions 7 a) et7 d) sont fausses.

Nous répondons :

Question 07 : a) Fausse b) Vraie c) Vraie d) Fausse e) Fausse

TABLE DES MATIÈRES

Pour chaque exercice, le premier numéro de page est celui de l’énoncé et le deuxième celuidu corrigé.

Page

Introduction . . . . . . . . . . . . . . . . . . . . . . . . . . . . . . . . . . . . . . . . . . . . . . . . . . 3

Quelques conseils . . . . . . . . . . . . . . . . . . . . . . . . . . . . . . . . . . . . . . . . . . . . . 4

Un peu de méthode . . . . . . . . . . . . . . . . . . . . . . . . . . . . . . . . . . . . . . . . . . 5

01 : Un peu de logique . . . . . . . . . . . . . . . . . . . . . . . . . . . . . . . . . . . . . . . . . . . . 13 / 47

02 : Barycentre et nombres complexes . . . . . . . . . . . . . . . . . . . . . . . . . . . . . 14 / 49

03 : Barycentre. Fonction scalaire de Leibniz . . . . . . . . . . . . . . . . . . . . . . 15 / 57

04 : Propriétés géométriques d’aires . . . . . . . . . . . . . . . . . . . . . . . . . . . . . . . 16 / 66

05 : Géométrie analytique . . . . . . . . . . . . . . . . . . . . . . . . . . . . . . . . . . . . . . . . . 18 / 71

06 : Isométries dans un triangle . . . . . . . . . . . . . . . . . . . . . . . . . . . . . . . . . . . 19 / 73

07 : Théorème de Céva . . . . . . . . . . . . . . . . . . . . . . . . . . . . . . . . . . . . . . . . . . . . 20 / 77

08 : Etude affine d’un quadrilatère . . . . . . . . . . . . . . . . . . . . . . . . . . . . . . . . 22 / 82

09 : Similitudes dans C . . . . . . . . . . . . . . . . . . . . . . . . . . . . . . . . . . . . . . . . . . . 24 / 89

10 : Equation du quatrième degré dans C . . . . . . . . . . . . . . . . . . . . . . . . . 25 / 92

11 : Formules de trigonométrie . . . . . . . . . . . . . . . . . . . . . . . . . . . . . . . . . . . . 26 / 96

12 : Equation du troisième degré dans C . . . . . . . . . . . . . . . . . . . . . . . . . . 27 / 97

13 : Module et argument de nombres complexes . . . . . . . . . . . . . . . . . . . 28 / 105

144 Questionnements Automatisables

14 : Transformation homographique dans C . . . . . . . . . . . . . . . . . . . . . . . 29 / 107

15 : Construction d’un pentagone régulier avec la règle et le compas 30 / 109

16 : Equation du troisième degré dans C . . . . . . . . . . . . . . . . . . . . . . . . . . 31 / 114

17 : Racines cubiques de l’unité . . . . . . . . . . . . . . . . . . . . . . . . . . . . . . . . . . . 32 / 118

18 : Résolution d’un problème de géométrie par l’analytique et parles complexes . . . . . . . . . . . . . . . . . . . . . . . . . . . . . . . . . . . . . . . . . . . . . . . . .

34 / 121

19 : Transformation homographique dans C . . . . . . . . . . . . . . . . . . . . . . . 36 / 126

20 : Partitions et probabilités . . . . . . . . . . . . . . . . . . . . . . . . . . . . . . . . . . . . . 38 / 129

21 : Probabilités conditionnelles . . . . . . . . . . . . . . . . . . . . . . . . . . . . . . . . . . . 40 / 132

22 : Arbres stochastiques . . . . . . . . . . . . . . . . . . . . . . . . . . . . . . . . . . . . . . . . . . 41 / 135

23 : Dénombrement et probabilités . . . . . . . . . . . . . . . . . . . . . . . . . . . . . . . . 42 / 137

24 : Partitions et probabilités . . . . . . . . . . . . . . . . . . . . . . . . . . . . . . . . . . . . . 43 / 140